You are on page 1of 192

FLUID MECHANICS

For
MECHANICAL ENGINEERING
CIVIL ENGINEERING
FLUID MECHANICS
SYLLABUS
Fluid properties; fluid statics, manometry, buoyancy, forces on submerged bodies,
stability of floating bodies; control-volume analysis of mass, momentum and energy;
fluid acceleration; differential equations of continuity and momentum; Bernoulli’s
equation; dimensional analysis; viscous flow of incompressible fluids, boundary layer,
elementary turbulent flow, flow through pipes, head losses in pipes, bends and fittings.
Turbomachinery: Impulse and reaction principles, velocity diagrams, Pelton-wheel,
Francis and Kaplan turbines.

ANALYSIS OF GATE PAPERS


MECHANICAL CIVIL
Exam 1 Mark 2 Mark Exam 1 Mark 2 Mark
Year Ques. Ques. Total Year Ques. Ques. Total
2003 1 6 13 2003 3 8 19
2004 2 9 20 2004 6 10 26
2005 1 3 7 2005 3 7 17
2006 3 7 17 2006 3 7 17
2007 3 7 17 2007 2 7 16
2008 1 5 11 2008 1 7 15
2009 - 7 14 2009 1 3 7
2010 4 3 10 2010 3 2 7
2011 1 3 7 2011 3 2 7
2012 3 2 7 2012 3 3
2013 2 2 6 2013 3 2 7
2014 Set-1 1 5 11 2014 Set-1 2 7 16
2014 Set-2 2 3 8 2014 Set-2 2 4 10
2014 Set-3 3 3 9 2015 Set-1 3 4 11
2014 Set-4 2 2 6 2015 Set-2 3 3 9
2015 Set-1 1 4 9 2016 Set-1 1 3 7
2015 Set-2 2 2 6 2016 Set-2 1 3 7
2015 Set-3 2 2 6 2017 Set-1 1 2 5
2016 Set-1 3 3 9 2017 Set-2 1 2 5
2016 Set-2 2 3 8 2018 Set-1 4 3 10
2016 Set-3 3 3 9 2018 Set-2 1 3 7
2017 Set-1 4 3 10
2017 Set-2 4 3 10
2018 Set-1 3 4 11
2018 Set-2 2 1 4

© Copyright Reserved by Gateflix.in No part of this material should be copied or reproduced without permission
CONTENTS
Topics Page No
1. BASICS OF FLUID MECHANICS
1.1 Definition of Fluid 1
1.2 Basic Equations 1
1.3 System and Control Volume 1

2. PROPERTIES OF FLUIDS
2.1 Density 2
2.2 Specific Gravity/Relative Density 2
2.3 Viscosity 2
2.4 Surface Tension 5
2.5 Capillarity 6
2.6 Thermo Dynamic Properties 7
2.7 Compressibility and Bulk Modulus 7
2.8 Vapor Pressure 8
2.9 Cavitations 8
Gate Questions 9

3. PRESSURE & FLUID STATICS


3.1 Pressure 12
3.2 The Barometer and Atmospheric Pressure 13
3.3 Principles of Fluid Statics 14
3.4 Pressure Measurement 14
3.5 Hydrostatic Forces on Surfaces 21
3.6 Buoyancy & Floatation 33
Gate Questions 40

4. KINEMATICS
4.1 Introduction 48
4.2 Methods of Describing Fluid Motion 48
4.3 Types of Fluid Flow 48
4.4 Continuity Equation in Three-Dimensions 50
4.5 Continuity Equation in One Dimension 51
4.6 Motion of Fluid Element 51
4.7 Flow Patterns 52
4.8 Stream Function 53
4.9 Velocity Potential Function 53
4.10 Equipotential Line 54
Gate Questions 59

5. BERNOULLI’S EQUATION & ITS APPLICATIONS


5.1 Introduction 69

© Copyright Reserved by Gateflix.in No part of this material should be copied or reproduced without permission
5.2 Euler’s Equation 69
5.3 Bernoulli’s Equation 69
5.4 Application Of Bernoulli’s Equation 70
5.5 Bernoulli’s Equation For Real Fluids 72
5.6 Free Liquid Jets 72

6. DYNAMICS OF FLUID FLOW


6.1 Introduction 77
6.2 Impulse Momentum Principle 77
6.3 Force Exerted By a Fluid on a Pipe Bend 77
6.4 Force Exerted By a Fluid on Vertical Stationary Plate 77
6.5 Force Exerted By a Fluid on Moving Blade 78
Gate Questions 85

7. FLOW THROUGH CONDUITS/PIPES


7.1 Internal Flow 93
7.2 Laminar Flow/Viscous Flow 93
7.3 Turbulent Flow in Pipes 95
7.4 Loss of Energy in Fluid Flow 98
7.5 Flow through Pipes in Series or Flow through Compound Pipes 99
7.6 Flows through Nozzles 100
Gate Questions 108

8. EXTERNAL FLOW
8.1 Boundary Layer Formation 114
8.2 Regions of Boundary Layer 114
8.3 Boundary Layer Thickness 115
8.4 Drag Force on a Flat Plate Due To Boundary Layer 115
8.5 Boundary Condition for the Velocity Profile 116
8.6 Analysis of Turbulent Boundary Layer 117
8.7 Boundary Layer Separation 117
Gate Questions 122
9. HYDRAULLIC TURBINES
9.1 Introduction 136
9.2 Classification of Turbines 136
9.3 Turbine Efficiencies 136
9.4 Power Developed by Turbine (Euler’s Equation) 137
9.5 Pelton Turbine 137
9.6 Reaction Turbine 138
9.7 Francis Turbine 140
9.8 Kaplan Turbine 140
9.9 Specific Speed Significance 141
9.10 Model Testing (Dimensionless Turbine Parameters) 141
Gate Questions 149

10. ASSIGNMENT QUESTIONS 159

11. CIVIL GATE QUESTIONS 189

© Copyright Reserved by Gateflix.in No part of this material should be copied or reproduced without permission
1 BASICS OF FLUID MECHANICS

1.1 DEFINITION OF FLUID 1.3 SYSTEM AND CONTROL VOLUME

Fluid mechanics deals with the behaviour A system is defined as a fixed, identifiable
of fluids at rest and in motion. A fluid is a quantity of mass. The system boundaries
substance that deforms continuously under separate the system from the surroundings.
the application of a shear (tangential)
stress no matter how small the shear stress The boundaries of the system may be fixed
may be. or movable; however, there is no mass
Fluids comprise the liquid and gas (or transfer across the system boundaries. In
vapour) phases of the physical forms in the familiar piston–cylinder assembly the
which matter exists. The distinction gas in the cylinder is the system. Heat and
between a fluid and the solid state of work may cross the boundaries of the
matter is clear if you compare fluid and system, but the quantity of the matter
solid behaviour. A solid deforms when a within the system boundaries remains
shear stress is applied, but its deformation fixed. There is no mass transfer across the
does not continue to increase with time. system boundaries.
A control volume is an arbitrary volume in
1.2 BASIC EQUATIONS the space through which fluid flows. The
geometric boundary of the control volume
Analysis of any problem in fluid mechanics is called the control surface. The control
necessarily begins, either directly or surface may be real or imaginary; it may be
indirectly, with statements of the basic at rest or in motion.
laws governing the fluid motion. The basic
laws, which are applicable to any fluid, are:

1. The conservation of mass


2. Newton’s second law of motion
3. The principle of angular momentum
4. The first law of thermodynamics
5. The second law of thermodynamics

Clearly, not all basic laws always are


required to solve any one problem. On the
other hand, in many problems it is
necessary to bring into the analysis
additional relations, in the form of
equations of state or conservation
equations, that describe the behaviour of
physical properties of fluid under given
conditions.

© Copyright Reserved by Gateflix.in No part of this material should be copied or reproduced without permission
2 PROPERTIES OF FLUIDS

2.1 DENSITY solid or when two fluids move relative to


each other. A property that represents the
Density is defined as mass per unit volume internal resistance of a fluid to motion is
and denoted by ρ. SI unit of density is called viscosity. The force, a flowing fluid
Kg/m3 exerts on a body in the flow direction is
mass of fluid called the drag force, and the magnitude of
Density(ρ) =
volume of fluid this force depends, in part, on viscosity.
The reciprocal of density is the specific
volume ( ), which is defined as volume per 2.3.1 VISCOUS FORCE IN LIQUIDS AND
GASES
unit mass.
SI unit of density is m3/Kg 1) Molecular momentum transfer:
1 In the flow of liquids and gases,
v=
ρ molecules are free to move from one
The density of a substance, in general, layer to another. When the velocity in
depends on temperature and pressure. The the layers are different as in viscous
density of most gases is proportional to flow, the molecules moving from the
pressure and inversely proportional to layer at lower speed to the layer at
temperature. Liquids and solids, on the higher speed have to be accelerated.
other hand, are essentially incompressible Similarly, the molecules moving from
substances, and the variation of their the layer at higher velocity to a layer at
density with pressure is usually negligible. lower velocity, carry with them a higher
value of momentum and these are to be
2.2 SPECIFIC GRAVITY/RELATIVE DENSITY slowed down. Thus, the molecules
diffusing across layers transport a net
Specific gravity or relative density of a momentum, introducing a shear stress
substance is defined as the ratio of the between the layers. The force will be
density of a substance to the density of zero if both layers move at the same
standard substance at a specified speed or if the fluid is at rest.
temperature (usually water at 4°C, for
which water is 1000 kg/m3). 2) Cohesive force:
Specific Gravity =
Density of liquid When cohesive forces exist between
Density of water atoms or molecules these forces have to
be overcome, for relative motion
2.3 VISCOSITY between layers. A shear force is to be
exerted to cause fluids to flow.
When two solid bodies in contact move
relative to each other, a friction force 2.3.2 TYPES OF FLUID
develops at the contact surface in the
direction opposite to motion. Fluid is a Ideal fluid: Consider a fluid layer between
substance that deforms continuously under two very large parallel plates (or
the action of shear stress. The situation is equivalently, two parallel plates immersed
similar when a fluid moves relative to a in a large body of a fluid) separated by a

© Copyright Reserved by Gateflix.in No part of this material should be copied or reproduced without permission
small distance (dy). Now a constant Where,
tangential force F is applied to the upper μ is constant of proportionality and is
plate while the lower plate is held fixed. known as Dynamic viscosity. SI unit is
After some time upper plate moves kg/m-s, N. s/m2 or Pa / s
continuously under the influence of this
force at a constant velocity V. The fluid in Note:
contact with the upper plate sticks to the 1) Pa is the pressure unit Pascal.
plate surface and moves with same velocity 2) A common viscosity unit is poise
as that of the surface. This condition is 10 Poise = 1Ns / m 2
known as no slip condition. = 10
1centipoise = −2
Poise 10−3 Ns / m 2
3) The viscosity of water at 20°C is 1
centipoise, and thus the unit centipoise
serves as a useful reference.

2) Non Newtonian Fluids: Fluids in which


shear stress is not directly proportional
to deformation rate are known as non-
Newtonian fluids. Non Newtonian fluids
The shear stress τ yx acting on this fluid commonly are classified as having time
independent or time dependent
layer is behavior. Numerous equations have
τ yx = F / A been proposed to model the observed
Deformation or shear stain is denoted by dα relations between τ and du/dy for time-
Rate of shear stain or deformation is given  du 
n

dα independent fluids. τ =K  
by  dy 
dt
Where,
dl
tan(dα) = dα = n is called the flow behavior index
dy k the consistency index.
 dl   du 
n −1
du du
dα  dt  du τ =k   = η
= =  dy  dy dy
dt dy dy n −1
Where,  du 
The term η = k   is referred to as
u is the velocity is x direction.  dy 
du the apparent viscosity.
is known as velocity gradient. Fluids in which the apparent viscosity
dy
decreases with increasing deformation
1) Newtonian fluid: Fluids for which the rate (n<1) are called pseudo plastic (or
shear thinning) fluids. Most non-
rate of deformation is proportional to
the shear stress are called Newtonian Newtonian fluids fall into this group;
fluids. Examples include polymer solutions,
colloidal suspensions, and paper pulp in
τ yx ∝ rate of deformation or rate of
water. If the apparent viscosity
shear strain. increases with increasing deformation
d rate (n>1) the fluid is termed dilatants
τ =μ
dt (or shear thickening).Suspension of
du starch and of sand are examples of
τ =μ
dy dilatants fluids.

© Copyright Reserved by Gateflix.in No part of this material should be copied or reproduced without permission
A “fluid” that behaves as a solid until a 1) Liquids: In case of liquids the viscosity
minimum yield stress,τ, is exceeded and force is mainly due to cohesive force.
subsequently exhibits a linear relation The cohesive force decreases. So,
between stress and rte of deformation viscosity of liquids decreases when
is referred to as an ideal or Bingham temperature increases. The relation of
plastic. viscosity with temperature is given by
Thixotropic fluids show a decrease in η  1 
with time under a constant applied µ = µ0  
 1 + αT + 7βT 
shear stress. Rheopectic fluids show an
2) Gases: In the case of gases, the
increase in η with time. After
contribution to viscosity is more due to
deformation some fluids partially
momentum transfer. As temperature
return to their original shape when the
increases, more molecules cross over
applied stress is released; such fluids
with higher momentum differences.
are called viscoelastic.
Hence, in the case of gases, viscosity
increases with temperature.
µ = µ 0 + αT + β T 2
where,
μ = Viscosity atT 0in poise
μ 0 = Viscosity at00 in poise
α, β = are constants for liquid and gas

SOLVED EXAMPLES

Example:
‘An infinite plate is moved with a velocity of
0.3m/s over a second plate on a layer of
liquid for small gap width d=0.3mm,
2.3.3 KINEMATIC VISCOSITY
assume a linear velocity distribution the
Kinematic viscosity is defined as the ratio liquid viscosity is 0.65 ×10−3 kg / ms
between dynamic viscosity and density and S.G is 0.88.
denoted by ‘ν’ a) Calculate kinematic viscosity
ν = μ/ρ b) The shear stress on the lower plate
The unit in SI system is m2/s. Stoke is CGS
unit of kinematic viscosity given by Solution:
1 (cm2/s) = 10–4 m2/s. a)
1 centistoke = 10–6 m2/s. µ
V=
ρ
Kinematic viscosity gives the rate of
−3 kg m3
momentum flux or momentum diffusivity. = 0.65 ×10 ×
For liquids and gases absolute (dynamic) m.s ( 0.88 ×1000 ) kg
viscosity is not influenced significantly by = 7.39 ×10−7 m 2 / s
pressure. But kinematic viscosity of gases is b)
influenced by pressure due to change in u
density. τlower =µ 
d
2.3.4 EFFECT OF TEMPERATURE ON
VISCOSITY

© Copyright Reserved by Gateflix.in No part of this material should be copied or reproduced without permission
kg m 1 resulting compression effect causes the
=0.65 ×10−3 × 0.3 ×
m.s s 0.3 ×10−3 m liquid to minimize its surface area. This is
=0.65Pa the reason for the tendency of the liquid
Direction of shear stress on lower plate droplets to attain a spherical shape, which
 has the minimum surface area for a given
is τlower
volume.
The surface of the liquid acts like a
Example:
stretched elastic membrane under tension.
Calculate the dynamic viscosity of oil,
The pulling force that causes this tension
which is used for lubrication b/w a square
acts parallel to the surface and is due to the
of size 0.8 × 0.8 m2 and an inclined plane
attractive forces between the molecules of
with angle of inclination30° , wt = 300 N, the liquid. The magnitude of this force per
slides down the inclined plane with a unit length is called surface tension and is
uniform velocity of 0.3 m/s. the thickness usually expressed in the unit N/m.
of oil film is 1.5 mm. Surface tension is also defined as the
Solution: surface energy per unit surface area or
W sin θ =Foil work that needs to be done to increase the
Foil ∆u surface area of the liquid by a unit amount.
= µ
Fcontact ∆y Surface tension is a binary property of the
Foil u liquid & gas or two liquids. The surface
=µ tension of air and water at 20° c is about
A y
0.73 N/m.
F u
µ = oil .
A y
W sin θ.y
µ=
A.u
300.sin 30 ×1.5 ×10−3
=µ = 0.117 ×10Poise
0.64 × 0.3
=1.17Poise

2.4 SURFACE TENSION If F is the tensile force on the surface, L is


the length of the surface. Surface tension is
Consider two liquid molecules, one at the given by
surface and one deep within the liquid σ=
F
or
body. The attractive forces applied on the L
interior molecule by the surrounding E surface
molecules balance each other because of σ=
surface Area
symmetry. But the attractive forces acting
on the surface molecule are not symmetric,
2.4.1 SURFACE TENSION ON LIQUID
and the attractive forces applied by the gas
DROPLET
molecules above are usually very small.
Therefore, there is net attractive force
acting on the molecule at the surface of the
liquid, which tends to pull the molecules on
the surface toward the interior of the
liquid. This force is balanced by the
repulsive forces from the molecules below
the surface that are being compressed. The Let σ is surface tension
R is radius of droplet

© Copyright Reserved by Gateflix.in No part of this material should be copied or reproduced without permission
Ap is area of projection

1) Force due to difference in pressure


inside & outside the liquid drop
= ∆PA p
=∆PπR 2 … (i)
2) Tensile force due to surface tension
= σ × circumference
= σ × 2πR … (ii)
Under equilibrium condition, these 2
forces will be equal
1) Force due to pressure inside & outside
2σ the liquid jet
∴ ∆P =
R = ∆p.L 2R ( ) … (i)
2.4.2 SURFACE TENSION ON A SOAP 2) Tensile force due to surface tension
BUBBLE = 2L.σ … (ii)
Equating the forces, we get
σ
∆P =
R

2.5 CAPILLARITY

Another consequence of surface tension is


the capillary effect, which is the rise or fall
1) Force due to pressure inside the liquid of a liquid in a small-diameter tube inserted
drop into the liquid. Such narrow tubes or
= ∆PA p confined flow channels are called
=∆PπR 2 … (i) capillaries. The curved free surface of a
2) Tensile force due to surface tension liquid in a capillary tube is called the
meniscus. It is commonly observed that
Fsurface = 2πR.σ
water in a glass container curves up slightly
Fsurface2 =2π ( R + t ) σ … (ii) at the edges where it touches the glass
Equating the forces, we have surface; but the opposite occurs for
(
∆Pπr2 =σ2π R + t + σ2πR ) mercury: it curves down at the edges.. The
strength of the capillary effect is quantified
Assuming thickness is very small by the contact (or wetting) angleβ, defined
T << R as the angle that the tangent to the liquid
4σ surface makes with the solid surface at the
∆P =
R point of contact. The surface tension force
acts along this tangent line toward the solid
2.4.3SURFACE TENSION ON A LIGHT JET surface.

© Copyright Reserved by Gateflix.in No part of this material should be copied or reproduced without permission
Where,
R' is characteristic gas constant & for air
J
R ' = 287
kg − K
Mw is molecular wt of gas
ρ is density of gas

2.6.2 THERMODYNAMIC PROCESS


1) Force due to surface tension
=σ.2πr
a) Isothermal: Constant Temperature
Fy = σπr.cos β
PV=mRT
Fx = 0 T = const
2) Vertical force is responsible for lifting PV = const
the liquid in capillary b) Adiabatic Process: No heat transfer
takes place
Fy = mg = ρπr2h.g
P
Equating the vertical forces, we get = const
ργ
2πrσ cos β = ρπr2h.g γ is ratio of specific heat
2σ cos β Cp
⇒h= γ=
ρgr Cv
Note: γ = 1.4 for air
Same expression is used for capillary fall. c) Isobaric process: Constant pressure
Angle, β = 0 for glass tube & water. process
β = 128o for glass tube & mercury. P = const
V
2.6 THERMO DYNAMIC PROPERTIES = const
T
d) Isochoric process: Constant volume
2.6.1 IDEAL GAS EQUATION process
V = const
PV = nRT P
Where, = const
T
P is pressure in Pa
V is volume in m3
2.7 COMPRESSIBILITY AND BULK
n is moles of gas
MODULUS
R is universal gas constant (8.314 KJ/mole
K)
It is the measure of volume change under
 M  the action of external force.
PV =   RT
 Mw 
 R 
⇒ PV = m   T
 Mw 
⇒ PV =
m R' T ( )
m
⇒P=  R''T
V
⇒P=ρR''T

© Copyright Reserved by Gateflix.in No part of this material should be copied or reproduced without permission
Let the volume of gas decrease from V to Likewise, at a given temperature, the
(v-dv), the pressure is increased from P to pressure at which a pure substance
P+dP changes phase is called the saturation
Increase in pressure pressure Psat . The vapour pressure Pv of a
= dp kgf / m2 pure substance is defined as the pressure
Decrease in volume = dv exerted by its vapour in phase equilibrium
−dv with its liquid at a given temperature.
Volumetric strain =
v Pv is a property of the pure substance, and
dp turns out to be identical to the saturation
Bulk Modulus (k) =
−dv / v pressure Psat of the liquid ( Pv ” Psat ).
Compressibility = 1/k
2.9 CAVITATIONS
Relationship b/w bulk modulus and
pressure (p) for a gas undergoing
The liquid pressure in liquid-flow systems
compression process
drops below the vapour pressure at some
locations, results in vaporization of liquid.
a) For Isothermal process:
For example, water at10°C will convert into
P vapour and form bubbles at locations (such
= const
ρ as the tip regions of impellers or suction
for closed system. sides of pumps) where the pressure drops
Pv = const below 1.23 kPa. The vapour bubbles (called
by taking log cavitation bubbles since they form
ln P+ ln V = ln(const) “cavities” in the liquid) collapse as they are
By differentiating swept away from the low pressure regions,
dp dV generating highly destructive, extremely
+ =0
p V high-pressure waves. This phenomenon,
dp which is a common cause for drop in
=P performance and even the erosion of
−dv / v
impeller blades, is called cavitation, and it
k=p
is an important consideration in the design
of hydraulic turbines and pumps.
b) Adiabatic Process:
PV y = const
by taking log both side
In P + γ ln v = ln const
by differentiating
dp ydV
+ = 0
p V
dp
= γP
−dv / v
k = γP

2.8 VAPOR PRESSURE

At a given pressure, the temperature at


which a pure substance changes phase is
called the saturation temperature Tsat .

© Copyright Reserved by Gateflix.in No part of this material should be copied or reproduced without permission
GATE QUESTIONS

0.04 m2 . The velocity distribution


Q.1 The SI unit of kinematic viscosity within the oil film gap is linear as
(v) is shown in the figure.
a) m2/s b) kg/m-s Take dynamic viscosity of oil as
c) m/s 2 d) m3/ s2 7 × 10 −3 P-s and acceleration due to
[GATE–2001] gravity as 10 m / s2 .
Neglect weight of the oil. The
Q.2 For a Newtonian fluid terminal velocity V (in m/s) of the
a) Shear stress is proportional to block is _________ (correct to one
shear strain decimal place).
b) Rate of shear stress is
proportional to shear strain
c) Shear stress is proportional to
rate of shear strain
d) Rate of shear stress is
proportional to rate of shear
strain
[GATE–2006]

Q.3 Assuming constant temperature


condition and air to be an ideal gas,
the variation in atmospheric
pressure with height calculated
from fluid statics is
a) linear b)exponential [GATE–2018(2)]
b) quadratic d) cubic
[GATE–2016(2)]

Q.4 A solid block of 2.0 kg mass slides


steadily at a velocity V along a
vertical wall as shown in the figure
below. A thin oil film of thickness h
= 0.15 mm provides lubrication
between the
block and the wall. The surface area
of the face of the block in contact with
the oil film is
ANSWER KEY:

1 2 3 4
(a) (c) (b) 10.71
EXPLANATIONS

Q.1 (a) Q.4 (10.71)


Q.2 (c)

From the Newton’s law of Viscosity, the shear


stress (τ) is directly
proportional to the rate of shear strain
(du/dy).
Terminal velocity is reached when weight
du du force of the block is balanced by the
So, τ ∝ or τ =μ viscous drag forces due to shear stress
dy dy
generated in the fluid-solid interface.
Where, µ = constant of proportionality and it ∴τ A = W
is known as coefficient of viscosity
Using Newton's law of viscosity
du V
=τ μ= μ (Q linear profile)
dy h
V Wh mgh
Q.3 (b) ⇒ μ A= W⇒ V= =
h μA μA
2 10  0.1510−3
dP = ρ gdh = = 10.714 m/s
7 10−3  0.04
P
⇒ dP = ⋅ g ⋅ dh
RT
on Integnating,
P h
dP g
∫ P
=
RT ∫0
dh
Patm

 P  gh
ln  = +
 Patm  RT
 gh 
+ 
⇒ P = Patm e  RT 

Hence, exponential

© Copyright Reserved by Gateflix.in No part of this material should be copied or reproduced without permission
3 PRESSURE & FLUID STATICS

3.1 PRESSURE
Pressure is defined as the normal force
exerted by fluid per unit area. We speak of
pressure only when we deal with a gas or a
liquid. The counterpart of pressure in
solids is normal stress.
F
P=
A
Since pressure is defined as force per unit
area, it has the unit of Newton’s per square
meter (N/m2), which is called Pascal (Pa)
1bar = 105 Pa = 0.1 MPa = 100 kPa 3.1.2 PRESSURE AT A POINT
3.1.1 ABSOLUTE, GAUGE, ATMOSPHERIC & Pressure is the compressive force per unit
VACUUM PRESSURE area, and it gives the impression of being a
vector. However, in fluids under static
1) The pressure values must be stated conditions, pressure is found to be
with respect to a reference level. If the
independent of the orientation of the area.
reference level is vacuum (i.e. absolute This concept is explained by Pascal’s law
zero pressure), pressures are termed
which states that the pressure at a point
absolute pressure.
in a fluid at rest is equal in magnitude in
2) Most pressure gauges indicate a all directions. Pressure has magnitude but
pressure difference—the difference
not a specific direction, and thus it is a
between the measured pressure and the
scalar quantity.
ambient level (usually atmospheric
P= P= Pz
pressure). Pressure levels measured x y

with respect to atmospheric pressure 3.1.3 PRESSURE VARIATION IN A STATIC


are termed gauge pressures
FLUID (HYDROSTATIC LAW):
3) Pressures below atmospheric pressure
are called vacuum pressures. For fluids at rest or moving on a straight
Absolute, gauge, and vacuum pressures path at constant velocity, all components of
are all positive quantities and are acceleration are zero. In fluids at rest, the
related to each other by pressure remains constant in any
=Pabsolute Patmospheric + Pgauge horizontal direction (P is independent of x
=
Pvacuum Patmospheric + Pabsolute and y) and varies only in the vertical
direction.
As a result of gravity, these relations are
applicable for both compressible and
incompressible fluids.
dp dp dp
= −ρ(g), = 0, = 0
dz dx dy

© Copyright Reserved by Gateflix.in No part of this material should be copied or reproduced without permission
The negative sign is taken because dz is Case 2: Acceleration in negative x
taken positive in upward direction and direction
pressure decrease in upward direction. dp dp dp
= −ρ ( g ) , = −ρ ( −a x ) , = 0
For incompressible fluid ρ is constant. dz dx dy
p z

∫ dp = −ρg ∫ dz
po zo
3.2 THE BAROMETER & ATMOSPHERIC

P − Po = −ρg(z − z o ) PRESSURE
P − Po = ρgh Atmospheric pressure is measured by a
For compressible fluid, ρ varies with device called barometer; thus, the
pressure, i.e. ρ =f (P) atmospheric pressure is often referred to
For gases, variation of density with as the barometric pressure. The pressure at
pressure can be expressed by ideal gas point B is equal to the atmospheric
equation pressure, and the pressure at C can be
P taken to be zero since there is only mercury
ρ= vapour above point C and the pressure is
RT very low relative to Patm and can be
Where ,
P is pressure neglected for an excellent approximation.
T is temperature Writing a force balance in the vertical
direction gives
3.1.4 FLUIDS IN RIGID-BODY MOTION Patm = ρgh
When fluid is in stationary container, the Where,
pressure remains constant along horizontal ρ is the density of mercury,
direction. The pressure varies only along g is the local gravitational acceleration,
vertical direction. When a fluid is placed in h is the height of the mercury column
an accelerated container, initially fluid above the free surface.
splashes and there is a relative motion Note that the length and the cross-sectional
between fluid & container boundary. After area of the tube have no effect on the height
some time, the liquid comes to rest and of the fluid column of a barometer. A
attains fixed shape relative to container. frequently used pressure unit is the
The pressure varies in the direction of standard atmosphere, which is defined as
acceleration. the pressure produced by a column of
1) When container accelerates in mercury 760 mm in height at 0°C
vertical direction ( ρHg is13,595 kg / m3 ) . If water instead of
Case1: Downward acceleration of az mercury were used to measure the
dp dp dp
= −ρ ( g − a z ) , = 0, = 0 standard atmospheric pressure, a water
dz dx dy column of about 10.3m would be needed.
Case 2: upward acceleration of az
dp dp dp
= −ρ ( g + a z ) , = 0, = 0
dz dx dy
2) When container accelerates in
horizontal direction
Case1: Acceleration in positive x
direction
dp dp dp
= -ρ ( g ) , = -ρa x , =0
dz dx dy

© Copyright Reserved by Gateflix.in No part of this material should be copied or reproduced without permission
3.3 PRINCIPLES OF FLUID STATICS alcohol, or oil. The pressure of a fluid is
measured by the following.
1) When fluid is at rest, in a continuous 1) Manometers
fluid, fluid at the same elevation has the a. Simple
same pressure. • Piezometer
2) The pressure at the bottom of a column • U-tube manometer
of fluid is equal to the pressure at the • Single column manometer
top, plus density multiplied by gravity b. Differential
multiplied by the height of the column of • U-tube differential manometer
fluid. • Inverted U-tube manometer
A consequence of the second principle 2) Mechanical gauge
is that when different columns of fluid
stack on top of one another, the 3.4.1 SIMPLE MANOMETER
pressures due to each column simply
add up. 1) Piezometer: It is the simplest kind of
3.3.1 HYDRAULIC LIFT manometer. It does not have any high
density liquid. The tube is connected to
A consequence of the pressure in a fluid
the point where pressure is to be
remaining constant in the horizontal
measured. The liquid rises in tube to
direction is that the pressure applied to a
balance the pressure at ‘A’.
confined fluid increases the pressure
The gauge pressure PA is given by
throughout by the same amount. This is
PA = ρgh
called Pascal’s law. The application of
Pascal’s law in hydraulic lift is shown in fig Where
P1= P2 ρ is the density of liquid inside
F1 / A1 = F2 / A 2 vessel/pipe

Piezometer is used to measure low


pressures. The height of column
increases if pressure is high. E.g. height
of water column is 10.3 m if gauge
Fig.: Schematic of a hydraulic lift pressure at ‘A’ is 1atm (105N/m2).
3.4 PRESSURE MEASUREMENT 2) U-tube manometer: It has a glass U-
tube with liquid having density higher
Hydrostatic law indicates the pressure than the density of fluid in the
difference b/w two points in a static fluid. A container.
device based on this principle is called
manometer, and it is commonly used to
measure small and moderate pressure
differences. A manometer mainly consists
of a glass or plastic U tube containing one
or more fluids such as mercury, water,

© Copyright Reserved by Gateflix.in No part of this material should be copied or reproduced without permission
The atmospheric pressure exists and ii) large cross-sectional area (100
should be taken into account for times)
evaluation of absolute pressure, but iii) due to large area of cross section
while evaluating gauge pressure it is and small change of pressure, the
not accounted in equations. change in level of reservoir will be
The gauge pressure at ‘A’ is given by very small and can be neglected.
P=
1 PA + ρc gh1
P2 = ρm gh 2 4) Inclined Single Column Manometer:
This manometer is more sensitive than
Where,
straight column. The liquid rises more
ρC is the density of fluid in container, it in the column due to inclination.
can be water or oil
ρm is the density of manometric fluid.
Usually mercury is chosen as
manometric fluid
h2 is difference in mercury level
PA is pressure in vessel/pipe in gauge
From principle of fluid statics, when
fluid is at rest, fluid at the same =
h 2 L sin θ
elevation has the same pressure. ah 2
=
PA . ( ρ2 g − ρ1g ) + ( ρ2 gh 2 − ρ1gh1 )
PA = ρm gh 2 − ρw gh1 A
PA = ρ2 g 2 h 2 − ρ1g1h1
3) Single Column Manometers: In this PA = ρ2 g 2 L sin θ − ρ1gh1
manometer a large cross-sectional area
reservoir is placed in one of the limbs. 3.4.2 DIFFERENTIAL MANOMETERS
When pressure is applied, the fluid
lowers slightly in the reservoir as Differential Manometers are devices used
compared to the fluid rise in the other for measuring the difference of pressure
limb.
between two points in a pipe or two
different pipes. It contains of a U-tube with
manometric liquid. The manometric liquid
can be of higher density or lower density
than pipe liquid.

1) U-tube differential manometer:

Gauge Pressure at point A is given as


ah 2
=PA . ( ρm g − ρc g ) + ( ρm gh 2 − ρc gh1 )
A
Since A>>α
PA = ρm gh 2 − ρc gh1

Salient features of Single column


manometer:
i) modified form of U-tube manometer Pressure above a-a

© Copyright Reserved by Gateflix.in No part of this material should be copied or reproduced without permission
• ρ1g ( h + x ) + PA
In the left limb = respectively. If the depth of water in the
tank is 1m and tank is open at the top then,
• In the right limb= PB + ρ2 gy + ρHg gh
Calculate:
By equating I. angle of the water surface with the
PA + ρ1g(h + x)= PB + ρ2 gy + ρHg g.H horizontal
PA − PB = ρ2 gy + ρHg .g.H − ρ1g ( h + x ) II. the maximum and minimum
pressure intensities at the bottom
A & B are at same level :

ρ2 gx + ρHg gh − ρ1g ( h + x )
PA − PB = Solution:
If liquid is same, then ρ1gx =
ρ2 gx Given:
Constant acceleration a = 2.4m/ s2
∴ PA − P= gh ( ρHg − ρ1 )
B
Length = 6m; Width = 2.5m and depth =
2m,
2) Inverted U-tube differential Depth of water in tank, h = 1m
Manometer:
i) The angle of the water surface to the
horizontal
Let θ = the angle of water surface to the
horizontal
Using equation, we get
a 2.4
tan θ = − = − = −0.2446
g 9.81
(the –ve sign shows that the free
surface of water is sloping downward
as shown in Fig)
PA − ρ1gh=
1 PB − ρ2 gh 2 − ρs gh ∴ tan θ = −0.2446 (slope downward)
−1
=
∴ θ tan
= 0.2446 13.7446o or 13o 44.6 '
PA − PB = ρ1gh1 − ρ2 gh 2 − ρs gh
It is used for measuring difference of ii) The maximum and minimum
low pressures pressure intensities at the bottom of
the tank
SOLVED EXAMPLES From the figure, depth of water at the
front end,
Example: = h1 1 - 3tanθ = 1 - 3 × 0.2446 = 0.2662m
A rectangular tank is moving horizontally Depth of water at the rear end :
in the direction of its length with a constant
= h2 1 + 3tanθ = 1 + 3 × 0.2446 = 1.7338m
acceleration of 2.4m/ s2 . The length, width
and depth of the tank are 6m, 2.5m and 2m

© Copyright Reserved by Gateflix.in No part of this material should be copied or reproduced without permission
The pressure intensity will be
maximum at the bottom, where depth
of water is maximum.
Now, the maximum pressure intensity
at the bottom will be at point A and it is
given by,
Solution:
Pmax = ρ × g× h2
Given:
=1000×9.81×1.7338N/ m2 =17008.5N/ m2 Dia. of ram D=30cm = 0.3m
The minimum pressure intensity at the Dia. Of plunger, d = 4.5cm = 0.045m
bottom will be at point B and it is given Force on plunger, F =500N
by Let the weight lifted = W
pmin = ρ × g× h1 π π
( 0.3) = 0.07068m2
2
2
Area of ram, A = D2 =
=1000×9.81×0.2662= 2611.4N/ m 4 4
Area of plunger,
Example: π π
( )
2

A U-tube as shown in figure, filled with


a = d2 = 0.045 = .00159m2
4 4
water to mid level is used to measure the Pressure intensity due to plunger
acceleration when fixed on moving Force on plunger F 500
equipment. Determine the acceleration ax = = = N / m2
Area of plunger a .00159
as a function of the angle θ and the
distance A between legs.
Due to Pascal’s law, the intensity of
pressure will be equally transmitted in all
directions. Hence the pressure intensity at
the ram
500
= =314465.4N/ m2
.00159
But pressure intensity at ram
Solution: Weight W W
This is similar to the formation of free = = = N/ m2
Area of ram A .07068
surface with angle θ
W
=
tan θ –a x / g + a y( ) .07068
=314465.4
As= =
a y 0, tan θ –a x / g ∴ Weigth
The acute angle θ will be given by, = 314465.4×0.7068 = 22222N = 22.222kN
θ =tan–1 ( a x / g )
Example:
ax =g × tan θ
The diameters of a small piston and a large
As tan θ =2h / A piston of hydraulic jack are 3 cm and 10cm
h = A a x / 2g respectively. A force of 80 N is applied on
the small piston. Find the load lifted by the
Example: large piston when:
A hydraulic press has a ram of 30cm a) The pistons are at the same level.
diameter and a plunger of 4.5cm diameter. b) Small piston is 40cm above the large
Find the weight lifted by the hydraulic piston.
press when the force applied at the plunger The density of the liquid in the jack is given
is 500N. as 1000kg/ m3

© Copyright Reserved by Gateflix.in No part of this material should be copied or reproduced without permission
Solution:
Given:
Dia. of small piston, d =3cm
∴ Area of small piston,
π π
A= × d2 = × 32 = 7.068cm2
4 4
Dia. of large piston, D=10cm
∴ Area of larger piston,
π ∴ Pressure intensity at section A-A
A= × 102 = 78.54cm2
4 F
= + Pressure intensity due to height
Force on small piston, F = 80N a
Let the load lifted = W of 40cm of liquid.
Pressure intensity due to 40cm of liquid
a) When the pistons are at the same = ρ × g× h = 1000×9.81×0.4N/ m2
level 1000×9.81×.40
Pressure intensity on small piston = N/ cm2 = 0.3924N/ cm2
104
F 80 ∴ Pressure intensity at section A-A
= N/ m2
a 7.068 80
= +0.3924
7.068
11.32 + 0.3924 =
11.71 N/cm2
∴ Pressure intensity transmitted to the
large piston = 11.71N/ cm2
∴ Force on the large piston = Pressure ×
Area of the large piston
This is transmitted equally to the large = 11.71× A = 11.71×78.54 = 919.7N
piston.
∴ Pressure intensity on the large piston Example:
80 A U-tube manometer is used to measure
=
7.068 the pressure of water in a pipe line, which
∴ Force on the large piston is in excess of atmospheric pressure. The
= pressure x area right limb of the manometer contains
80 mercury and is open to water in the main
= × 78.54N = 888.96N line, if the difference in level of mercury is
7.068 in the left limb. Determine the pressure of
water in the main line, if the difference in
b) When the small piston is 40cm above level of mercury in the limbs of U-tube is 10
the large piston cm and the free surface of mercury is in
Pressure intensity on the small piston level with the centre of the pipe. If the
F 80 pressure of water in pipe line is reduced to
= N/ m2
a 7.068 9810N/m2, calculate the new difference in
the level of mercury. Sketch the
arrangements in both cases.
Solution:
Given:
Difference in mercury level =10 cm =0.1m
The arrangement is shown in fig (a)

© Copyright Reserved by Gateflix.in No part of this material should be copied or reproduced without permission
mercury in left limb will rise. The rise of
mercury in left limb will be equal to the fall
of mercury in right limb as the total volume
of mercury remains same.
Let x =Rise of mercury in left limb in cm.
Then fall of mercury in right limb = x cm
The points B, C and D shows the initial
conditions whereas points B*, C* and D*
show the final conditions.
pressure at B* = Pressure at C*
Or
1st Part Pressure at A + Pressure due to (10-x)cm
Let PA = (pressure of water in pipe line (i.e., of water
at point A) = Pressure at D* + Pressure due to (10-2x)
The points B and C lie on the same cm of mercury
horizontal line. Hence pressure at B should Or pA + ρ1 × g× h1 = pD *+ρ2 × g× h2
be equal to pressure at C. But pressure at B
= Pressure at A + Pressure due to 10cm (or  10 - x 
Or 1910+1000×9.81×  
0.1 m) of water  100 
= pA + ρ × g× h
 10 - 2x 
Where ρ = 1000kg/ m3 and h = 0.1m = 0+ (13.6×1000) ×9.81×  
= pA +1000×9.81×0.1  100 
Dividing by 9.81, we get
= pA + 981N/ m2 …………(i) 1000 + 100 - 10x = 1360 - 272x
Pressure at C = pressure at D+ Pressure Or
due to 10 cm of mercury 272x - 10x = 1360 - 1100
= 0+ ρ0 × g× h0 Or
Where 𝜌𝜌0 for mercury = 13.6×1000kg/ m3 262x = 260
260
And h0 =10cm = 0.1m ∴ x= = 0.992cm
262
∴ Pressure at C
= 0 + (13.6 × 1000) × 9.81 × .01
∴ New difference of mercury
=10 − 2xcm =10 − 2×0.992
= 13341.6N ………(ii)
= 8.016cm
But pressure at B is equal to pressure at C.
Hence equating the equations (i) and (ii),
Example:
we get,
Fig. shows a conical vessel having its outlet
pA + 981 = 13341.6
at A to which a U-tube monometer is
∴ pA = 13341.6 - 981 connected. The reading of the manometer
N given in the figure shows when the vessel is
= 12360.6 2
empty. Find the reading of the manometer
m when the vessel is completely filled with
water.
2nd Part
Given, pA = 9810N/ m2
Find new difference of mercury level. The
arrangement is shown in following figure.
In this case, pressure at A is 9810𝑁𝑁/𝑚𝑚2
which is less than the 12360.6𝑁𝑁/𝑚𝑚2 . Hence

© Copyright Reserved by Gateflix.in No part of this material should be copied or reproduced without permission
= 1000×9.81× (3+ h1 + y/ 100 )
13.6 × ( 0.2 + 2y / 100 ) =( 3 + 2.72 + y / 100 )

Or
2.72+27.2y/ 100 = 3+2.72+ y/ 100
Or
( )
27.2y- y / 100 = 3.0
Or
26.2y =3×100 =300
300
∴ y= =11.45cm
26.2
Solution: The difference of mercury level in two
Vessel is empty: limbs
Given: = ( 20+2y ) cm of mercury
Difference of mercury level
= 20+2×11.45= 20+22.90
h2 = 20cm
= 42.90cm of mecury
Let h1 = Height of water above X-X ∴ Reading of monometer = 42.90 cm
S.G. of mercury, S2 =13.6
S.G. of water, S1 = 1.0
Density of mercury,
ρ2 = 13.6×1000
Density of water, Example:
A single column manometer is connected to
ρ1 = 1000
a pipe containing a liquid of S.G. 0.9 as
Equating the pressure above datum line X- shown in Fig. Find the pressure in the
X, we have, pipe if the area of the reservoir is 100
ρ2 × g× h2 = ρ1 × g× h1 times the area of the tube for the
Or 13.6×1000×9.81×0.2=1000×9.81×h1 manometer reading shown in Fig. The
h1 = 2.72m of water. specific gravity of mercury is 13.6

Vessel is full of water:


When vessel is full of water, the pressure in
the right limb will increase and mercury
level in the right limb will go down. Let the
distance through which mercury goes
down in the right limb be, y cm as shown in
following figure. The mercury will rise in
the left by a distance of y cm. Now the Solution:
datum line is Z-Z. Equating the pressure Given:
above the datum line Z-Z, Sp. gr. of liquid in pipe, S1 = 0.9
Pressure in left limb =Pressure in right
∴ Density ρ1 = 900kg/ m3
limb
 Sp. gr. of heavy liquid, S2 =13.6
2y 
13.6×1000×9.81×  0.2+  ∴ Density, ρ2 = 13.6×1000
 100 

© Copyright Reserved by Gateflix.in No part of this material should be copied or reproduced without permission
Area of reservoir A PA =1kgf/ cm2 =1×104 kgf/ m2
= = 100
Area of right limb a = 104 × 9.81N / m2 (Q 1kgf =
9.81N)
Height of liquid, h1 = 20cm = 0.2m Pressure at B,
Rise of mercury in right limb, PB =1.8kgf/ cm2
h2 = 40cm = 0.4m
= 1.8×104 kgf/ m2
Let pA = Pressure in pipe
Using equation, =1.8 × 104 × 9.81N / m2(Q 1kgf =9.81N)
a Density of mercury = 13.6×1000kg/ m3
pA = h2 ρ2 g − ρ1 g  + h2 ρ2 g − h1 ρ1 g,
A Taking X-X as datum line
we get, Pressure above X-X in the left limb
1 =13.6×1000×9.81×h+1500×9.81× ( 2+3) + pA
= ×0.4 13.6×1000×9.81- 900×9.81
100 =13.6×1000×9.81× h+7500×9.81+9.81×104
+0.4×13.6×1000×9.81 − 0.2×900×9.81
Pressure above X-X in the right limb
0.4
= 133416 - 8829 +53366.4 -1765.8 = 900×9.81× ( h+2) + pB
100 
= 533.664 + 53366.4 − 1765.8N / m 2 = 900×9.81× ( h+2) +1.8×104 ×9.81
= 52134N / m 2 Equating the two pressures, we get
= 5.21N / cm 2 13.6 × 1000 × 9.81h + 7500 × 9.81 + 9.81 × 104
= 900×9.81× ( h+2) +1.8×104 ×9.81
Example:
Dividing by 1000×9.81 , we get
A differential manometer is connected at
the two points A and B of two pipes as 13.6h+7.5+10 = ( h+2.0 ) ×.9+18
shown in fig. The pipe A contains a liquid of Or
S.G. = 1.5 while pipe B contains a liquid of 13.6h+17.5 = 0.9h+1.8 +18 = 0.9h+19.8
S.G. = 0.9. The pressures at A and B are Or
1kgf/cm2and 1.80kgf/cm2 respectively.
Find the difference in mercury level in the
( )
13.6 - 0.9 h =19.8 -17.5or12.5h = 2.3
differential manometer. 2.3
∴h= = 0.181m =18.1cm
12.7

3.5 HYDROSTATIC FORCES ON SURFACES


In fluid statics, there is no relative motion
between adjacent fluid layers, and thus
there are no shear (tangential) stresses in
the fluid trying to deform it. The only stress
we deal with in fluid statics is the normal
stress, which is the pressure, and the
variation of pressure is only due to the
weight of the fluid. The force exerted on a
surface by a fluid at rest is normal to the
Solutions: surface at the point of contact since there is
Given: no relative motion between the fluid and
S.G. of liquid at A, = the solid surface, and thus no shear forces
S1 1.5 ∴ =
ρ1 1500
can act parallel to the surface.
S.G. of liquid at B, =
S2 1.5 ∴ =
ρ2 1500 Fluid statics is used to determine the forces
Pressure at A, acting on floating or submerged bodies and

© Copyright Reserved by Gateflix.in No part of this material should be copied or reproduced without permission
the forces developed by devices like Force on strip dF = ρ. g. h. b. dh
hydraulic presses and car jacks. The design Total pressure force on the whole
of many engineering systems such as water surface is
dams and liquid storage tanks requires the
determination of the forces acting on the
∫ dF = ∫ρghbdh
s

surfaces using fluid statics. ∫ dF = ρg ∫ h.dA


s

3.5.1 TOTAL PRESSURE F = ρ.g.h.A

Force is exerted by a static fluid on a


∫h.dA is moment of surface area about
surface, either plane or curved when fluid free surface of liquid is equal moment of
comes in contact with the surfaces. This C.G. about free surface.
force always acts normal to the surface. ∫h.dA = A.h
3.5.2 CENTRE OF PRESSURE
b) Centre of Pressure: (𝐡𝐡∗ )
Principle of Moments: Moment of the
It is defined as the point of application of
resultant force about an axis is equal to
the total pressure on the surface. The
the sum of moments of the components
submerged surfaces may be
about the same axis.
1) Vertical plane submerged
Ft . h∗ = ∑moments about free surface of
2) Horizontal plane surface
liquid. …(1)
3) Inclined plane
4) Curved surface ∑ moments
= ∫ dA.h.ρgh
= ρg ∫ bh2dh
3.5.3 VERTICAL PLANE SURFACE
SUBMERGED IN = ρgb∫ h2dh
Where,
LIQUID
∫dA.h = Io is the moment of Inertia of
2

Consider a plane vertical surface of


surface about free surface of liquid.
arbitrary shape immersed in a liquid as
shown ∑ moments = ρgI0 …(2)
A = Total area of surface ∴ Ft .h* = ρgIo
h = Distance of C. G. of the area from free
ρgIo
surface of liquid h* =
G = Centre of Gravity of plane surface ρgh.A
P = Centre of Pressure I0
h∗ = Distance of centre of pressure from h* =
hA
free surface of liquid. Where,
h is the distance of C.G. from free
surface
A is the area.
From II axis theorem
=
Io IC.G. + Ah2

*IC.G. +Ah2
a) Total Pressure h=
h.A
Pressure Intensity at strip = ρgh 1) h∗ lies below the C.G. of the surface
Area of strip dA = b. dh

© Copyright Reserved by Gateflix.in No part of this material should be copied or reproduced without permission
2) It is independent of the density of ∴ F = ∫ρg y sinθdA
liquid & depends only on surface
area. But ∫y dA = Ay is the moment of

3.5.4 HORIZONTAL PLANE SURFACE surface at distance ‘y’


SUBMERGED IN LIQUID
∴ F = ρg sinθAy
∴ F = ρg A h (Q h = y sin θ)
As every point of the surface is at the same Note: the above expression of force is
depth from free surface of the liquid, the for fluid with no pressure acting on the
pressure intensity will be equal on the surface. If pressure acts on the surface
entire surface and equal to P = ρgh where =
F P0 A + ρg Ah
h is depth of surface
=
F1 ρg h × Area
b) Centre of Pressure
h= h= h* Pressure force on the strip,
dF = ρgh dA = ρgy sin θdA
3.5.5 INCLINED PLANE SUBMERGED IN
Moment of the force, dF, about axis O-O
LIQUID
= dF × y
= ρgy sinθdA.y
Sum of moments of all such forces
about O − O = ∫ρgsinθy 2dA

= ρgsinθ∫y 2dA
Where,
∫y dA = Moment of Inertia of the
2

surface about O − O = Io
∴ Sum of moments of all force
= ρg sinθIo
Let
F × y* =
ρg sinθIo
A = Total area of surface
h = Distance of C. G. of the area from free ρgsinθIo
y* =
surface of liquid ρg Ah
G = Centre of Gravity of plane surface I sinθ
P = Centre of Pressure y* = o
h∗ = Distance of centre of pressure from Ah
free surface of liquid. I sin 2 θ
h* = 0
Ah
a) Total Pressure sin 2 θ
Pressure intensity on the strip P = ρgh = h *
(IG + Ay 2 )
Ah
Pressure force dF on the strip
sin2 θ  Ah2 
dF = P × dA = ρghdA = h*  IG + 
Ah  sin 2 
θ
Total pressure force on the whole area,  
F= ∫ dF= ∫ ρghdA
a) Rectangle
*
h h h
From fig. sinθ= = = *
y y y
∴ h = y sinθ

© Copyright Reserved by Gateflix.in No part of this material should be copied or reproduced without permission
ab3
A=ab, I xx,C = πR 2
12 =A = , I xx,C 0.109757R 4
b) Circle 2
f) Semi ellipse

πR 4
A=
πR , I xx,C
2
=
4
c) Triangle πab 2
=A = , I xx,C 0.109757ab3
2

3.5.6 CURVED SURFACE SUB-MERGED IN


LIQUID

For a submerged curved surface, the


determination of the resultant hydrostatic
force is more involved since it typically
πab3 requires the integration of the pressure
A=
πab, I xx,C = forces that change direction along the
4 curved surface. The way to determine the
d) Triangle resultant hydrostatic force FR acting on a
two-dimensional curved surface is to
determine the horizontal and vertical
components Fx and Fy separately. This is
done by considering the free-body diagram
of the liquid block enclosed by the curved
surface and the two plane surfaces (one
horizontal and one vertical) passing
through the two ends of the curved surface.
Note that the vertical surface of the liquid
block considered is simply the projection of
ab ab3
=A = , I xx,C the curved surface on a vertical plane, and
2 36 the horizontal surface is the projection of
e) Semicircle the curved surface on a horizontal plane.

© Copyright Reserved by Gateflix.in No part of this material should be copied or reproduced without permission
The resultant force acting on the curved Where,
surface is given by = ρ 1000kg
= / m3 , g 9.81m / s 2
=
FR Fx + Fy
2 2
1
A = 3 × 2 = 6m 2 , h = × 3 = 1.5m
Inclination of resultant with horizontal is 2
given by ∴= F 1000 × 9.81× 6 ×1.5
Fy = 88290N
tan θ =
Fx
1) The horizontal component of the Depth of centre of pressure is given by
hydrostatic force acting on a curved equation as
surface is equal (in both magnitude and IG
=h* +h
the line of action) to the hydrostatic Ah
force acting on the vertical projection of Where,
the curved surface. IG = M.O.I. about C.G. of the area of
2) The vertical component of the
surface
hydrostatic force acting on a curved
surface is equal to the weight of liquid bd 3 2 × 33
= = = 4.5m 4
supported by the curved surface. 12 12
4.5
Example: ∴=
h* + 1.5
= 0.5+1.5 = 2.0m
6 ×1.5
A rectangular plane surface is 2m wide and
3m deep. It lies in vertical plane in water. b) Upper edge is 2.5, below water
Determine the total pressure and position surface
of centre of pressure on the plane surface
when it’s upper edge is horizontal and (a)
coincides with water surface, (b) 2.5m
below the free water surface.

Solution:
Given:
Width of plane surface, b=2m
Depth of plane surface, d=3m

a) Upper edge coincides with water


surface
Total pressure is given by equation as Total pressure (F) is given by
F = ρ gAh F = ρ gAh

© Copyright Reserved by Gateflix.in No part of this material should be copied or reproduced without permission
Where, π 4
h =Distance of C.G. from free surface of IG d
h* = +h= + 4.0 64
water Ah π 2
3 d × 4.0
= 2.5+ = 4.0m 4
2 π 4

∴ F =1000×9.81×6×4.0 Q IG = d 
= 235440N  64 
Center of pressure is given by d2 32
= +4.0= +4.0=0.14+4.0= 4.14m
I 16×4.0 16×4.0
h* = G + h
Ah
Where
IG = 4.5, A = 6.0,h = 4.0
4.5
h* = + 4.0
6.0×4.0
= 0.1875+ 4.0 = 4.1875= 4.1875m.

Example:
A circular opening, 3m diameter, in a The force F is acting at a distance of
vertical side of a tank is closed by disc of 4.14 m from free surface.
3m diameter which can rotate a horizontal Moment of this force about horizontal
diameter. diameter X-X
Calculate: = F ( h* -h ) = 277368 ( 4.14 - 4.0) =38831Nm
i) The force on the disc, and Hence a torque of 38831 Nm must be
ii) The torque required to maintain the applied on the disc in the clockwise
disc in equilibrium in the vertical direction.
position when the head of water above
the horizontal diameter is 4m. Example:
Solution: A pipe line which is 4m in diameter
Given: contains a gate valve. The pressure at the
Dia. of opening d =3m centre of the pipe is 19.6N/ cm2 . If the pipe
π 2 2 is filled with oil of S.G. 0.87; find the force
Area, A = ×3 =7.0685m
4 exerted by the oil upon the gate and
Depth of C.G. h = 4m position of centre of pressure.
Solution:
i) Force on the disc is given by equation as
F = ρ gAh =1000×9.81×7.0685×4.0
= 277368N = 277.368kN
ii) To find the torque required to maintain
the disc in equilibrium, first calculate
the point application of force acting on Given:
the disc, i.e., center of pressure of the Dia. of pipe, d = 4m
force F. The depth of centre of pressure ∴ Area,
(h*) is given by equation as π
A = ×42 = 4pm2
4
∴Density of oil ρ=0
0.87 × 1000
= 870kg / m3

© Copyright Reserved by Gateflix.in No part of this material should be copied or reproduced without permission
∴ Weight density of oil, Solution:
w 0 = ρ0 × g = 870×9.81N/ m3 Given:
Pressure at the centre of pipe, Width of gate, b = 2m
Depth of gate, d =1.2m
p = 19.6N/ cm2 = 19.6×104 N/ m3
∴ Area,
∴ Pressure head at the centre
A = b×d = 2×1.2= 2.4m2
p 19.6×104
= = = 22.988m Sp. gr. of liquid =1.45
w 0 870×9.81 ∴ Density of liquid,
∴ The height of equivalent free oil surface ρ1 =1.45×1000 =1450kg/ m3
from the centre of pipe =22.988m
The depth of C.G. of the gate valve from free Let F1 = Force exerted by the fluid of sp. gr
oil surface h = 22.988m 1.45 on gate
F = ρ gAh F2 = Force exerted by water on the gate.
Where ρ = density of oil = 870kg/ m3 The force
F = 870×9.81×4π ×22.988 = 2465500N = 2.465MN F1 = is given by F1 = ρ1 g× A× h1
(ii)Position of centre of pressure (h*) is Where
given as ρ1 =1.45×1000 =1450kg/ m2
π 4 h1 = Depth of C.G. of gate from free surface
IG d 2 2
d 4
h*= +h = 64 +h = +h = of liquid
+22.988
Ah π 2 16h 16×22.988 1.2
d ×h =1.5+ = 2.1m
4 2
= 0.043+22.988 = 23.031m
Or, centre of pressure is below the centre of
∴ F1 = 1450×9.81×2.4×2.1 = 71691N
the pipe by a distance of 0.043m Similarly, F2 = ρ2 g .Ah2
Where ρ2 =1000kg/ m3
h2 = Depth of C.G. of gate from free surface
Example:
A vertical sluice gate is used to cover an of water
opening in a dam. The opening is 2m wide 1
= ×1.2= 0.6m
and 1.2 m high. On the upstream of the 2
gate, the liquid of S.G. 1.45 lies upto a ∴ F2 = 1000×9.81×2.4×0.6 = 14126N
height of 1.5m above the top of the gate.
Find the resultant force acting on the gate (i)Resultant force on the gate
and position of centre of pressure. Find
= F1 − F2 = 71691 − 14126 = 57565N
also the force acting horizontally at the top
of the gate which is capable of opening it. (ii) Position of centre of pressure of
Assume that the gate is hinged at the resultant force.
bottom. The force F1 will be acting at a depth of
h1 * from free surface of liquid, given by the
relation
I
h* = G + h1
Ah
where
bd3 2×1.23
IG = = = 0.288m4
12 12

© Copyright Reserved by Gateflix.in No part of this material should be copied or reproduced without permission
∴ ii) the position of centre of pressure from
.288 one side of the tank, which is 2m wide
h1 * = +2.1= 0.0571+2.1= 2.1571m
2.4×2.1 Solution:
Given:
∴ Distance of F1 from hinge Depth of water = 0.5m
Depth of liquid = 1m
= (1.5+1.2) − h1 * = 2.7 − 2.1571= 0.5429m Sp. gr of liquid = 0.8
The force F2 will be acting at a depth of h2 * Density of liquid,
from free surface of water and is given by ρ1 = 0.8×1000 = 800kg/ m3
I Density of water,
h2 * = G + h2
Ah2 ρ2 =1000kg/ m3
Where Width of tank = 2m
IG = 0.288m4 ,h2 = 0.6m, A = 2.4m2 (i) Total pressure on one side is
calculated by drawing pressure diagram,
.288 which is shown in fig .
h2 * = +0.6 = 0.2+0.6 = 0.8m
2.4×0.6 Intensity of pressure on top, pA = 0
Distance of F2 from hinge Intensity of pressure on D (or DE),
=1.2 − 0.8 = 0.4m pD = ρ1 g .h1
The resultant force 57565N will be acting = 800×9.81×1.0 = 7848N/ m2
at a distance given by
71691×.5429 − 14126×0.4
=
57565
38921-5650.4
= m above hinge
57565
= 0.578m above the hinge

(iii)Force at the top of gate which is Intensity of pressure on base (or BC),
capable of opening the gate. pB = ρ1 gh1 + ρ2 g×0.5
Let F is the force required on the top of the 12753N
gate to open it as shown in fig. =7848+1000×9.81×0.5=7848+ 4905=
m2
Taking the moments of F, F1andF2 about
Now Force,
the hinge, we get F1 = Area of ∆ ADE × Width of tank
F×1.2+ F2 × 0.4 =F1 × .5429
1 1
Or = × AD×DE×2.0= ×1×7848×2.0=7848N
2 2
F1 ×.5429 − F2×0.4 Force,
F=
1.2 F2 = Areaof rectangle DBFE× Width of tank
71691×.5429-14126×0.4 38921-5650.4 = 0.5×7848×2=7848N
= =
1.2 1.2 F3 = Area of ∆ EFC × Width of tank
= 27725.5N.
1 1
= ×EF×FC×2.0= ×0.5×4905×2.0= 2452.5N
Example: 2 2
A tank contains water up to a height of ∴ Total force F = F1 + F2 + F3
0.5m above the base. An immiscible liquid =7848+7848+2452.5=18148.5N
of sp. gr. 0.8 is filled on the top of water up
to 1m height. Calculate: (ii) Centre of pressure (h*). Taking the
i) total pressure on one side of the tank, moments of all forces about A, we get

© Copyright Reserved by Gateflix.in No part of this material should be copied or reproduced without permission
2  1   2  i) Total Pressure (F)
F×h* = F1 × AD+F2  AD+ BD  +F3  AD+ BD
3  2   3  F = ρ gAh
2 =1000×9.81×7.0685×2.749=190621N
18148.5 × h=
*
7848 × × 1 ii) Centre of pressure (h*)
3
 0.5  Using equation, we have
+7848  1.0 + 
 2  IGsin2 θ
2  h* = +h
+2452.5 1.0 + × 0.5  Ah
 3  Where
=5232+9810+3270=18312 π π
IG = ( d ) = (3) =3.976m4
4 4
18312
∴ h* = = 1.009m from top 64 64
18148.5
3.976× ( .8333) ×.8333
h* = +2.749=0.1420+2.749
Example: 7.0685×2.749
A circular plate 3.0m diameter is immersed = 2.891m.
in water in such a way that their greatest
and least depths below the free surface are Example:
4m and 1.5m respectively. Determine the If in the above problem, the given circular
total pressure on one face of the plate and plate is having a concentric circular hole of
position of the centre of pressure. diameter 1.5m, then calculate the total
Solution: pressure and position of the centre of
Given pressure on one face of the plate.
Dia. of plate, d =3.0m Solution:
∴ Area, Given: [referring to given figure]
π π Dia. of plate, d =3.0m
( )
2
A = d2 = 3.0 = 7.0685m2 ∴ Area of solid plate
4 4
π π 2
Distance DC =1.5m,BE = 4m ()
= d2 = 3 =7.0685m2
4 4

Distance of C.G. from free surface


= h = CD+GC sin θ = 1.5+1.5sin θ Dia. of hole in the plate, d0 = 1.5m
But
π π
∴ Area of hole = d20 = (1.5) =1.7671m2
2
AB BE- AE 4.0- DC 4.0-1.5
sin θ = = = = 4 4
BC BC 3.0 3.0
∴ Area of the given plate A
2.5
= = 0.8333 = Area of solid plate − Area of hole
3.0
=7.0685-1.7671=5.3014m2
∴ h =1.5+1.5×.8333=1.5+1.249= 2.749m
Distance of CD =1.5,BE = 4m
Distance of C.G. from the free surface,

© Copyright Reserved by Gateflix.in No part of this material should be copied or reproduced without permission
h = CD+GCsin θ A = Area of gate =1.2×5.0 = 6.0m2
=1.5+1.5sinθ Depth of C.G. of the gate from free surface
But of the water = h
AB BE- AE 4 -1.5 2.5 = DG = BC- BE
sin θ = = = =
BC BC 3 3 =5.0 − BG sin45°
2.5 1
∴ h =1.5+1.5× =1.5+1.25= 2.75m 5.0 − 0.6 × =
4.576m
3 2
i) Total pressure force (F) The total pressure force (F) acting on the
F = ρ gAh gate,
=1000×9.81×5.3014×2.75 F = ρgAh
=143018N =143.018kN = 1000 × 9.81 × 6.0 × 4.576
ii) Position of centre of pressure (h*) = 269343N
Using equation, we have This force is acting at H, where the depth of
I sin2 θ h from free surface is given by
h* = G +h
Ah I sin2 θ
where h* = G +h
Ah
π π
IG = d4 - d04  = 34 -1.54  m4 Where, IG = M.O.I. of gate
64 64
π π bd3 5.0×1.23
A = d2 - d20  = 32 -1.52  m2 = = = 0.72m
4 4 12 12
2.5 ∴ Depth of centre of pressure
sin θ = and h = 2.75 0.72×sin2 45°
3 h* = +4.576 = .013+4.576 = 4.589m
π 4  
2 6×4.576
 2.5
3 − 1.5  ×  3 
4

64 h*
∴ h* =   +2.75 But from fig, = sin45°
π 2 OH
3 − 1.52  ×2.75 ∴
4
= 0.177+2.75= 2.927m h* 4.589
OH = = = 4.589× 2 = 6.489m
sin45° 1
Example: 2
An inclined rectangular sluice gate AB,
5
1.2m x 5m size as shown in fig is installed Distance, BO = =5× 2 =7.071m
sin45°
to control the discharge of water. The end A
Distance, BH = BO− OH = 0.071 − 6.489= 0.582m
is hinged. Determine the force normal to
the gate applied at B to open it.
∴ Distance,
AH = AB− BH =1.2 − 0.582= 0.618m
Taking the moments about the hinge A
P× AB = F× ( AH )
Where P is the force normal to the gate
applied at B
∴ P×1.2= 269343×0.618
269343×0.618
∴ P= =138708N
1.2
Solution:
Given: Example:

© Copyright Reserved by Gateflix.in No part of this material should be copied or reproduced without permission
Fig shows a quadrant shaped gate of radius Example:
2m. Find the resultant force due to water Find the horizontal and vertical component
per meter length of the gate. Find also the of water pressure acting on the face of a
angle at which the total force will act. sector gate of 90° with radius 4m as shown
in fig. Take width of gate as unity.

Solution:
Given:
Radius of gate =2m Solution:
Width of gate =1m Given:
Horizontal Force, Radius of gate, R=4m
Fx = Force on the projected area of the Horizontal component of force acting on
curved surface on vertical plane the gate is
= Force on BO = ρ gAh Fx = Force on area of gate projected on
Where, vertical plane
1 = Force on area ADB
A = Area of BO = 2×1= 2m2 ,h = ×2=1m
2 = ρ gAh
Fx = 1000×9.81×2×1 = 19620N Where A = AB× Width of gate
2 4
×2= m from
= 2× AD×1 (Q AB = 2AD)
This will act at depth of
3 3 = 2×4×sin45° = 8×.707 =5.656m2
free surface of liquid, {Q AD = 4sin 45°}
Vertical Force,
Fy = Weight of water (imagined) supported
AB 5.656
h= = = 2.828m
2 2
by AB
∴ Fx =1000×9.81×5.656×2.828N =156911N
= ρ g× Areaof AOB×1.0
Vertical component
π 4
=1000×9.81× ( 2) ×1.0 =30819N Fy = Weight of water supported or
4
This will act a distance of enclosed by the curved surface
= Weight of water in portion ACBDA
4R 4 × 0.2
= = 0.848m from OB. = ρ g× Area of ACBDA× Width of gate
3π 3π
= 1000 × 9.81 ×  Area of sector ACBOA − Area of ∆ABO  × 1
∴ Resultant force, F is given by
π AO×BO 
F = Fx2 +Fy2 = 9810×  R 2 −  Q ∆ AOB is a right angled 
4 2 
= 196202 + 308192 π 4×4 
=36534.4N. = 9810×  42 −  = 44796N
4 2 
The angle made by the resultant with
horizontal is given by
Example:
F 30819 A cylindrical gate of 4m diameter & 2m
tan θ = y = =1.5708
Fx 19620 long has water on its both sides as shown
in Fig. Determine the magnitude, location
∴ θ = tan-1 1.5708 =57°31'
and direction of the resultant force exerted

© Copyright Reserved by Gateflix.in No part of this material should be copied or reproduced without permission
by the water on the gate. Find also the least i) Resultant force, F is given as
weight of the cylinder so that it may not be
(117720) + (184914)
2 2
F = Fx2 +Fy2 = = 219206N
lifted away from the floor.
ii) Direction of resultant force is given
by
F 184914
tan θ = y = =1.5707
Fx 117720
∴ θ =57°31'

Solution: iii) Location of the resultant force


Given: Force, Fx acts at a distance of
1

Dia. of gate =4m 2×4


Radius =2m = 2.67m from the top surface of
3
(i) The force acting on the left sides of the water on left side, while Fx acts at a
cylinder are 2

The horizontal component, Fx 2


distance of ×2=1.33m from free
1
3
Where Fx = Force of water on area surface on the right side of the cylinder
1

projected on vertical plane .The resultant force Fx in the direction


= Force on area AOC of x will act at a distance of y from the
=ρgA h bottom as
Where A = AC × width = 4×2= 8cm2 Fx × y = Fx 4 − 2.67  − Fx 2 − 1.33
1 2
1
h = × 4 = 2m Or
2 117720=
× y 156960 ×1.33 − 39240 × .67
Fx = 1000×9.81×8×2 = 208756.8 − 26290.8 = 182466
1

=156960N 182466
∴ y= =1.55m from the bottom
Fy = Weight of water enclosed by ABCOA 117720
1

π  π Force Fy acts at a distance 4R from


=1000×9.81×  R 2  ×2.0= 9810× ×22 ×2.0=123276N 1

2  2
4 × 2.0
AOC or at a distance = 0.8488m

Right Side of the Cylinder
from AOC towards left of AOC.

Fx = ρ gA 2h2 = Force on vertical area CO Also Fy acts at a distance 4R = 0.8488m


2
2

= 1000 × 9.81× (2 × 2) ×
2 =39240N from AOC towards the right of AOC. The
2 resultant force Fy will act at a distance
Fy = Weight of water enclosed by DOCD
2
x from AOC which is given by
π  Fy × x = Fy ×.8488 − Fy ×.8488
= ρ g×  R 2  × Width of gate 1 2

4  Or 184914× x =123276×.8488 − 61638×.8488


π = .8488 123276 − 61638
=  52318.4
=1000×9.81× ×22 ×2= 61638N
4
∴ Resultant force in the direction of x, ∴ x = 52318.4 = 0.2829m from AOC
184914
Fx = Fx − Fx =156960 − 39240 =117720N
1 2

Resultant force in the direction of y, iv) Least weight of cylinder. The resultant
Fy = Fy +Fy =123276+61638 =184914N force in the upward direction is
1 2

© Copyright Reserved by Gateflix.in No part of this material should be copied or reproduced without permission
Fy = 184914N
ii) Vertical thrust exerted by water
Thus the weight of cylinder should not
Fy = Weight of water supported by
be less than the upward force Fy
. curved surface OA upto free surface of
Hence, weight of cylinder should be at water
least 184914N = Weight of water in the portion ABO
= ρ g × Area of OAB × Width of dam
Example:
9 
=1000×9.81×  ∫ x×dy  ×1.0
2
 x 
A dam has a parabolic shape y = y 0    0 
 x0   9

as shown in fig. below having x 0 = 6m and =1000×9.81×  ∫ 2y 1/2×dy  ×1.0
 0 
y 0 = 9m. The fluid is water with density
= 1000kg/ m . Compute the horizontal,
3 (Q x = 2y ) 1/2

9
vertical and the resultant thrust exerted by  y 3/2  2
water per meter length of the dam. =19620×   =19620× 93/2 
 (3/ 2)  3 
0

2
=19620× ×27 =353160N
3

iii) Resultant thrust exerted by water

F= FX 2 + F Y2 = 3973052 + 3531602 = 531574N

iv) Direction of resultant is given by


Fy 353160
θ =
tan= = 0.888
Solution: Fx 397305
Given: =θ tan
= −1
0.888 41.63o
Equation of the curve OA is
x
2
x
2
x2 x2 3.6 BUOYANCY & FLOATATION
y = y 0   = 9   = 9× =
 x0  6 36 4
When a body is immersed in a fluid, an
Or x 2 = 4y upward force is exerted on the body; this
∴ x = 4y = 2y 1/2 upward force is known as the buoyant
force. This force is because of difference in
Width of dam, b =1m
pressure.
i) Horizontal thrust exerted by water
Fx = Force exerted by water on vertical
surface OB, i.e., the surface obtained by
projecting the curved surface on
vertical plane
= ρ gAh
9
=1000×9.81× ( 9×1) × =397305N
2

© Copyright Reserved by Gateflix.in No part of this material should be copied or reproduced without permission
Force acting on the element because of
difference in pressure on the top and
bottom.
dF = ρg ( h2 − h1 ) dA
=FB ∫ρg ( h 2 )
− h1 dA
FB = ρgV
Where,
ρ is the density of fluid
V is the volume of body immersed in fluid Meta centric height = GM
or volume of fluid displaced by that body. I ( Moment of Inertia about yy of the plan )
= − BG
The relation ρgV is simply the weight of the Volume of fluid displaced
liquid whose volume is equal to the B.G is the distance between CG & CB points.
immersed volume of the body. Thus the
buoyant force acting on the body is equal to 3.6.3 OSCILLATION OF A FLOATING
the weight of the liquid displaced by the BODY
body. Note that the buoyant force is
independent of the distance of the body When body floats in the fluid and it is given
from the free surface. It is also independent a disturbance in clockwise direction or anti
of the density of the solid body clock wise direction. The body oscillates
about its metacenter. The time period of
3.6.1 CENTRE OF BUOYANCY oscillation is given by

The buoyant force acting on a body k2


T = 2π
immersed in a fluid is equal to the weight of GM.g
the fluid displaced by the body, and it acts Where,
upward through the centroid of the GM is Meta centric ht
displaced volume. The centroid of K is radius of gyration
displaced fluid is known as centre of
buoyancy. 3.6.4 CONDITIONS OF EQUILIBRIUM OF
SUBMERGED & FLOATING BODIES

There are 3 types of equilibrium conditions


i) Stable Equilibrium
ii) Neutral equilibrium
iii) Unstable equilibrium
3.6.2 META CENTRE
It is defined as the point about which a
body starts oscillating when the body is
tilled by a small angle. The Meta centre may
also be defined as the intersection point of
line of action of buoyant force and normal i) Stable equilibrium:
to the body when the body is tilted by an Any small disturbance (someone moves the
angle. ball to the right or left) generates a
restoring force (due to gravity) that returns
it to its initial position.

© Copyright Reserved by Gateflix.in No part of this material should be copied or reproduced without permission
ii) Neutral equilibrium: floating body, the weight of body is equal to
If someone moves the ball to the right or the buoyant force.
left, it will stay at its new location. It has no
tendency to move back to its original 1) Stable Equilibrium: When M is above
location, nor does it continue to move. G, because of a small displacement to
the body in the clock wise direction, the
iii) Unstable Equilibrium: couple between Wt & FB causes rotation
It is a situation, in which the ball may be at in anti-clockwise direction.
rest at the moment, but any disturbance, 2) Unstable Equilibrium: When M is
even an infinitesimal one, causes the ball to below G, because of small displacement
roll off the hill—it does not return to its to the body in the clock wise direction,
original position; rather it moves away the couple between Wt & FB causes
from it. rotation in clockwise direction.
3) Neutral: If M lies at the C.G. of body, the
displacement of body does not result in
3.6.4.1 STABILITY IN SUBMERGED BODIES any couple of Wt & FB. Body remains at
its displaced position.
1) Stable Equilibrium: When W = FB and
point B is above G. A small displacement
in clockwise direction, gives couple due
to FB & weight in anticlockwise
direction. Thus, the body will return to
its original position. Hence, equilibrium
is stable.
2) Unstable Equilibrium: If W = FB and
point B is below point ‘G’. A small
Stable equilibrium
displacement to the body, in the clock
wise direction, gives couple due to
W & FB also in the clockwise direction.
Thus, body will move away from its
original position. Hence, equilibrium is
unstable
3) Neutral Equilibrium: If FB = W and B
& G are at the same point, the
displacement of body does not result in
any couple of Wt & FB. Body remains at Unstable equilibrium
its displaced position
Example:
Find the volume of the water displaced and
position of centre of buoyancy for a
wooden block of width 2.5 m and of depth
1.5m, when it floats horizontally in water.
The density of wooden block is 650kg/ m3
Stable equilibrium Unstable equilibrium and its length is 6.0m.
Solution:
3.6.42 STABILITY IN FLOATING BODY Given:
Width = 2.5m
The stability of floating body is determined Depth = 1.5m
from position of metacentre(M). In case of Length = 6.0m

© Copyright Reserved by Gateflix.in No part of this material should be copied or reproduced without permission
Volume of the block 60
3 = V = 0.6Vm3
= 2.5×1.5×6.0= 22.50m 100
Density of wood, ρ = 65kg/ m3

For the equilibrium of the body


Total buoyant force (upward force)
∴ Weight of block = ρ ×g× Volume =Weight of the body
= 650×9.81×22.50N = 143471N But total buoyant force= Force of buoyancy
For equilibrium, the weight of water due to water + Force of buoyancy due to
displaced = Weight of wooden block mercury
= 143471N Force of buoyancy due to water = Weight of
∴ Volume of water displaced water displaced by body
= Density of water × Volume of mercury
Weight of water displaced 143471
displaced
=14.625m3
= 1000 × g ×Volume of body in water
= =
Weight density of water 1000×9.81
= 1000×g ×0.6× VN
Position of center of Buoyancy: And, force of buoyancy due to mercury
Volume of wooden block in water = Weight of mercury displaced by body
= Volume of water displaced = g × Density of water× Volume of mercury
Or displaced
=g ×13.6×1000× volume of body in
2.5×h×6.0=14.625m3 ,
mercury
Where,
= g ×13.6×1000×0.4VN
h is depth of wooden block in water
Weight of the body
14.625 =Density x g x Volume of body
∴ h= = 0.975m
2.5×6.0 ∴ For equilibrium, we have
∴ Centre of Buoyancy Total buoyant force =Weight of the body
0.975 1000×g×0.6×V+13.6×1000×g×.4V = ρ ×g×V
= = 0.4875m from base
2 Or
ρ =600+13600×.4 =600+54400=6040.00kg/ m3
Example: ∴ Density of the body = 6040.00kg/ m3
Find the density of a metallic body which
floats at the interface of mercury of S.G.
Example:
13.6 and water such that 40% of its volume
A float valve regulates the flow of oil of S.G.
is sub-merged in mercury and 60% in
0.8 into a cistern. The spherical float is 15
water.
cm in diameter. AOB is a weightless link
Solution:
carrying the float at one end, and a valve at
Let the volume of the body = Vm3 the other end which closes the pipe
Then volume of body sub-merged in through which oil flows into the cistern.
mercury The link is mounted on a frictionless hinge
40 at O and the angle AOB is 135°. The length
= V = 0.4Vm3 of OA is 20cm, and the distance between
100
Volume of body sub-merged in water the centre of the float and the hinge 50 cm.
When of the flow is stopped AO will be

© Copyright Reserved by Gateflix.in No part of this material should be copied or reproduced without permission
vertical. The valve is to be pressed on to the = 800×9.81×0.00945=7.416N
seat with a force of 9.81 N to completely The buoyant force and weight of the float
stop the flow of oil into the cistern. It was passes through the same vertical line,
observed that the flow of oil is stopped passing through B.
when the free surface of oil in the cistern is Let the weight of float is W. Then net
35 cm below the hinge. Determine the vertical force on float
weight of the float. =Buoyant force –Weight of float = (7.416-
W)
Taking moments about the hinges O, we get
P×20= (7.416 − W ) ×BD= (7.416 − W ) ×50×cos45°

(
Or 9.81×20 = 7.416 − W ×35.355 )
20×9.81
∴ W =7.416 − =7.416 − 5.55=1.866N
Solution: 35.355
Given:
Sp. gr. of oil =0.8
∴ Density of oil
= ρ0 = 0.8×1000 = 800kg/ m3 Example:
A rectangular pontoon is 5m long, 3m wide
Dia. of float, D=15cm
and 1.20m high. The depth of immersion of
∠ AOB=135°
the pontoon is 0.80m in sea water. If the
OA = 20cm
centre of gravity is 0.6m above the bottom
Force, P=9.81N
of the pontoon, determine the Meta –
OB=50 cm
centric height. The density for sea water
Let the weight be W.
= 1025kg/ m3
When the flow of oil is stopped, the centre Solution:
of float is shown in Fig. The level of oil is Given:
also shown. The centre of float is below the Dimension of pontoon = 5m × 3m × 1.20m
level of oil, by a depth ‘h’ Depth of immersion =0.8m
From ∆ BOD, Distance AG=0.6m
OD OC+CD 35+ h
sin45° = = =
OB OB 50
50×sin45° =35+ h Or
1
h =50× − 35=35.355 − 35= 0.355cm = .00355m
2
The weight of float is acting through B, but
the upward buoyant force is acting through
the centre of weight of oil displaced
Volume of oil displaced :
2  D 15 
= π r3 + h× π r2 r = = =7.5cm 
3  2 2 
2 1
= × π × ( 0.75) +.00355× π × ( 0.75) =0.000945m3
3 2
Distance AB = ×Depth of immersion
3 2
=Weight of oil displaced 1
= ρ0 × g× Volume of oil = ×.8 = 0.4m
2

© Copyright Reserved by Gateflix.in No part of this material should be copied or reproduced without permission
Density for sea water = 1025kg/ m3 π 4 π
D = × ( 4.0)
4
=
Meta-centre height GM, given by equation 64 64
as ∀ = Volume of cylinder in water
I π π
GM = − BG = ×D2×Depth of cylinder in water = ×42 ×2.4m3
∀ 4.0 4
Where I = Moment of Inertia of the plan of π
×44
the pontoon about Y-Y axis 1 1 42 1
∴ = 64 = × = = 0.4167m
1 45 ∀ π 2 16 2.4 2.4
= ×5×33 m4 = m4 ×4 ×2.4
12 4 4
∀ = Volume of the body sub- merged in 1
GM = − BG = 0.4167 − 0.8 = −0.3833m
water ∀
=3×0.8×5.0=12.0m3 -ve sign means that the meta–centre (M) is
BG = AG = AB = 0.6 - 0.4 = 0.2m below the centre of gravity (G). Thus the
45 1 45 cylinder is in unstable equilibrium.
GM = × − 0.2= − 0.2= 0.9375 − 0.2= 0.7375m
4 12.0 48

Example: Example:
A solid cylinder of diameter 4.0 m has a A wooden cylinder of S.G. 0.6 and circular
height of 4.0m. Find the meta-centric height cross–section is required to float in oil (S.G.
of the cylinder if the specific gravity of the 0.90). Find the L/D ratio for the cylinder to
material of cylinder = 0.6 and it is floating float with its longitudinal axis vertical in
in water with its axis vertical. State oil, where L is the height of cylinder and D
whether the equilibrium is stable or is its diameter.
unstable. Solution:
Solution: Given:
Given: D=4m Dia of cylinder =D
Height, h=4m Height of cylinder =L
S.G. = 0.6 Sp. Gr. Of cylinder S1 = 0.6
Depth of cylinder in water = S.G x h
Sp. Gr of oil S2 = 0.9
= 0.6×4.0= 2.4m
∴ Distance of centre of buoyancy (B) from Let the depth of cylinder immersed in oil=h
A
2.4
AB = =1.2m
2
Distance of centre of gravity (G) from A
h 4.0
AG = = = 2.0m
2 2
∴ BG = AG− AB = 2.0 − 1.2= 0.8m
Now the meta–centric height GM is given
by
I For the principle of buoyancy
GM = − BG Weight of cylinder = wt. of oil displaced
∀ π 2 π
Where ×D ×L×0.6×1000×9.81= ×D2×h×0.9×1000×9.81
4 4
I=M. O. I. of the plan of the body about Y-Y
axis
Or L×0.6 = h×0.9

© Copyright Reserved by Gateflix.in No part of this material should be copied or reproduced without permission
0.6×L 2 The time period of rolling of a ship of
∴ h= = L weight 29430kN in sea water is 10seconds.
0.9 3
The centre of buoyancy of the ship is 1.5 m
The distance of centre of gravity G from A,
below the centre of gravity. Find the radius
L of gyration of the ship if the moment of
AG =
2 inertia of the ship at the water line about
The distance of centre of buoyancy B from fore and aft axis is1000m4 . The specific
A weight of sea water as 10100N/ m3
h 1 2  L Solution:
AB = =  L  =
2 2 3  3 Given:
Time period T=10sec
L L 3L− 2L L Distance between centre of buoyancy and
∴ BG = AG− AB = − = =
2 3 6 6 centre of gravity, BG=1.5m
The meta–centric height GM is given by Moment of Inertia, I =10000m4
I Weight W = 29430kN = 29430×1000N
GM = − BG
∀ Let the radius of gyration =K
π 4 First calculating the meta–centric height,
Where I = D and ∀ = Volume of which is given as
64
π 2 I
GM = BM− BG = − BG
cylinder in oil = D ×h ∀
4
Where I= Moment of Inertia
I  π 4 π 2  1 D2 D2 3D2 And ∀ = Volume of water displaced
∴ = D / D h= = =
∀  64 4  16 h 16× 2 L 32L Weight of ship 29430×1000
3 = = = 2912.6m3
Sp.weight of sea water 10104
 2  10000
Q h = L  ∴GM = − 1.5=3.433 − 1.5=1.933m
 3  2912.6
3D2 L K2
∴ GM = − Using equation, T = 2π
32L 6 GM× g
For stable equilibrium, GM should be +ve We get
or, K2 2π K
3D2 L 10 = 2π =
GM > 0 or − >0 1.933×9.81 1.933×9.81
32L 6 Or
Or
10× 1.933×9.81
3D2 L 3×6 L2 K= = 6.93m
> or > 2π
32L 6 32 D2
Or
L2 18 9
< or
D 32
2
16
L 9 3
∴ < =
D 16 4
L
∴ <3/ 4
D

Example:

© Copyright Reserved by Gateflix.in No part of this material should be copied or reproduced without permission
GATE QUESTIONS

Q.1 A static fluid can have c)(ρ−ps)ghA d)(ρh−ρsH)gA


a) non-zero normal and shear [GATE–2003]
stress Q.4 The pressure gauges G1and G2
b) negative normal stress and zero installed on the system show
shear stress pressure of PG1=5.00 bar and PG2=
c) positive normal stress and zero 1.00 bar. The value of unknown
shear stress pressure P is [Atmospheric Pressure
d) zero normal stress and non-zero 1.01 bar]
shear stress
[GATE–2001]
Q.2 The horizontal and vertical
hydrostatic forces Fx and Fy on the
semi-circular GATE, having a width a) 1.01 bar b) 2.01 bar
w into the plane of figure, are c) 5.00 bar d) 7.01 bar
[GATE–2004]

Q.5 A closed cylinder having a radius R


and height H is filled with oil of
density ρ. If the cylinder is rotated
about its axis at an angular velocity
of ω, the thrust at the bottom of the
a) Fx = ρghrw and Fy = 0
cylinder is
b) Fx = 2ρghrw and Fy = 0
a) πR2 ρgH
ρgwr 2 ρω2 R 2
c) Fx = ρghrw and Fy = b) πR 2
2 4
πρgwr 2 c) πR 2 (ρω2 R 2 + ρgH)
d) Fx = 2ρghrw and y
F =
2  ρω2 R 2 
[GATE–2001] d) πR 2  + ρgH 
 4 
Q.3 A cylindrical body of cross-sectional [GATE–2004]
area A, height H and density ρs , is
immersed to a depth h in a liquid of Q.6 Oil in a hydraulic cylinder is compressed
density ρ, and tied to the bottom from an initial volume 2 m3 to 1.96
with a string. The tension in the m3. If the pressure of oil in the
string is cylinder changes from 40 MPa to 80
MPa during compression the bulk
modulus of elasticity of oil is
a) 1000 MPa b) 2000 MPa
c) 4000 MPa d) 8000 MPa
[GATE–2007]

a)ρghA b)(ρs−ρ)ghA Q.7 For the stability of a floating body,

© Copyright Reserved by Gateflix.in No part of this material should be copied or reproduced without permission
under the influence of gravity alone,
which of the following is TRUE?
a) Metacenter should be below
centre of gravity.
b) Metacenter should be above a) 5000 b) 6600
centre of gravity. c) 7546 d) 9623
c) Metacenter and centre of gravity [GATE–2013]
must lie on the same horizontal
line. Q.10 For a completely submerged body
d) Metacenter and centre of gravity with centre of gravity 'G' and centre
must lie on the same vertical of buoyancy ‘B', the condition of
line. stability will be
[GATE–2010] a) G is located below B
b) G is located above B
Q.8 A large tank with a nozzle attached c) G and B are coincident
contains three immiscible, inviscid d) independent of the locations of G
fluids as shown. Assuming that the and B
changes in h1, h2 and h3 are [GATE–2014]
negligible, the instantaneous Q.11 The difference in pressure (in N/m2)
discharge velocity is across an air bubble of diameter
0.001 m immersed in water (surface
tension = 0.072 N/m) is______.
[GATE–2014]

Q.12 An aluminium alloy (density 2600


kg/m3) casting is to be produced. A
 ρ h ρ h  cylindrical hole of 100 mm diameter
a) 2gh 3  1 + 1 1  + 2 2  and 100 mm length is made in the
 ρ3 h 3 ρ3 h 3  casting using sand core (density
b) 2g ( h1 + h 2 + h 3 ) 1600 kg/m3). The net buoyancy
force (in newton) acting on the core
 ρ h +  ρ 2 h 2 + ρ3 h 3  is ______.
c) 2g  1 1  [GATE–2014]
 ρ1  + ρ 2  + ρ3 
 ρ h h +  ρ 2 h 3 h1 + ρ3 h1h 2  Q.13 A spherical balloon with a diameter
d) 2g  1 2 3 
ρ1h1 + ρ 2 h 2 + ρ3 h 3 of 10 m, shown in the figure below is
 
used for advertisements. The
[GATE–2012] balloon is filled with helium (RHe =
2.08 kJ/kgK) at ambient conditions
Q.9 A hinged GATE of length 5 m, of 15°C and 100 kPa. Assuming no
inclined at 30° with the horizontal disturbances due to wind, the
and with water mass on its left, as maximum allowable weight (in
shown in the figure below. The newton) of balloon material and
minimum mass of the GATE in kg rope required to avoid the fall of the
per unit width (perpendicular to balloon (Rair = 0.289 kJ/kgK) is
the plane of paper), required to _____.
keep it closed is

© Copyright Reserved by Gateflix.in No part of this material should be copied or reproduced without permission
volume is in oil while the rest is in
water. The density of the body is
_________ kg/m3. The specific gravity
of oil is 0.7 and density of water is
1000 kg/m3.
Acceleration due to gravity g = 10
m/s2.

[GATE–2014]

Q.14 For a floating body, buoyant force


acts at the
a) centroid of the floating body
b) center of gravity of the body
c) centroid of the fluid vertically
below the body
d) centroid of the displaced fluid
[GATE–2016]
Q.15 An inverted U-tube manometer is
[GATE–2016]
used to measure the pressure
difference between two pipes A and
Q.17 Consider a frictionless, massless and
B, as shown in the figure. Pipe A is
leak-proof plug blocking a
carrying oil (specific gravity = 0.8)
and pipe B is carrying water. The rectangular hole of dimensions 2𝑅𝑅 ×
𝐿𝐿 at the bottom of an open tank as
densities of air and water are 1.16
shown in the figure. The head of the
kg/m3 and 1000 kg/m3,
plug has the shape of a semi-
respectively. The pressure
cylinder of radius R. The tank is
difference between pipes A and B is
filled with a liquid of density ρ up to
__________ kPa.
the tip of the plug. The gravitational
Acceleration due to gravity g = 10
acceleration is g. Neglect the effect
m/s2.
of the atmospheric pressure.

The force F required to hold the plug


in its position is
 π
a) 2ρR2gL 1 − 
 4
 π
b) 2ρR2gL 1 + 
[GATE–2016]  4
c) πR ρgL
2

Q.16 The large vessel shown in the figure π


d) ρR2gL
contains oil and water. A body is 2
submerged at the interface of oil and [GATE–2016(2)]
water such that 45 percent of its

© Copyright Reserved by Gateflix.in No part of this material should be copied or reproduced without permission
Q.18 For the stability of a floating body the c) centre of gravity must be above the centre of
a) centre of buoyancy must coincide with the buoyancy
centre of gravity d) metacentre must be above the centre of
b) centre of buoyancy must be above the centre of gravity
gravity
[GATE–2017(2)]

ANSWER KEY:

1 2 3 4 5 6 7 8 9 10 11 12 13 14
(c) (d) (d) (d) (d) (b) (b) (a) (d) (a) 288 7.7 5303.7 (d)
15 16 17 18
-2.19 865 (a) (d)

© Copyright Reserved by Gateflix.in No part of this material should be copied or reproduced without permission
EXPLANATIONS

Q.1 (c) Q.3 (d)


Fluid static deals with problems Given:
associated with fluids at rest. In Cross section area of body = A
static fluid, there is no relative Height of body = H
motion between adjacent fluid Density of body = ρs
layers and thus there are no shear Density of liquid = ρ
(tangential) stresses in the fluid Tension in the string = T
trying to deform it. We have to make the FBD of the
The only stress in static fluid is the block.
normal stress, which is the pressure B = Buoyancy force
and the variation of pressure is only
due to the weight of the fluid and it
is always positive.
Therefore, the topic of fluid statics
has significance only in gravity field.

Q.2 (d)
T + mg = ρhAg
T + ρs Vg  =
ρhAg
T + ρsAHg = ρhAg
T = ρhAg − ρsAHg = Ag(ρh−ρsH)

Q.4 (d)
Here F1 = weight of water column Given: PG1 = 5.00 bar, PG2 = 1.01 bar
above the top surface. Absolute pressure of G2
F2 = weight of water column above = Atmospheric pressure + Gauge
the bottom surface. pressure
At the depth, pressure is given by =1.01 + 1.00= 2.01 bar
p = ρgh Absolute pressure of G1
then horizontal force, = PG1 + Pabs(G2) = 5.0 + 2.01 = 7.01 bar
Fx = A×p = (2r×w) ρgh
where Q.5 (d)
A = Normal area, when viewed in
the direction of Fx
Fx = 2ρghrw
Fy = F2− F1 = weight of water
contained in volume of semi circular
π 
GATE. = Fy mg =  r 2 × w  ρg
2 
m = ρV and V = A × w
πρgwr 2
Fy =
2

© Copyright Reserved by Gateflix.in No part of this material should be copied or reproduced without permission
Total thrust at the bottom of Given data :
cylinder = Weight of water in V1 = 2 m3
cylinder + Pressure force on the
V2 = 1.96 m3
cylinder
For rotating motion, ∴ dV = V2 − V1
∂p ρV 2 ρr 2 ω2 =1.96 − 2 =− 0.04 m3
= = = ρω2 r
∂r r r p1 = 40 MPa
∂p = ρω2rdr
p 2 = 80 MPa
Integrating both the sides
P R ∴ dp = p 2 − p1 =80 − 40 = 40 MPa
∫∂p =∫ρω rdr
2
Bulk modulus of elasticity.
0 0
dp dp −2 40
K=− =
− V1 =
R
 r2 
[p] = ρω  
P
0
2
dV/V1 dV −0.04
 2 0 = 2000 MPa
ρω2 r 2
P=
2 Q.7 (b)
Dividing whole area of cylinder in
the infinite small rings with
thickness dr,
Force on elementary ring =
Pressure intensity × Area of ring
ρω2 r 2
= × 2πrdr
2
ρω2 r 2
R
=
Total F   ∫0 2 × 2πrdr
As shown in figure above. If point B’
R
R4 is sufficiently far from B, these two
= πρω2 ∫r 3dr = πρω2
4 forces
0
(Gravity force and Buoyant force)
Weight = mg = 𝜌𝜌Vg = ρπR2Hg
create a restoring moment and
ω2 R 4 return the body to the original
So, Net force = ρπ + ρπR 2 Hg
4 position.
 ρω R 
2 2
A measure of stability for floating
πR 2 ρgH +
 4  bodies is the metacentric height GM,
which is the distance between the
Q.6 (b)
centre of gravity G and the
metacenter M (the intersection
point of the lines of action of the
buoyant force through the body
before and after rotation.)
A floating body is stable if point M is
above the point G, and thus GM is
positive, and unstable if point M is
below point G, and thus GM is
negative. Stable equilibrium occurs
when M is above G

Q.8 (a)

© Copyright Reserved by Gateflix.in No part of this material should be copied or reproduced without permission
Converting all the height
corresponding to liquid (3)
x 3ρ3 = h1ρ1
ρ
X 3 = h1 1
ρ3
y3ρ3 = h 2ρ 2

y3 = 2 2 (A submerged body in stable
ρ3
equilibrium)
ρ ρ
H= h 3 + h1 1 + h 2 2 ∴
ρ3 ρ3 Q.11 (288)
Instantaneous discharge velocity 2σ
For bubble, ∆P =
= 2gH r
 (this is a case of droplet)
ρ ρ 
= 2g  h 3 + h1 1 + h 2 2  where,
 ρ3 ρ3  ∆P is pressure difference
 ρ h ρ h  σ is surface tension = 0.072 N/m
=
V 2gh 3 1 + 1 1 + 2 2  0.001
 ρ3 h 3 ρ3 h 3  r is bubble radius =
2
m
= 0.0005m
Q.9 (d) 2 × 0.072
∴ ∆P=
0.0005
= 288 N/m2

Q.12 (7.7)
π 2
Volume of core = DH
4
= 0.785×0.12 × 0.1
FR = �ρgh� × A
= 7.85×10−4 m3
= 103 × 9.81× 2.5sin sin 30° × 5 ×1 Net buoyancy force
=61.31kN =(ρAl – ρcore)Vcoreg
l sin 2 θ = 1000×9.81×7.85×10−4
h cp= h + G
Ah = 7.7 N
1× 53 0.25
= 1.25 + × Q.13 (5303.688)
12 5 ×1×1.25
= 1.667m Mass of Balloon:-
4
h cp
= 3.34m Volume, V = πr 3
sin 30° 3
Taking moment about the hinge 4 × 3.14 × 53
=
61.31×103 × 3.34 3
= m × 9.81× 2.5cos 30° = 523.33 m3
m = 9622.8kg P 100 ×103
ρ= =
RT 2080 × 288
He

Q.10 (a) = 016693 kg/m3


Total mass = 0.16693 × 523.33
= 87.359 kg

© Copyright Reserved by Gateflix.in No part of this material should be copied or reproduced without permission
Mass of Air:-
P 100 ×103 Q.16 (865)
ρ =
= Weight of the body will be balanced
RT 289 × 288
= 1.2 kg/m3 by the buoyant force
Mass of displaced volume of air ∴ FB = mg
= 1.2 × 523.33 = 628 kg ( 0.7ρω )( 0.45V ) g + ρω ( 0.55V ) g =
ρs Vg
Net Buoyant Mass = 628 − 87.359 ⇒ 700×0.45 + 1000 ×0.55 = ρs
= 540.64 kg ρs = 865kg/m3
∴Maximum allowable weight =
540.64 × 9.81 Q.17 (a)
= 5303.688 N Force required
 πR 2 
Q.14 (d) =  2R × R × L − L  ρg
 2 
Q.15 (−2.19)  π
= 2ρR 2 gL 1 − 
 4
Q.18(𝒅𝒅)

For stable equilibrium

Given data
Specific gravity of oil
Soil=0.8
∴ ρoil =0.8× 1000 = 800kg/m3
Density of air ρoil = 1.16 kg /m3
Density of water
ρoil =1000 kg/m3
Acceleration due to gravity
g = 10 kg /m2
h1 = 80 mm = 0.08 m
h2 = 200 mm = 0.2 m
h3 = 100 mm = 0.1 m
Pressure at section1 on left limb =
Pressure at section 1 on right limb
pa - ρoil gh2 – ρair gh1= pb – ρwater g(h1+
h2+ h3)
pa - 800×10×0.2-1.16×10×0.08= pb
– 1000×10(0.08+ 0.2+ 0.1)
pa – 1600-0.928= pb – 3800
pa –pb= -2199.07Pa
= -2.199KpA

© Copyright Reserved by Gateflix.in No part of this material should be copied or reproduced without permission
4 KINEMATICS

4.1 INTRODUCTION

Fluid kinematics deals with describing the constant mass flow rate. The velocity of
motion of fluids without necessarily fluid may vary with position but remains
considering the forces and moments that constant w.r.t time.
cause the motion. The velocity at any point
in a flow field at any time is studied in this
branch of fluid mechanics. Once the
velocity is known, then the pressure
distribution & hence forces acting on the
fluid can be determined.

4.2 METHODS OF DESCRIBING FLUID


MOTION Unsteady flow is that type of flow in which
the velocity, pressure, density at a point
1) Lagrangian Method: A single fluid changes with respect to time.
particle is followed during its motion  dP 
  ≠0
and its velocity, acceleration, density  dt ( x ,y ,z )
etc. are described. o o o

(
P = f x,y,z,t )
2) Eulerian Method: The velocity, acc,
density etc. are described as a point in Example:
flow field. It is commonly used in fluid Fluid flowing in pipe of uniform cross
mechanics. section or varying cross section with
variable mass flow rate
4.3 TYPES OF FLUID FLOW
4.3.2 UNIFORM FLOW & NON UNIFORM
4.3.1 STEADY & UNSTEADY FLOW FLOW
Steady flow is defined as that type of flow in Uniform flow is defined as that type of flow
which the fluid characteristics like velocity in which the velocity at any given time does
etc. at a point do not change with time. not change w.r.t. space.
 dP 
  =0  dP 
 dt ( xo ,y o ,zo )   =0
 ds t
(
P = f x,y,z )
Where, Example:
P is flow parameter (velocity, acceleration, Fluid flowing through uniform cross
density etc) section. The velocity remains constant
w.r.t. space. The mass flow rate can vary
Example: w.r.t. time.
Fluid flowing in pipe of uniform cross
section or varying cross section with

© Copyright Reserved by Gateflix.in No part of this material should be copied or reproduced without permission
i) Fluid flowing between two parallel
plates

V1 = V2

Non Uniform flow is that type of flow in Velocity is along ‘x’ direction
which velocity changes with respect to U = f (y)
space at given instant. ii) Velocity flow field given by
Example: V ax ˆi + bxjˆ
=
Fluid flowing through non-uniform cross Here velocity has two components but
section. The velocity varies w.r.t space. The depends only on one dimension.
mass flow rate can vary Therefore one dimensional flow

2) Two dimensional: Two dimensional


flows is that type of flow in which the
flow parameter such as velocity is a
function of time and two space
coordinates.
Velocity flow field given by
V1 ≠ V2 =V ax ˆi − byjˆ

1) Steady Uniform flow: Flow at constant 3) Three dimensional: Three


rate through a duct of uniform cross- dimensional flows is that type of flow in
section. which the flow parameter such as
2) Steady non-uniform flow: Flow at velocity is a function of time and three
constant rate through a duct of non- spaces coordinate.
uniform cross-section (tapering pipe). Velocity flow field given by
3) Unsteady Uniform flow: Flow at V = ax ˆi + byˆj + czk̂
varying rates through a long straight
pipe of uniform cross-section. (Again =V axyˆi − byzjˆ
the region close to the walls is ignored.
4) Unsteady non-uniform flow: Flow at 4.3.4 LAMINAR AND TURBULENT FLOWS
varying rates through a duct of non-
uniform cross-section Viscous flow regimes are classified as
laminar or turbulent on the basis of flow
4.3.3 ONE-, TWO- AND THREE- structure. In the laminar regime, flow
DIMENSIONAL FLOWS structure is characterized by smooth
motion in lamina or layers. Flow structure
1) One dimensional: One dimensional in the turbulent regime is characterized by
flow is that type of flow in which the random, three-dimensional motions of fluid
flow parameter such as velocity is a particles in addition to the mean motion.
function of time and one space In laminar flow there is no mixing between
coordinate only. adjacent fluid layers. A thin filament of dye
injected into a laminar flow appears as a
Example: single line; there is no dispersion of dye
throughout the flow. A dye filament

© Copyright Reserved by Gateflix.in No part of this material should be copied or reproduced without permission
injected into turbulent flow disperses The ratio of the flow speed V, to the local
quickly throughout the flow field; the line speed of sound C, in the gas is defined as
of dye breaks up into entangled threads of the Mach number.
dye. This behaviour of turbulent flow is due V
to the velocity fluctuations present; the M=
C
mixing of fluid particles from adjacent Thus gas flows with M<0.3 can be treated
layers of fluid results in rapid dispersion of as incompressible.
the dye. If one measures the x component
of velocity at a fixed locations in a pipe for 4.3.6 INTERNAL AND EXTERNAL FLOWS
both laminar and turbulent steady flow, the
traces of velocity versus time appears. In Flows completely bounded by solid
the turbulent flow the flow velocity trace surfaces are called internal or duct flows.
indicates random fluctuations of the Flows over bodies immersed in an
instantaneous velocity, unbounded fluid are termed external flows.
u= u + u′ Both internal and external flows may be
In one dimensional laminar flow, the shear laminar or turbulent, compressible or
stress is related to the velocity gradient by incompressible.
the simple relation
Tyx = µdu / dy 4.4 CONTINUITY EQUATION IN THREE-
DIMENSIONS
In turbulent flow there is no universal
relationship between the stress field and Continuity equation is based on mass
the mean velocity field. Thus in turbulent conservation principle.
flows we must rely heavily on semi (mass flow rate in) - (mass flow rate out) =
empirical theories and on experimental (rate of change of mass in control volume)
data.
dm
The type of flow is determined by min − mout =
Reynold’s No. dt
In Cartesian coordinate system
ρVL
Re = ∂ ∂ ∂ ∂ρ
µ
∂x
( )
ρu +
∂y
( )
ρv +
∂z
ρw + ( )
∂t
=0
Where,
ρ =Density Where,
L = Characterstic length u = f1 ( x,y,z )
V = Velocity ( )
v = f2 x,y,z
μ = Dynamic Viscosity
w = f ( x,y,z )
3

4.3.5 COMPRESSIBLE & INCOMPRESSIBLE This equation is applicable to


FLOWS
1. Steady & unsteady
Flows in which variations in density are 2. Compressible, incompressible.
negligible are termed incompressible; 3. Uniform, non uniform
when density variations within a flow are ∂ρ
not negligible, the flow is called For steady flow =0
∂t
compressible. Gas flowing with negligible ∂ ∂ ∂
heat transfer may also be considered
∂x
( )
ρu +
∂y
ρv +
∂z
( )
ρw =
0 ( )
incompressible provided that the flow
speed is small relative to the speed of
sound. For incompressible & steady flow

© Copyright Reserved by Gateflix.in No part of this material should be copied or reproduced without permission
∂ ∂ ∂ The magnitude of resultant velocity is given
∂x
( )
u +
∂y
v + ( )
∂z
w =
0 ( ) by
For two dimensional, incompressible & V= u2 + v 2 + w 2
steady flow
∂ ∂ The acceleration is given by
∂x
( )
u +
∂y
v = ( )
0
a = a xi + a yj + a z 
k
Where,
4.5 CONTINUITY EQUATION IN ONE
du dv dw
DIMENSION =ax = , ay = , az
dt dt dt
For steady state condition du ∂u  ∂x  ∂u  ∂y  ∂u ∂z ∂u
ax = =   +   + . +
(massin ) = (massout ) dt ∂x  ∂t  ∂y  ∂t  ∂z ∂t ∂t
ρ1 A1 V1 = ρ2 A2 V2 u.∂u v.∂u w.∂u ∂u
ax = + + +
For incompressible fluid ∂x ∂y ∂z ∂t
ρ1 = ρ2 u.∂v v.∂v w.∂v ∂v
ay = + + +
∂x ∂y ∂z ∂t
A1 V1 = A 2 V2
u.∂w v.∂w w.∂w ∂w
Continuity equation is valid for az = + + +
incompressible steady state. ∂x ∂y ∂z ∂t
For steady state,
4.6 MOTION OF FLUID ELEMENT dv
=0
dt
a) linear motion ∂u ∂v ∂w
b) Rotation motion = 0,= 0,= 0
∂t ∂t ∂t
c) linear deformation
Hence acceleration
d) angular deformation
u.∂u v.∂u w.∂u
ax = + +
4.6.1 LINEAR MOTION ∂x ∂y ∂z

In pure translation motion the fluid particle 4.6.3 LOCAL ACCELERATION &
retains its shape. It does not deform. CONVECTIVE ACCELERATION

Local acceleration is defined as the rate of


change of velocity with respect to time at a
given point in a flow field.
∂u ∂v ∂w
4.6.2 VELOCITY & ACCELERATION , ,
∂t ∂t ∂t
The velocity in flow field is given by
4.6.4 CONVECTIVE ACCELERATION
V = uˆi + vˆj + wkˆ
Where, Convective acceleration is defined as
(
u = f1 x,y,z ) change of velocity with change in position
of fluid particle in a fluid flow.
v = f ( x,y,z )  u.∂u u.∂u u.∂v 
2
 + + 
w = f ( x,y,z )  ∂x ∂y ∂z 
3

© Copyright Reserved by Gateflix.in No part of this material should be copied or reproduced without permission
4.6.5 FLUID ROTATION ∂u ∂v ∂w
ε xx= , ε yy= , εzz=
∂x ∂y ∂z

4.6.7 ANGULAR DEFORMATION/SHEAR


STRAIN

Rotation of fluid particle is a vector


quantity given by
ω = ωx ˆi + ωy ˆj + ωz kˆ

Where, Rate of shear strain is given by


ωx is rotation about x axis, 1  ∂u ∂v 
ε=  + 
xy
2  ∂y ∂x 
ωyis rotation about y axis &
4.7 FLOW PATTERNS
ωz is rotation about z axis
In vector notation Fluid Mechanics is a subject with
1  visualisations. Patterns of flow can be
ω= (∇ × V) visualized in several ways. Basic types of
2 line patterns used to visualize flow are
streamline, path line, streak line and time
Rotational Components are given by line.
1  ∂v ∂u 
=
ωz  −  4.7.1 STREAM LINE
2  ∂x ∂y 
1  ∂u ∂w  Stream lines are lines drawn in the flow
=
ωy  − 
2  ∂z ∂x  field so that the tangent at any point gives
the direction of velocity of particle at that
1  ∂w ∂v  point. Since the streamlines are tangent to
=
ωx  − 
2  ∂y ∂z  the velocity vector at every point in the
For irrotational fluid flow, flow field, there can be no flow across a
streamline.
ωx =ωy =ωz =0
Differential equation of streamline
ds
4.6.6 LINEAR STRAIN RATE =V
dt
dx dy dz
It is defined as the rate of increase in length = = =
dt
per unit length. Mathematically, the linear u v w
strain rate of a fluid element depends on dx dy dz
= =
the initial orientation or direction of the u v w
line segment upon which we measure the
linear strain 4.7.11 STREAM TUBE

A bundle of neighboring streamlines may


be imagined to form a passage through
which the fluid flows. This passage is
known as a stream-tube.
Linear strain is given by

© Copyright Reserved by Gateflix.in No part of this material should be copied or reproduced without permission
4.8 STREAM FUNCTION ( Ψ )

It is defined as scalar function of space &


time, such that its partial derivative w.r.t. to
Properties of Stream tube: any direction gives the velocity
perpendicular to that direction. It is defined
1) The stream-tube is bounded on all sides for two dimensional flows.
by streamlines. ∂ψ ∂ψ
U= , V= −
2) Fluid velocity does not exist across a ∂y ∂x
streamline; no fluid may enter or leave
a stream-tube except through its ends. Properties of stream function (Ψ ) :
3) The entire flow in a flow field may be
1) If stream function exists, it is possible
imagined to be composed of flows
case of continuous, incompressible
through stream-tubes arranged in some
steady flow.
arbitrary positions
2) If stream function (Ψ ) satisfies Laplace
4.7.2 PATH LINE Equation i.e.,
∂ 2ψ ∂ 2ψ
A path line means the path or line actually + =
0
∂y 2 ∂x 2
described by a single fluid particle as it
then, it is possible case of irrotational flow.
moves during a period of time. The path
line indicates direction of the velocity of the
same fluid particle at successive instant of
4.8.1 CONSTANT STREAM FUNCTION
time.
4.7.3 STREAK LINE Ψ = const
Let at any instant these particles arrive at
dΨ = 0
points Q, R and S.Q, R and S represent the ∂Ψ ∂Ψ
=
dΨ dx + dy
end points of the trajectories of these three ∂x ∂y
particles at the instant. The curve joining =
0 vdx − udy
the points S, R, Q and the fixed point P will
dy v
define the streak line at that instant. The =
fixed point P will also lie on the line, since dx u
at any instant; there will be always a
particle of some identity at that point. 4.9 VELOCITY POTENTIAL FUNCTION

In the steady flow, the velocity at each It is defined as a scalar function of space &
point in the flow field remains constant time, such that its negative derivative with
with time and, consequently, the respect to any direction gives the fluid
streamlines do not vary from one instant to velocity in that direction.
the next. This implies that a particle located φ = f ( x, y, z ) for steady flow.
on a given streamline will remain on the dϕ dϕ dϕ
same streamline. Furthermore, consecutive u=- , v= - , w= -
dx dy dz
particles passing through a fixed point in
space will be in the same streamline and
subsequently, will remain on this Properties of Potential Function:
streamline. Thus in a steady flow, path 1. If velocity potential ( φ ) exists, the flow
line, streak lines, and streamlines are should be irrotational.
identical lines in the flow field

© Copyright Reserved by Gateflix.in No part of this material should be copied or reproduced without permission
2. If ( φ ) satisfy the Laplace equation, the ii) v = 2 y 2 , w = 2 xyz
flow is continuous, incompressible, and Solution:
steady. The continuity equation for incompressible
∂ 2φ ∂ 2φ ∂ 2φ fluid is given by equation as
+ + = 0 δu δv δw
∂x 2 ∂y 2 ∂z2 + + =0
δx δy δz
4.10 EQUIPOTENTIAL LINE Case 1:
It is defined as line along which the velocity u = x 2 + y 2 + z 2   
potential is constant. δu
∴ = 2x
φ = const δx
d φ=0 v = xy 2 − yz 2 + xy
( )
φ = f x,y for steady, 2D flow ∴
δv
δy
= 2 xy – z 2 + x
dφ dφ
=
dφ dx + dy δu δv
dx dy Substituting the values of and in
δx δy
=
0 udx + vdy
continuity equation
dy u δw
=− 2x + 2xy – z 2 + x + =
0
dx v δz
Or
4.10.1 RELATION B/W STREAM δw
=
−3x – 2xy + z 2 or
FUNCTION & VELOCITY POTENTIAL δz
FUNCTION ( −3x – 2xy + z 2 ) δz
δw =
Integration of both sides gives,
∂ψ ∂ϕ
u= =−
∫ ( −3x – 2xy + z ) δz
∫δw =
2
∂x ∂y
∂ψ ∂ϕ z3
=v = w =
(−3xz – 2xyz + ) +C
∂y ∂x 3
Hence, Where constant of integration cannot be
function of z, but can be a function of x and
∂φ ∂Ψ
= − y that is f(x,y).
∂x ∂y  z 
3

∂ϕ ∂ψ w =  −3 xz – 2 xyz+  + f ( x, y )
=  3
∂y ∂x Case 2:
Stream line and equipotential line are δv
orthogonal to each other v = 2y 2 ∴ = 4y
δy
 dy   dy  δw
  .  = −1 w = 2xyz ∴ = 2 xy
 dx equipotential  dx stream line δz
δv δw
Example: Substituting the values of and in
The following cases represents the two δy δz
velocity components, determine the third continuity equation, we get
component of velocity such that they δu
+ 4y + 2xy = 0
satisfy the continuity equation: δx
i) u = x + y + z , v = xy − yz + xy or
2 2 2 2 2

© Copyright Reserved by Gateflix.in No part of this material should be copied or reproduced without permission
δu Substituting the values x =2, y = 1 and z = 3
=
−4y − 2xy in velocity field, we get
δx
2  2  2 
or V = x yi + y zj − (2xyz + yz )k
δu = ( −4y − 2xy ) δx = (22 × 1)i+ (12 × 3)j− (2 × 2 × 1 × 3 + 1 × 32 )k
Integrating we get,
= 4i + 3j
− 21k

x2
u= −4 xy − 2 y + f ( y, z ) And resultant velocity
2
= −4 xy − x y+ f(y, z)
2 = 42 + 32 + ( −21)2
= 16 + 9 + 441 = 466 = 21.587 units

Example: Acceleration (2, 1, 3)


A fluid flow field is given by The acceleration components
V = x yi + y 2zj
2 − (2xyz + yz2 )k

a x ,a y ,and a z
Prove that it is a case of possible steady
for steady flow are
incompressible fluid flow. Calculate the
velocity and acceleration at the point (2,1,3). ∂u ∂u ∂u
ax = u + v + w
Solution: ∂x ∂y ∂z
For the given fluid flow field ∂v ∂v ∂v
ay = u +v +w
∂u ∂x ∂y ∂z
u = x2y ∴ =2xy
∂x ∂w ∂w ∂w
∂v az = u +v +w
v = y 2z ∴ = 2yz ∂x ∂y ∂z
∂y
u = x2y
∂w
w = −2xyz − yz2 ∴ = −2xy − 2yz ∂u ∂u 2 ∂u
∂z = 2xy, = x= , 0
∂x ∂y ∂z
For a case of possible steady
incompressible fluid flow, the continuity v = y 2z
equation should be satisfied ∂v ∂v ∂v
= 0,= 2yz,= y 2
∂u ∂v ∂w ∂x ∂y ∂z
+ + = 0
∂x ∂y ∂z w = −2xyz − yz2 ,
∂u ∂v ∂w ∂w ∂w
Substituting the value of , and , =
−2xyz, =
−2xz − z2 ,
∂x ∂y ∂z
∂x ∂y
we get,
∂w
δu δv δw = −2xy − 2yz
+ + ∂z
δx δy δz
Substituting these values in acceleration
= 2 xy + 2 yz − 2 xy − 2 yz = 0 components, we get acceleration at (2, 1, 3)
Hence, the velocity field a= x2y(2xy) + y 2z(x2 ) − (2xyz + yz2 )(0)
(
V = x yiˆ + y zjˆ − 2xyz + yz2 kˆ
2 2
) x

= 2x3y 2 + x2y 2z
is a possible case of fluid flow = 2(23 )(12 ) + (2)2 (1)2 x 3
= 2 × 8 + 12
Velocity at (2, 1, 3)
= 16 + 12
= 28 units

© Copyright Reserved by Gateflix.in No part of this material should be copied or reproduced without permission
a y = x2y(0) + y 2z(2yz) − (2xyz + yz2 )(y 2 )
Example:
= 2y 3z2 − 2xy 3z − y 3z2 The stream function for a two-dimensional
= 18 − 12 − 9 flow is given by ψ =2xy . Calculate the
= −3units velocity at the point P ( 2, 3) . Also, find the
velocity potential function φ
a z = x 2 y(−2yz) + y 2 z(−2xz − z 2 )
Solution:
− (2xyz + yz 2 )(−2xy − 2yz) Given:
= −2x 2 y 2 z − 2xy 2 z 2 − y 2 z3 ψ =2xy
+[4x 2 y 2 z + 2xy 2 z 2 + 4xy 2 z 2 + 2y 2 z3 ] The velocity components u and v in terms
of ψ are
=−
( 2 × 22 × 12 × 3) − (2 × 2 × 12 × 32 ) − 12 × 33
δψ δ
+ {(4 × 22 × 12 × 3) + (2 × 2 × 12 × 32 ) u= − = − (2xy) = −2x
δy δy
+ (4 × 2 × 12 × 32 ) + (2 × 12 × 33 )} δψ δ
= −24 − 36 − 27 + (48 + 36 + 72 + 54) =
v = (2xy)= 2y
δx δx
At the point P ( 2, 3) . , we get
= 123
     
∴ Acceleration= a x i + a y j + az k = 28i − 3j + 123k u =−2 × 2 =−4units
Or v = 2× 3 = 6
magnitude of resultant acceleration ∴ Resultant velocity @P
282 + ( −3)2 + 123
= 2
784 + 9 + 15129
= 15922 = u2 + v 2 = 42 + 62 = 52 = 7.21units / sec
= 126.18units Velocity potential function φ
We know
Example: δφ
The velocity potential function is given by =−u =−( −2x) =2x ..... ( i )
δx
=
φ 5(x2 − y 2 ) . Calculate the velocity
δφ
component at the point ( 4, 5) . =−v =−2y ..... ( ii )
δy
Solution: Integrating equation ( i ) , we get
=
φ 5(x 2 − y 2 )
∫ dφ =∫ 2xdx
δφ
= 10x 2x 2
δx φ= + C= x2 + C ..... ( iii )
2
δφ Where C is a constant which is independent
= −10y
δy of x but can be a function of y
But the velocity components u and v are Differentiating equation (iii) w.r.t. ‘y’, we
given by equation as δφ δC
δφ get =
u = − = −10x δy δy
δx δφ
δφ But from ( ii ) , = −2y
v =− =−( −10y) =10y δy
δy
δC
The velocity components at the point (4, 5), ∴ = −2y
i.e., at=x 4,y= 5 are δy
u =−10 × 4 =−40units Integrating this equation, we get
v = 10 × 5 = 50units

© Copyright Reserved by Gateflix.in No part of this material should be copied or reproduced without permission
2y 2 δC
C= ∫ −2y dy = −
2
=
−y 2
δy
=y

Substituting this value of C in equation ( iii ) , Integrating the above equation, we get
y2
we get φ= x − y
2 2
=
C + C1
2
Where C1 is a constant of integration,
Example:
In a two dimensional incompressible flow, which is independent of x and y.
the fluid velocity components are given by y2
Taking it equal to zero, we get c =
u = x − 4y and v = − y − 4x . Show that 2
velocity potential exists and determine its Substituting the value of C in equation ( iii ) ,
form. Find also the stream function. we get
Solution:
Given: u =x − 4yandv =−y − 4x x2 y2
φ = − + 4xy +
2 2
∂u ∂v
∴ = 1 and = −1
∂x ∂y Value of Stream functions
∂u ∂v Let ψ =stream function
∴ + =1−1 = 0
∂x ∂y The velocity components in terms of
Hence flow is continuous and velocity stream function are
potential exists. δψ
=v =−y − 4x ..... ( iv )
Let φ =Velocity potential. δx
Let velocity components in terms of and
velocity potential is given by δψ
=−u =−(x − 4y) =−x + 4y ...... ( v )
δφ δy
=−u =−(x − 4y) =−x + 4y....(i)
δx Integrating equation ( iv ) w.r.t. x, we get
δφ
=−v =−(y − 4x) =y + 4x .......(ii) 4x2
δy ψ = −yx − +k ....... ( vi )
2
Integrating equation ( i ) , we get Where k is a constant of integration which
x2 is independent of x but can be function of y
φ=− + 4xy + C .....(iii) Differentiating equation ( vi ) w.r.t. y, we get
2
Where C is constant of integration, and is δψ δk
independent of x. This constant can be a =−x − 0 +
δy δy
function of y.
Differentiating the above equation, i.e., But from equation (v) , we have
equation (iii) w.r.t. ‘y’, we get δψ
δφ δC =−x + 4y
=0 + 4x + δy
δy δy
δψ
But from equation ( iii ) , we have Equating the two values of , we get
δy
δφ δk
= y + 4x = 4y
δy δy
δφ Integrating the above equation, we get
Equating the two values of , we get 4y 2
δy =
k = 2y 2
2

© Copyright Reserved by Gateflix.in No part of this material should be copied or reproduced without permission
Substituting the values of k in equation ( vi ) x3
Also v=xy 2 − 2y −
, we get 3
ψ = − yx − 2x 2 + 2y 2 δv
∴= 2xy − 2
δy
Example: i) For a two-dimensional flow, continuity
V 8x3 $
A fluid flow is given by= i − 10x 2 yj$ δu δv
equation is + =0
Find the shear strain rate and state δx δy
whether the flow is rotational or δu δv
irrotational. Substituting the value of and ,
δx δy
we get
Solution:
Given: δu δv
+ = 2 − 2xy + 2xy − 2 = 0
V 8x3i − 10x 2yj
=  δx δy
∴ It is a possible case of a fluid flow.
δu 2 δu
=u 8x =3
, =
24x , 0 ii) Rotation, ωz is given by
δx δy
δv δv 1  δv δu  1 2 2
v= −10x2y, = −20xy, = −10x2 =
ωz  − = [(y − x ) − (y 2 − x=
2
)] 0
δx δy 2  δx δy  2
( i ) Shear strain rate is given by equation as
1  δv δu  1
= +  =( −20xy + 0) = −10xy
2  δx δy  2
( ii ) Rotation in x-y plane is given by
equation or
1  δv δu  1
ωz =  −  = ( −20xy − 0) =−10xy
2  δx δy  2
As rotation ωz ≠ 0 . Hence flow is rotational.

Example:
The velocity components in a two-
dimensional flow are
y3 x3
u= + 2x − x 2y and v=xy 2 − 2y −
3 3
Show that these components represent a
possible case of an irrotational flow.

Solution:
y3
Given u = + 2x − x 2y
3
δu
= 2 − 2xy
δx
δu 3y 2
= − x2 = y 2 − x2
δy 3

© Copyright Reserved by Gateflix.in No part of this material should be copied or reproduced without permission
GATE QUESTIONS

Q.1 A fluid flow is represented by the −2xyt respectively, where tis time.
�= axı̂+ ayȷ̂, where a is
velocity field V The equation of stream line is
constant. The equation of stream a) x2y= constant
line passing through a point (1, 2) is b) xy2= constant
a) x − 2y = 0 b) 2x + y = 0 c) xy= constant
c) 2x − y = 0 d) x + 2y = 0 d) not possible to determine
[GATE–2004] [GATE–2006]
Q.5 In a two-dimensional velocity field
Q.2 The velocity components in the x
with velocities u and v along the x
and y directions of a two
and y directions respectively, the
dimensional potential flow are u and
convective acceleration along the x -
∂u
v , respectively. Then is equal to direction is given by
∂x ∂v ∂u ∂u ∂v
∂v ∂v a) u + v b) u + v
a) b) − ∂x ∂y ∂x ∂y
∂x ∂x ∂u ∂u ∂u ∂u
∂v ∂v c) u + v d) v + u
c) d) − ∂x ∂y ∂x ∂y
∂y ∂y
[GATE–2006]
[GATE–2005]
Q.6 In a steady flow through a nozzle,
Q.3 A leaf is caught in a whirlpool. At a the flow velocity on the nozzle axis
given instant, the leaf is at a distance is given by v = u0(1+ 3x/L)𝚤𝚤⃗, where
of 120 m from the centre of the x is the distance along the axis of
whirlpool. The whirlpool can be the nozzle from its inlet plane and L
described by the following velocity is the length of the nozzle. The time
 60 ×103  required for a fluid particle on the
distribution: Vr = −  m / s
 2πr 
axis to travel from the inlet to the
exit plane of the nozzle is
300 ×103
and Vθ = m / s where r (in a)
L
b)
L
ln4
2πr uo 3u o
metres) is the distance from the
centre of the whirlpool. What will be L L
c) d)
the distance of the leaf from the 4u o 2.5u o
centre when it has moved through [GATE–2007]
half a revolution ?
a) 48 m b) 64 m Q.7 Which combination of the following
c) 120 m d) 142 m statements about steady
[GATE–2005] incompressible forced vortex flow
is correct ?
Q.4 A two-dimensional flow field has P: Shear stress is zero at all points
velocities along the x and y in the flow.
directions given by u = x2t and v = Q: Vorticity is zero at all points in
the flow.

© Copyright Reserved by Gateflix.in No part of this material should be copied or reproduced without permission
R: Velocity is directly proportional 3V 2 R V2R
to the radius from the center of a) b)
4h 2 4h 2
the vortex.
V2R V2h
S: Total mechanical energy per c) d)
unit mass is constant in the 2h 2 2R 2
entire flow field. [GATE–2008]
a) P and Q b) R and S Q.11 You are asked to evaluate assorted
c) P and R d) P and S fluid flows for their suitability in a
[GATE–2007] given laboratory application. The
following three flow choices,
Q.8 For the continuity equation given expressed in terms of the two
dimensional velocity fields in the xy
by ∇. �V⃗ = 0 to be valid, whereV�⃗ is
-plane, are made available
the velocity vector, which one of the
P: u= 2y,v=-3x
following is a necessary condition?
Q: u= 3xy, v=0
a) steady flow
R: u=−2x, v=2y
b) irrotational flow
Which flow(s) should be
c) inviscid flow
recommended when the application
d) incompressible flow
requires the flow to be
[GATE–2008]
incompressible and irrotational ?
a) P and R b) Q
Common Data For Q.9and Q.10
c) Q and R d) R
The gap between a moving circular plate
[GATE–2009]
and a stationary surface is being
continuously reduced, as the circular plate Q.12 Velocity vector of a flow field is
comes down at a uniform speed V towards given as V �⃗ = 2xy𝐢𝐢⃗ − x2z�⃗. 𝐣𝐣 The
the stationary bottom surface, as shown in vorticity vector at (1, 1, 1) is
the figure. In the process, the fluid
a) 4ı⃗ − ⃗ȷ b) 4ı⃗ − �⃗
k
contained between the two plates flows
c) ⃗ı − 4ȷ�⃗ �⃗
d) ⃗ı − 4k
out racially. The fluid is assumed to be
incompressible and inviscid. [GATE–2010]
Q.13 A streamline and an equipotential
line in a flow field
a) are parallel to each other
b) are perpendicular to each other
c) intersect at an acute angle
d) are identical
[GATE–2011]
Q.9 The radial velocity Vr at any radius
r, when the gap width is h, is
Vr Vr Q.14 For an incompressible flow field, V,
a) Vr = b) Vr = which one of the following
2h h
2Vh Vh conditions must be satisfied?
r
c) Vr = d) Vr = a) ∇.V = 0
r r r
[GATE–2008] b) ∇ × V = 0
r r
c) (V.∇)V = 0
r
Q.10 The radial component of the fluid ∂V r r
acceleration at r = R is d) + (V.∇)V =0
∂t
[GATE–2014 (2)]

© Copyright Reserved by Gateflix.in No part of this material should be copied or reproduced without permission
Q.15 A flow field which has only
convective acceleration is
a) a steady uniform flow
b) an unsteady uniform flow
c) a steady non-uniform flow
d) an unsteady non-uniform flow
[GATE–2014 (4)]
Q.16 Consider the following statements a) P – IV, Q – I, R – II, S – III
regarding streamline(s): b) P – IV, Q – III, R – I, S – II
i) It is a continuous line such that c) P – III, Q – I, R – IV, S – II
the tangent at any point on it d) P – III, Q – I, R – II,S-IV
shows the velocity vector at that [GATE–2015 (1)]
point
ii) There is no flow across Q.20 The velocity field of an
streamlines incompressible flow is given by
dx dy dz V = (a1x + a2y + a3z)ı̂ + (b1x + b2y +
iii) = = is the differential b3z)ȷ̂ + (c1x + c2y + c 3z)k� , where a1=
u v w
2 and c3 = − 4. The value of b2 is
equation of a streamline, where
______.
u, v and w are velocities in
[GATE–2015 (1)]
directions x, y and z, respectively
iv) In an unsteady flow, the path of a
Q.21 The volumetric flow rate (per unit
particle is a streamline
depth) between two streamlines
Which one of the following
having stream functions Ψ1 and Ψ2
combinations of the statements
is
is true?
a) |Ψ1 + Ψ2 | b)Ψ1 Ψ2
a) (i), (ii), (iv) b) (ii), (iii), (iv)
c) Ψ1 /Ψ2 d) |Ψ1 − Ψ2 |
c) (i), (iii), (iv) d) (i), (ii), (iii)
[GATE–2016 (2)]
[GATE–2014 (4)]
�⃗ = K(yı̂ +
Q.17 Consider a velocity field V Q.22 For a certain two-dimensional

xk). where K is a constant. The incompressible flow, velocity field is
vorticity, Ωz ,is given by 2xy �ı − y 2 ȷ̂ . The
a) − K b) K streamlines for this flow are given
c) − K/2 d) K/2 by the family of curves
[GATE–2014 (4)] a) 𝑥𝑥2𝑦𝑦2 = constant
b) 𝑥𝑥𝑥𝑥2 = constant
Q.18 If the fluid velocity for a potential c) 2𝑥𝑥𝑥𝑥 − 𝑦𝑦2 = constant
flow is given by V(x, y) = u(x, y)ı�+ d) 𝑥𝑥𝑥𝑥 = constant
v(x, y)ȷ̂ with usual notation then the [GATE–2016 (3)]
slope of the potential line at (x, y) is
a) v/u b) —u/v Q.23 For a two-dimensional flow, the
c) v2/u2 d) u/v velocity field is
[GATE–2015 (2)] r x $ y $
= u i+ 2 j , where 𝚤𝚤̂ and
Q.19 Match the following pairs: x 2 + y2 x + y2
𝚥𝚥̂ are the basis vectors in the x-y
Cartesian coordinate system.
Identify the CORRECT statements
from below.

© Copyright Reserved by Gateflix.in No part of this material should be copied or reproduced without permission
1) The flow is incompressible. drainage of the fluid between the plate
2) The flow is unsteady. and the wall as shown in the figure.
3) y-component of acceleration, Assume two-dimensional
−y incompressible flow and that the plate
ay =
( x 2 + y2 ) remains parallel to the wall.
2

The average velocity, uavg of the fluid


4) x-component of acceleration, (in m/s) draining out at the instant
−(x + y)
ax = shown in the figure is
( x 2 + y2 )
2
_____________(correct to three
decimal places).
a) (2) and (3) b) (1) and (3)
c) (1) and (2) d) (3) and (4)
[GATE–2016, Set (3)]

Q.24 Consider the two dimensional velocity


field given by

V = ( 5+a1 x+b1 y ) ˆi+ ( 4+a 2 x+b 2 y ) ˆj, where a1 , b1 , a 2 , b 2 [GATE–2018, Set (1)]
are constants. Which one of the following
conditions needs to be satisfied for the flow to Q.28 In a Lagrangian system, the position of
be incompressible? a fluid particle in a flow is described
a) a1 +b1 = 0 b) a1 +b 2 = 0 as x = x0 e − kt and y = y0 e kt where t is
the time while 𝑥𝑥𝑜𝑜, 𝑦𝑦𝑜𝑜, and k are
c) a 2 +b 2 = 0 d) a 2 +b1 = 0
constants. The flow is
[GATE–2017, Set (1)] a) unsteady and one-dimensional
Q.25 For a steady flow, the velocity field is b) steady and two-dimensional

V = ( − x 2 +3y ) ˆi+ ( 2xy ) ˆj The magnitude of the c) steady and one-dimensional
d) unsteady and two-dimensional
acceleration of a particle at (1, -1) is
[GATE–2018, Set (1)]
a) 2 b) 1
c) 2 5 d) 0 Q.29A sprinkler shown in the figure rotates
[GATE–2017, Set (1)] about its hinge point in a horizontal
plane due to water flow discharged
Q.26For a two-dimensional incompressible through its two exit nozzles.
( )
r
=
flow field given by u A x$
i − x$j
, where A > 0 which one of the following
statements is FALSE?
a) It satisfies continuity equation. The total flow rate Q through the
b) It is unidirectional when x → 0 and y → ∞ sprinkler is 1 litre/sec and the cross-
sectional area of each exit nozzle is 1
c) Its streamlines are given by x = y .
cm2. Assuming equal flow rate through
d) It is irrotational both arms and a frictionless
hinge, the steady state angular speed
[GATE–2018, Set (1)] of rotation (in rad/s) of the sprinkler is
Q.27 A flat plate of width L = 1 m is pushed ______ (correct to two decimal
down with a velocity U = 0.01 m/s places).
towards a wall resulting in the [GATE–2018, Set (1)]

© Copyright Reserved by Gateflix.in No part of this material should be copied or reproduced without permission
ANSWER KEY:

1 2 3 4 5 6 7 8 9 10 11 12 13 14
(c) (d) (b) (a) (c) (b) (b) (d) (a) (a) (d) (d) (b) (a)
15 16 17 18 19 20 21 22 23 24 25 26 27 28
(c) (d) (a) (b) (c) 2 (d) (b) (b) (b) (c) (b) 0.05 b
29
(c)

© Copyright Reserved by Gateflix.in No part of this material should be copied or reproduced without permission
EXPLANATIONS

Q.1 (c) Vr 1
�= axı̂+ ayȷ̂ ....(i) ⇒ = −
Given V Vθ 5
The equation of stream line is. V
dx dy dy ⇒ Vr = − θ
= = ....(ii) 5
ux uy uz dr
⇒ Vr =
From equation (i) Ux = ax ,Uy = ay dt
and Uz=0 dθ  dθ 
Substitute there values in equation ⇒ V= θ r Q V= θ = r 

dt  dt 
(ii), we get
dr r dθ
dx dy
= ⇒
dy dy
= ⇒ = −
ax ay x y dt 5 dt
dr dθ
Integrating both sides, we get ⇒ = −
dx dy r 5
∫ x =∫ x Integrating both sides, we get
r π
dr 1
log x = log y + log c = log yc
⇒x = yc.... (iii)
∫ r 5 ∫0dθ
120
= −

1 π
At point (1, 2) 1 = 2c ⇒ C = ⇒ ln r 120 = −
r

2 5
From equation (iii) r π
y ⇒ ln = −
x = ⇒ 2x − y = 0 120 5
2 r
⇒ = e − π/5
Q.2 (d) 120
For two dimensional flow, continuity ∴ r=64m
equation should be satisfied.
∂u ∂v Q.4 (a)
i.e. + =0 Equation of streamline,
∂x ∂y
dx dy
∂u ∂v =
∴ = − u v
∂x ∂y dx dy
=
xy −2xy
Q.3 (b)
dx dy
Vr  300 ×103  2πr =
=
− × x −2y
 2πr  (60 ×10 )
3

dx dy
2 + = 0
x y
2 ln x + ln y = ln c
2
ln yx = ln c
x2 y = c

Q.5 (c)

© Copyright Reserved by Gateflix.in No part of this material should be copied or reproduced without permission
Convective Acceleration is defined ω2 2 2
as the rate of change of velocity due ρ
2
( r2 − r1 ) −ρg ( Z2 − Z1 ) = 0
to the change of position of fluid Δ K.E − ΔP.E = 0
particles in a fluid flow. Δ K.E = ΔP.E
In Cartesian coordinates, the So, total mechanical energy per unit
components of the acceleration mass is constant.
vector along the x direction is given
by. Q.8 (d)
∂u ∂u ∂u ∂u The continuity equation is given by
ax = u + v + w +
∂x ∂y ∂z ∂t r ∂ρ
In above equation term ∂u/∂t is ( )
∇. ρV +
∂t
=0
known as local acceleration and Now if ρ remains constant then only
terms other than this, called we can write the above equation as
convective acceleration. �⃗ = 0
∇. V
Hence for given flow. Convective So, the above equation represents
acceleration along x-direction. the incompressible flow
∂u ∂u ∂u
=
a x u + v (neglecting w
∂x ∂y ∂z Q.9 (a)
because it is a two dimensional Let us consider for small time dt,
flow) then for any cylindrical control
volume between 2 plates of radius
Q.6 (b) ‘r’ volume reduced by the movement
Given of upper plate in time ‘dt’ is equal to
dx  3x  u the volume of fluid moving out
= u 0 1 +  = x ( L + 3x )
dt  L L ∴ ( V × dt ) × πr 2 = (Vr × dt) × 2πrh
L 1 V.r
=
dt × dx ∴ Vr =
u 0 ( L + 3X ) 2h
On integration both the sides within
limits ⇒ 0 to t and x ⇒0 to L we get Q.10 (a)
L L Radial acceleration
L 1
∫0 u 0 ∫0 ( L + 3X )
dt = dx
=
∂V ∂V
a r Vr . r + r
∂r ∂t
L ∂
t= [ln(1 + 3x / L)]0
L V.r  V.r  ∂  V.r 
3u o = ar .  +  
2h ∂r  2h  ∂t  2h 
L For convective part r is variable and
t= ln4
3u o for local acceleration h is variable
V.r V V V.r  −1 ∂h 
= ∴ ar . . + . 
Q.7 (b) 2h 2h 2h 2  h 2 ∂t 
For forced Vortex flow the relation ∂h
is given by, V = rω ...(i) Also, − = V
∂t
From equation (i) it is shown easily
that velocity is directly proportional V 2 .r V.r  V 
∴= ar + . 
to the radius from the centre of the 4h 2 2  h2 
vortex. 3V 2 .r
(Radius of fluid particle from the a r =
4h 2
axis of rotation) At r=R
And also for forced vortex flow,

© Copyright Reserved by Gateflix.in No part of this material should be copied or reproduced without permission
3V 2 .R Q.12 (d)
ar =
4h 2 GivenV = 2xyı⃗ - x2z�⃗ȷ
 i j k
Q.11 (d)  
∂ ∂ ∂
Incompressible flow satisfy Vorticity = 
 dx dy dz 
continuity equation  
∂u ∂v u v w
+ = 0
∂x ∂y Substitute u = 2xy, v = − x 2 z, w = 0
P. u=2y, v= −3x  i j k
∂ (2y) ∂ (−3x)  
+ ∂ ∂ ∂
∂x ∂y So Vorticity= 
 ∂x ∂y ∂z 
Incompressible flow  
 2xy − x z
2
0
⇒ 0+0=0
Q. u=3xy, v=0 =x 2⃗ı − 0 + �⃗
k(−2xz−2x)
∂ (3xy) ∂ (0) Vorticity vector at P(1,1,1)
+
∂x ∂y = ⃗ı + �⃗
k [- 2 – 2] = ⃗ı ̶ 4 �⃗
k
Compressible flow
⇒ 3y+0 ≠0 Q.13 (b)
R. u=−2x,v= 2y
∂ (−2x) ∂ (2y) Q.14 (a)
+ For an incompressible flow field,
∂x ∂y r
divergence must be zero i.e. ∇.V =
0
Incompressible flow
−2+2=0
Q.15 (c)
For irrotational flow condition is
Total
r acceleration,
1  ∂v ∂u  r
wz =  − = 0 DV ∂V r r
2  ∂x ∂y  = + (V.∇)V
Dt ∂t
For P : u=2y, v=−3x r r
(V.∇)V → Convective acceleration
1  ∂ (−3x) ∂ (2y)  r
= − ∂V
2  ∂x ∂y 
wz
→ Local acceleration
∂t
1
= [−3 − 2] ≠ 0 In steady non-uniform flow,
2 conditions change from point to
(Rotational flow) point in the stream but do not
For Q: u=3xy,v=0 change with time.
1  ∂ (0) ∂ (3xy) 
= −
2  ∂x ∂y 
wz
Q.16 (d)
1
= [0 − 3x] ≠ 0 Q.17 (a)
2
Vorticity about z-axis:
(Rotational flow)
∂v ∂u
For R: u = −2x, v=2y Ω= −
∂x ∂y
z
1  ∂ (2y) ∂ (−2x) 
= − ∂v ∂u
2  ∂x ∂y 
wz
= 0,= K
∂x ∂y
1
= [0 − 0]= 0 ∴ Ω z =−0 K= −K
2
(irrotational flow)
Q.18 (b)

© Copyright Reserved by Gateflix.in No part of this material should be copied or reproduced without permission
Here V(x, y) = u(x, y)ı̂+ v(x, y)ȷ̂ y2 − x 2 x 2 − y2
−∂φ = +
As we know that u =
− ∂ϕ
& v= x 2 + y 2 ( x 2 + y 2 )2
∂x ∂y
∂∅ ∂∅ ∂u ∂v
=
d∅ dx + = dy 0 ∴ + = 0
∂x ∂y ∂x ∂y
∴−udx − vdy = 0 ∴ (1) statement is correct
r
∂u
dy u Also, =0
= − ∂t
dx v
∴ Flow is steady
So the correct answer is (b)
Q.19 (c)
Q.24 (b)
Q.20 (1.9 to 2.1)
∂u ∂v ∂w
+ + =0 For flow to be incompressible
∂x ∂y ∂z ∂u ∂v
a1 + b 2 + c3 = 0 + = 0...........(1)
∂x ∂y
2 − 4 + b2 = 0 Here u=5 + a1x +b1 y
b2 = 2 & v=4 + a2 x +b2 y
∂u ∂v
=
∴ a1 = & b2
Q.21 (d) ∂x ∂y
put value in equation (1)
Q.22 (b) a1 + b 2 = 0
Equation of the streamline is given
by
dx dy
=
u v Q.25 (c)
dx dy
= 2
2xy − y Given u = − x 2 + 3y
dx 2dy ∂u
= = −2x
x −y ∂x
⇒ lnx = −2lny + lnc ∂u
=3
ln x + lny 2 =
ln c ∂y
2
∴ xy = c v = 2xy
∂v
= 2y
Q.23 (b) ∂x
For the flow to be incompressible ∂v
∂u ∂v = 2x
+ = 0 [Continuity equation] ∂y
∂x ∂y ∂u ∂u
=
ax u +v
∂u ∂v ∂  x  ∂  y  ∂x ∂y
∴= + +
∂x ∂y ∂x  x 2 + y 2  ∂y  x 2 + y 2  a x =−( x + 3y)(−2x) + 2xy(3)
2

=
(x 2
+ y 2 ) − x(2x)
+
(x 2
+ y 2 ) − y(2y) a x = (−1 + 3)(−2) + 2(3)
(x + y2 ) (x + y2 ) ax = 8 − 6 = 2
2 2 2 2

∂v ∂v
=
ay u + v
∂x ∂y

© Copyright Reserved by Gateflix.in No part of this material should be copied or reproduced without permission
=−( x 2 + 3y)(2y) + 2xy(2x) 1. Here flow is two dimensional because
= (−1 − 3)(−2) − 2(2) dx dy
≠ ≠0
=+8 − 4 =4 dt dt
at = a 2x + a 2y = 22 + 42 dx
= u= x0  ( −k ) e − kt
dt
at = 2 5
− x0 ke − kt =
= −kx
Q.26(c) dy
= v= = ky0 e kt ky
C is the false statement dt
2D incompressible flow continuity equation. As, both u and v are not depends on time so,
∂u ∂v it is steady flow
+ = 0 Q.29(10)
∂x ∂y
∂ ( Ax ) ∂ ( − Ay )
+ = 0 litre/sec, A = 1 cm 2
∂x ∂y
A − A = 0 it satisfies continuity equation. ⇒ First find the velocity of flow
ur Q
⇒ As V = Axi$− Ayi$ = A1 V1
2
A →∞ 110−3
dx dy

2
= (1    )  V1
=
u v ⇒ V1 = 5m/s
dx dy
= If the angular velocity of the sprinkler is ω
Ax − Ay then the absolute velocities of flow through
ln x = − ln y + ln c the nozzle are
ln xy = ln c
xy= c → streamline equation
Q.27
Volume swept by the plate per unit time
= Discharge going out or drainage from two V1ab =5 + 0.1 ω
sides V2ab =5 − 0.2 ω
∴ U  (L W ) = 2  uavg  d  W
Since the moment of momentum of flow
entering is zero and there is no friction the
(Assuming width of the plate (W) to be very momentum leaving the sprinkler must also be
long, the discharge perpendicular to zero.
the plane can be neglected.)
UL 0.01  1
u= = = 0.050 m/s ( ρQ) 
avg
2  0.1 ⇒ ( 5 + 0.1ω) 0.1 − ( 5 − 0.2ω) 0.2 =0
2 
2d

(1) 0.5 + 0.01ω − 1 + 0.04ω = 0


⇒ [ ]
2
Q.28(b)
⇒ − 0.5 + 0.05ω = 0
ω =10 rad / sec
x = x0 e − kt
y = y0 e kt

© Copyright Reserved by Gateflix.in No part of this material should be copied or reproduced without permission
5 BERNOULLI’S EQUATION & ITS APPLICATIONS

5.1 INTRODUCTION
dP
Consider a small element of fluid in flow ∫ ρ ∫
+ VdV + ∫ gdz =
∫0
field. The energy in the element as it moves
in the flow field is conserved. This principle P V2
+ + zg =const
of conservation of energy is used in the ρ 2
determination of flow parameters like Multiplying the Bernoulli’s equation by the
pressure, velocity and potential energy at density (ρ). Each term in this equation has
various locations in a flow. The concept is units of pressure.
used in the analysis of flow of ideal as well
ρV 2
as real fluids. Energy can neither be created P+ + ρzg =const
nor destroyed. It is possible that one form 2
of energy is converted to another form.
• P is the static pressure (it does not
5.2 EULER’S EQUATION incorporate any dynamic effects); it
represents the actual thermodynamic
Euler’s equation is obtained from the pressure of the fluid. This is the same as
conservation of momentum for a fluid the pressure used in thermodynamics
particle moving along a streamline. The and property tables.
forces due to gravity & pressure are taken P
into consideration. is known as pressure head
ρg
dP
+ VdV + gdz = 0
ρ
ρV 2
This equation is known as Euler’s equation • is the dynamic pressure; it
of motion. The assumptions involved are: 2
1) Steady flow represents the pressure rise when the
2) Motion along a stream line fluid in motion is brought to stop
3) Ideal fluid (frictionless) isentropically.
In the case of incompressible flow, this ρV 2
is known as dynamic head
equation can be integrated to obtain 2
Bernoulli Equation.
• ρgz is the hydrostatic pressure,
5.3 BERNOULLI’S EQUATION which is not pressure in a real sense
since its value depends on the reference
The Bernoulli’s equation states that the level selected. It accounts for the
sum of the flow, kinetic, and potential elevation effects, i.e., of fluid weight on
energies of a fluid particle along a pressure.
streamline is constant. Therefore, the Z is gradient or datum head.
kinetic and potential energies of the fluid The sum of the static, dynamic, and
can be converted to flow energy (and vice hydrostatic pressures is called the total
versa) during flow. pressure. Therefore, the Bernoulli’s
By integrating Euler’s equation for equation states that the total pressure
incompressible flow along a streamline is constant. The sum

© Copyright Reserved by Gateflix.in No part of this material should be copied or reproduced without permission
of the static and dynamic pressures is 2gh
called the stagnation pressure, and it Q=
theo
a=v
2 2
a2
2
is expressed as a 
Stagnation pressure = static pressure + 1− 2 
dynamic pressure.  a1 
ρv 2 In case of ideal fluid, the above equation is
Stagnation pressure= P + valid. To modify the equation for real fluid,
2
coefficient of discharge (Cd) is multiplied to
5.4 APLLICATION OF BERNOULLI’S theoretical flow. It accounts for viscous
EQUATION loss, expansion loss & boundary roughness.
It is defined as
Venturimeter and Orifice meters are the
actual discharge
obstruction type meters commonly used cd =
for the measurement of flow through pipes. theoretical discharge
In each case the meter acts as an obstacle 2gh
placed in the path of the flowing fluid =
Q act c=a v cd a2
d 2 2 2
causing local changes in pressure and a 
1− 2 
velocity
 a1 
i) Venturimeter
ii) Orifice plate PA − PB  ρm 
=h =  − 1 x
iii) Pitot tube ρg  ρ 
5.4.1 VENTURIMETER Where,
x = the difference in mercury level
A venturimeter consist of a short ρm = Density of heavy liquid
converging part, throat & diverging part. ρ = Density of flowing fluid
The liquid undergoes gradual contraction & Case1:
expansion, therefore it has lesser losses. Manometric fluid is lighter then liquid
By applying Bernoulli’s equations at 1 & 2
flowing in pipe (in case of inverted
P1 V12 P2 V22 manometer)
+ + z1 = + + z2
ρg 2g ρg 2g  ρ 
=h x1 − m 
Points 1 & 2 are at same horizontal level, ρ 

∴ z1 = z 2 Case2:
P P  V 2 − V12 Inclined/vertical venturimeter with
h= 1 − 2 + 2 differential U-tube Manometer
 ρg ρg  2g
 P1  P   ρm 
=
Q a= v a2v 2
1 1
h =+ z1  −  2 + z 2  =
 − 1 x
 ρg   ρg   ρ 
5.4.2 ORIFICE METER OR ORIFICE PLATE
It is a device used for measuring the rate of
flow of a fluid through a pipe. It is a cheaper
device as compared to venturimeter. The
orifice diameter is kept generally 0.35
2gh
v2 = times the diameter of pipe. Due to sudden
2
a  expansion & contraction, loss is high in
1− 2  orificemeter.
 a1 

© Copyright Reserved by Gateflix.in No part of this material should be copied or reproduced without permission
Due to the viscous effects, the actual flow
velocity through the orifice will always be
less than the theoretical possible velocity.
The velocity coefficient Cv is defined as
follows.
Actual of jet at venna contacta
Cv =
A differential Manometer is connected to Therotcial veocity of jet at orifice
measure the pressure difference at section The value of coefficient of contraction
(1), which is at distance of about 1.5-2.0 varies from 0.61 to 0.69 depending on the
times the pipe diameter and section (2), shape and size of the orifice.
which is at a distance of about half
diameter of the orifice on the downstream 2gh
Vact = c v
side. 2
a 
2

By applying Bernoulli’s equations at 1 & 2 1− 0 


P1 V12 P2 V22  a1 
+ + z1 = + + z2 2gh
ρg 2g ρg 2g =
Q act a=v c c a 0c v
2 act 2 2
a 
Points 1 & 2 are at same horizontal level, 1− 0 
∴ z1 = z2  a1 
 P1 P2  V22 − V12 2gh
h= − + Q act = cda0
 ρg ρg 
2
2g a 
1− 0 
=
Q a=v a2v 2
1 1  a1 
As the liquid comes out of orifice it Where,
contracts further and the area just outside Cd = Cc C v
the orifice is lower compared to the area of
Average value of Cd for orifices is 0.62.
the orifice. This section is called as vena-
contracta. Area of jet at the vena-contracta
is less than the area of the orifice itself due 5.4.3 PITOT TUBE
to convergence of stream lines. The
coefficient of contraction Cc is defined as It is a device used for measuring flow
velocity at any point in a pipe.
follows.
area of jet at vena contracta(a2 ) Principle: If velocity of flow at any point
cc =
area of orifice(a o ) becomes zero, the pressure is increased
The value of coefficient of contraction due to conversion of kinetic energy to
varies from 0.61 to 0.69 depending on the pressure energy.
shape and size of the orifice
a2 = cc .a0
=
Q a=v a=
1 1
v cc .a0 v 2
2 2

2gh
v2 =
2
a 
1 −  2  cc2 By applying Bernoulli’s equations at 1 & 2
 a1  Stagnation pressure = static pressure +
Dynamic pressure

© Copyright Reserved by Gateflix.in No part of this material should be copied or reproduced without permission
P1 V12 P2 =R Ucos θgt
+ = 2Ucos θgUsin θ
ρg 2g ρg =
g
V12 P2 P1
= − =h U2 sin2θ
2g ρg ρg =
g
Vact = c v 2gh
P2 − P1  ρm 
=h =  − 1 x 4. Value of 𝜃𝜃 for max range
ρg  ρ 
Where, U2 sin2θ
R=
Cv is velocity coefficient g
sin2θ = 1
5.5 BERNOULIS EQUATION FOR REAL 2θ =90o Or θ =45o
FLUIDS
Example:
2 2
P1 V P V Water is flowing through a pipe of 5cm
+ + z1 = 2 +
1
+ z2 + h f 2
diameter under a pressure of
ρg 2g ρg 2g 2
Where, 29.43N / cm (gauge) and with mean
hf is head loss due to viscous force & velocity of 2m / s . Find the total head or
minor losses. total energy per unit weight of the water at
a cross–section, which is 5m above the
5.6 FREE LIQUID JETS datum line.

Free liquid jet is defined as the jet of water Solution:


coming out from the nozzle in atmosphere. Given:
The path travelled by the free jet is Diameter of pipe= 5 cm = 0.05m
parabolic, given by equation of parabola Pressure,
1 =P 29.43N /= cm2 29.43 × 104 N / m2
=y x tan θ − gx 2 sec2 θ
2 Velocity, V = 2.0m / s
1. Max height Datum head, Z = 5m
( Usin θ) − (0) = Total head
2 2
2gy max
=Pressure head+kinetic head+datum head
U2 sin2 θ P 29.43 × 104
y max = Pressure =
head = = 30m
2g ρg 1000 × 9.81
V2 2× 2
2. Time of flight Kinetic head
= = = 0.204m
2g 2 × 9.81
1
=
y Usin θgt − gt 2 P V2
2 ∴ Total head = + + z = 30 + 0.204 + 5
For full flight, y = 0 ρg 2g
1 =35.204m
Usin θgt − gt 2 = 0
2
Example:
2Usin θ An oil of S.G. 0.8 is flowing through a
t=
g venturimeter having inlet diameter 20 cm
3. Max Distance (Range) and throat diameter 10cm. The oil–
mercury differential manometer shows a

© Copyright Reserved by Gateflix.in No part of this material should be copied or reproduced without permission
reading of 25 cm. Calculate the discharge of Dia at throat, d2 =15cm
oil through the horizontal venturimeter. π
( )
2
Take Cd = 0.98 ∴
= a2 =
15 176.7cm2
4
Solution: S   13.6 
Given: h= x  h − 1=
 20  − 1
S
 o   1 
S.G. Of oil, S0 = 0.8
= 20 × 12.6 =252cm of water
S.G. Of mercury, Sh =13.6
Cd = 0.98
Reading of differential manometer,
x =25cm a 1a 2
Discharge,
= Q cd × 2gh
∴ Difference of pressure head, a12 −a22
S   13.6  706.85×176.7
h= x  h − 1=
 25 − 1  cm of oil = 0.98× × 2×981×252
 So  0.8  (706.85) − (176.7)
2 2

=25(17-1) =400 cm of oil. 86067593.36 86067593.36
Dia at inlet, d1 = 20cm = =
499636.3-31222.9 684.4
π π = 125756cm3 / s = 125.756lit/ s
∴ a1 = × d12 = × 202 =314.16cm2
4 4
d2 =10cm Example:
In a vertical pipe conveying oil of specific
π
∴ a2 = × 102 = 78.54cm2 gravity 0.8, two pressure gauges have been
4 installed at A and B where the diameters
Cd = 0.98 are 16cm and 8cm respectively. A is 2
∴ The discharge Q is given by equation meters above B. The pressure gauge
a 1a 2 readings have shown that the pressure at B
=Or Q cd × 2gh is greater than at A by 0.981N/ cm2 .
a1 −a2
2 2
Neglecting all losses, calculate the flow rate.
314.16 × 78.54 If the gauges at A and B are replaced by
0.98 × 2 × 981 × 400 tubes filled with the same liquid and
(314.16)2 − (78.54)2 connected to a U-tube containing mercury,
21421375.7 calculate the difference of level of mercury
= 70422.4cm3 / s
98696 − 6168 in the two limbs of the U-tube.
= 70.422litres / sec
Solution:
Given:
Example:
S.G. of oil, S0 = 0.8
A 30cm ×15cm venturimeter is inserted in
vertical pipe carrying water, flowing in the kg
∴ Density, ρ = 0.8×1000 = 800 3
upward direction. A differential mercury m
manometer connected to the inlet and Dia. at =
A, da 16cm
= 0.16m
throat gives a reading of 20cm. Find the
∴ Area at A,
discharge. Take Cd = 0.98
π
( )
2
Solution: = a1 = 0.16 0.0201m2
4
Given:
Dia at B,= = 0.08m
db 8cm
Dia at inlet, d1 = 30cm
∴ Area at B,
π
( )
2
∴=a1 = 706.85cm
30 2
π
( )
2
4 = a2 = 0.08 0.005026m2
4

© Copyright Reserved by Gateflix.in No part of this material should be copied or reproduced without permission
∴ Rate of flow, Q= V × A
=
0.99 × 0.0201 =
0.01989m3 / s

ii) Difference of level of mercury in the U-


tube.
Let h=Difference of mercury level.
Where
S 
Then= h x  h − 1 
 So 
Where
PA PB
i) Difference of pressure, h= − +z −z
ρg ρg A B
pB − pA =0.981N / cm2
 P −P 
=
0.981 × 104 N / m2 =
9810N / m2 = −1.25+2.0 − 0 Q B A =1.25 
Difference of pressure head  ρg 
pB − pA
∴= 9810 = 0.75
= 1.25
ρg 800 × 9.81  
∴ 0.75 =x  13.6 − 1  =x × 16
Applying Bernoulli’s equation at A and  0.8 
B and taking the reference as line
0.75
passing through section B, we get, ∴=
x = 0.04687m
16
PA VA 2 PB VB2
+ + zA = + + zB
ρg 2g ρg 2g = 4.687cm
2 2
PA PB V V
Or − + z A − zB = B − A Example:
ρg ρg 2g 2g A pitot- static tube placed in the centre of a
 P −P  V2 V2 300mm pipe line has one orifice pointing
Or  A B  + 2.0 − 0.0 = B − A upstream and other perpendicular to it.
 ρg  2g 2g
The mean velocity in the pipe is 0.80 of the
Or central velocity. Find the discharge through
V2 V2  P −P  the pipe if pressure difference between the
−1.25 + 2.0 = B − A Q A B = 1.25
2g 2g 
 ρg  two orifices is 60 mm of water .Take the
coefficient of pitot tube as Cv = 0.98
=
0.75
VB2 VA 2

2g 2g
----- i () Solution:
Given:
Now applying continuity equation at A
Dia of pipe, d=300mm=0.30m
and B, we get
Diff of pressure head,
VA × A1 = VB × A 2
h = 60mm of water = 0.06m of water
V ×A Cv = 0.98
Or VB = A 1
A2
=
Mean velocity, V 0.80 × centre line velocity
Substituting the value of VB in equation
Centre line velocity is given by equation
(i), we get
16VA 2 VA 2 15VA 2 = Cv 2gh = 0.98 × 2 × 9.81 × 0.06 = 1.063m / s
0.7=
5 − =
2g 2g 2g ∴ V= 0.80 × 1.063= 0.8504m / s
0.75 × 2 × 9.81
=∴ VA = 0.99m / s
15

© Copyright Reserved by Gateflix.in No part of this material should be copied or reproduced without permission
=
Discharge, Q area of pipe × V 22.263 N/cm2, while the datum head at A
and B are 28m and 30m. Find the loss of
π
( 0.30) × 0.8504= 0.06m3 / s
2
Q= head b/w A & B.
4 Solution:
D = 400 m = 0.4 m
Example: VA= V= 25m / s
A pitot–tube is inserted in a pipe of 300mm
Total energy at A
diameter. The static pressure in pipe is
100mm of mercury (vacuum). The PA VA 2
+ + zA
stagnation pressure at the centre of the ρg 2g
pipe, recorded by the pitot–tube is Total energy at B
0.981N/ cm2 . Calculate the rate of flow of PB VB2
water through pipe, if the mean velocity of = + + zB
flow is 0.85 times the central velocity. Take ρg 2g
Cv = 0.98. H=
2 EA − EB
Solution:  PA VA 2   PB VB2 
Given:  + + zA  −  + + zB 
  ρg 2g 
Dia of pipe d=300mm = 0.30m  ρg 2g   
2
π PA 29.43 × 10 VA 4

(0.3) 0.07068m2
2
∴ Area,
= A = = = 30, = 31.85
4 ρg 1000 × 9.81 2g
Static pressure head = 100mm of mercury PB 22.563 × 104 VB2
(vacuum) = = 23, = 31.85
100 ρg 1000 × 9.81 2g
=− × 13.6 = −1.36m of water EA − EA = (30 + 28) − (23 + 30) = 5m
1000
Stagnation pressure
= .981N / cm = 2
981 × 104 N / m2 Example:
A jet of water from a 25 mm diameter
∴ Stagnation pressure head
nozzle is directed vertically upwards.
.981 × 103 0.981 × 103 Assuming that the jet remains circular and
= = = 1m
ρg 1000 × 9.81 neglecting any loss of energy, what will be
h = Stagnation pressure head –static the diameter of the jet at a point 4.5 m
pressure head above the nozzle, if the velocity with which
= 1.0 − ( −1.36)= 1.0 + 1.36= 2.36m of water the jet leaves the nozzle is 12 m/s.
Solution:
∴ Velocity at centre V = Cv 2gh Given:
= 0.98 × 2 × 9.81 × 2.36 = 6.668m / s Diameter of nozzle,
= = 0.025m
d1 25mm
Mean velocity, Velocity of jet at nozzle V1 = 12m / s
V =0.85 × 6.668 =5.6678m / s Height of point A h = 4.5m
Let the velocity of the jet at a height of
∴ Flow rate ofwater= V × area of pipe
4.5 m = V2
=5.6678 × 0.07068 =0.4006m3 / s

Example:
A pipe of dia 400 mm carries water at
velocity 25 m/s. The pressure at the points
A & B are given as 29.43 N/cm2 and

© Copyright Reserved by Gateflix.in No part of this material should be copied or reproduced without permission
Considering the vertical motion of the jet
from the outlet of the nozzle to the point A
(neglecting any loss of energy.)
Initial velocity,
=
u V=
1
12m / s
Final velocity,
V = V2
Value of g = 9.81m / s2 and h=4.5m
Using, V 2 − U2 =
2gh, we get
V22 − 122 = 2 × ( −9.81) × 4.5
∴ V=
2
122 − (2 × 9.81 × 4.5)

= 144 − 88.29 =7.46m / s


Now, applying continuity equation to the
outlet of nozzle and at point A, we get
a1 v 1 = a 2 v 2
Or
π 2
A1 V1 4 D1 × V1
=
A2 =
V2 V2
π× 0.0252 × 12
= 0.0007896
4 × 7.46
Let D2 = Diameter of jet at point A
Then,
π 2
=
A2 = D 0.0007896
4 2
0.0007896 × 4
=D2 = 0.0317m= 31.7mm
π

© Copyright Reserved by Gateflix.in No part of this material should be copied or reproduced without permission
6 DYNAMICS OF FLUID FLOW

6.1 INTRODUCTION
momentum minus net rate of in-flow
In case the surfaces cause a change in the across the control surface.
magnitude and direction of the velocity of This can also be written as
the fluid particles, the fluid particles exert a ∑F = ρ2 Q 2 V2 – ρ2 Q 2 V2
force on the surface. In turn the surfaces
Where,
exert an equal and opposite force on the
fluid particles. The force exerted by moving ⍴ is the density ( Kg / m3 )
fluid particles on the surface is called Q is the volume flow rate ( m3 / s )
dynamic force. Dynamic force always
If the fluid is incompressible, then
involves a change in the magnitude and
direction of the velocity of the fluid. ∑F = ρQ ( ∆V )

6.2 IMPULSE MOMENTUM PRINCIPLE 6.3 FORCE EXERTED BY A FLUID ON A


PIPE BEND
When applied to a single body Newton’s
second law can be started as “The sum of
forces on the body equals the rate of
change of momentum of the body in the
direction of the force.
∑ F = d(mV) / dt
This can also be written as
Free body of fluid element
∑ Fdt = d(mV) Force exerted on fluid element along x
Where,
direction
M is the mass of the body
P1A1 − A 2 A 2 cos θ − Fx
V is the velocity of the body
dt is the time. By impulse momentum principle for
This also means the impulse Fdt equals the incompressible fluid
change in momentum of the body during ∑ Fx =ρ ( ∆V )x
the time dt. P1A1 − P2 A 2 cos θ − Fx
For flowing fluid, the principle can be
= rate of change of momentum
stated as “The sum of forces on the fluid
= ρQ[V2 cos θ − V1 ]
equals the difference between the
momentum flowing in and momentum Fx =ρQ[V1 − V2 cos θ] + P1A1 − P2 A 2 cos θ
flowing out and the change in momentum ΣFy =ρQ ( ∆V ) y
of the fluid inside the control volume.
Under steady flow conditions, the last term P2 A2 sin   Fy  Q V2 sin  
vanishes. So the forces in the fluid is given Fy = −(ρQV2 sin θ + P2 A 2 sin θ)
by Dynamics of Fluid Flow -ve sign indicates that direction of force is
∑ F=d ( mV ) / dt out – d ( mV ) / dt in opposite i.e upwards
In other words, the net force on the fluid
mass is equal to the net rate of out flow of 6.4 FORCE EXERTED BY A FLUID ON
VERTICAL STATIONARY PLATE

© Copyright Reserved by Gateflix.in No part of this material should be copied or reproduced without permission
equals ρAV1 kg/s. But when the vane
moves away from the direction of the jet
with a velocity u, then the mass of water
striking the vane equals ρA(V1–u). (V-u) is
the relative velocity between the jet and
the vane.
Force exerted on fluid in x direction is
For tangential balde r= r2= r
equal to change in momentum 1

 1 = ρa V12 =
Force on blade  ( Vw1  ± Vw 2 )
F m 
Fx = mV
Fy = 0 =
Power  [ Vw1  ± Vw 2 ] U
P m  
 =
m ρA(V1 − U)
6.5 FORCE EXERTED BY A FLUID ON U = πDN / 60
STATIONARY BLADE Where D is mean diameter & N is speed of
rotation in rpm
The direction of the velocity is changed.
There is negligible change in the
magnitude. In the case considered pressure
forces are equal both at inlet and outlet.

Example:
Force exerted by fluid in x direction with A 45° reducing bend is connected in a pipe
the assumed direction. line, the diameters at the inlet and outlet of
−Fx = ρQ(V2 cosβ − V1cosα) the bend being 600mm and 300mm
respectively. Find the force exerted by
Q = AV1 water on the bend if the intensity of
Fx = ρQ(V1cosα − V2 cosβ) pressure at inlet to bend is 8.829N/ cm2
Force exerted by fluid in y direction with and rate of flow of water is 600 liters/s.
the assumed direction.
Fy = ρQ(V2sinβ − V1sinα) Solution:

6.5.1 FORCE EXERTED BY A FLUID


ON MOVING BLADE

In the case of the moving vane it is


necessary to consider both the absolute
and relative velocities. The other difference
is that the amount of fluid that strikes a
moving vane at any time interval is
Given:
different from that which strikes the
stationary vane. If a jet of area A with a Angle of bend, θ = 45°
velocity V1 strikes a stationary vane, the Dia at inlet, D1 = 600mm = 0.6m
mass impinging per unit time on the vane

© Copyright Reserved by Gateflix.in No part of this material should be copied or reproduced without permission
∴ π π
A1 = D12 = ( .6 ) = 0.2827 m2
2
∴ Resultant force, FR = Fx2 +Fy2
4 4
Dia at outlet, D2 = 300mm = 0.30m  1911.42  6322.2
2

∴ A2 = π ( .3) = 0.07068m2
2
 20890.9N
4
Pressure at inlet
p1 = 8.829 N/ cm2 = 8.829×104 N/ m2
Q = 600lit/ s = 0.6m3 / s
Q 0.6
V1 = = = 2.122m/ s
A1 .2827 The angle made by resultant force with x-
Q 0.6
axis is given by equation
V2 = = = 8.488m/ s F
A2 .07068 ∴ 6322.2
tan θ = y = = 0.3175
Applying Bernoulli’s equation at section (1) Fx 19911.4
and (2), we get
θ = tan-1 (.3175)=17°36'
p1 V12 p V2
+ +z1 = 2 + 2 +z2
ρ g 2g ρ g 2g Example:
But z1 = z2 A nozzle of diameter 20 mm is fitted to a
p1 V12 p2 V22 8.829×104 2.1222 p 8.4882 pipe of diameter 40mm. Find the force
+ = + or + = 2+
ρ g 2g ρ g 2g 1000×9.81 2×9.81 ρ g 2×9.81 exerted by the nozzle on the water which is
flowing through the pipe at the rate of
9+ .2295= p2 / ρ g +3.672 1.2m3 / minute .
p2 Solution:
∴ = 9.2295 − 3.672= Given:
ρg
Dia of pipe,
=5.5575m of water D1 = 40mm = 40×10−3 m = .04m
∴ p2 =5.5575×1000×9.81N/ m2 π 2 π
( )
2
2
∴ Area, A1 = D1 = .04 = 0.001256m
4 2 4 4
=5.45×10 N/ m
Dia of nozzle, D2 = 20mm = 0.02m
Forces on the bend in x- and y- directions
π
( .02) = .000314m2
2
are given by equations as ∴ Area, A2 =
Fx = ρ Q  V1 − V2cos θ  + p1 A1 − p2 A2cos θ 4
Discharge,
= 1000×0.6 2.122 − 8.488cos45° 1.2 3
3
+8.829×10 ×.2827 − 5.45×10 ×.07068×cos45°
4 Q =1.2m
4 / minute = m / s = 0.02m3 / s
60
= −2327.9+24959.6 − 2720.3= 2459.6 − 5048.2
=19911.4N
And
Fy  Q V2 sin    P2 A2 sin 
= 1000×0.6  −8.488sin 45°  − 5.45×104 ×.07068×sin45°
= −3601.1 − 2721.1= −6322.2N Applying continuity equation at section (1)
- ve sign means Fy is acting in the and (2)
A1 V1 = A 2 V2 = Q
downward direction

© Copyright Reserved by Gateflix.in No part of this material should be copied or reproduced without permission
Q 0.2 speed of rotation of the arm, if free to
∴ V1 = = =15.92m/ s rotate.
A1 .001256
Q 0.2
And V2 = = = 63.69m/ s
A 2 .000314
Applying Bernoulli’s equation at section (1)
and (2) ,we get
P1 V12 P2 V22
+ + z1 = + + z2
ρg 2g ρg 2g Solution:
p2 Given:
Now z1 = z2 , = atmospheric pressure=0 Dia of each nozzle =0.8cm =0.008 m
ρg ∴ Area of each nozzle
p1 V12 V22 π
( .008 ) = 0.00005026m2
2
∴ + = =
4
ρ g 2g 2g
Velocity of flow at each nozzle = 10m / s

p1 V22 V12
= − =
63.692

(
15.922
=
) ( ) ∴ Discharge through each nozzle
ρ g 2g 2g 2×9.81 2×9.81 Q = Area× Velocity
= 206.749 − 12.917 = .00005026×10 = .0005026m3 / s
= 193.83m of water Torque exerted by water coming through
∴ p1 =193.83×1000×9.81 N2 nozzle A on the sprinkler =moment of
m momentum of water through A
N = rA × ρ ×Q× VA = 0.25×1000×.0005026×10 clockwise
= 1901472 2
m Torque exerted by water coming through
Let the force exerted by the nozzle on nozzle B on the sprinkler
water = Fx = rB × ρ ×Q× VB = 0.20×1000×.0005026×10 clockwise
Net force in the direction of x=rate of ∴ Total torque exerted by water on
change of momentum in the direction of x sprinkler
∴ p1 A1 − p2 A 2 + Fx = ρ Q ( V2 − V1 ) = .25×1000×.0005026×10 + .20×1000×.0005026×10
=1.2565+1.0052= 2.26Nm
Where p2 =atmospheric pressure =0 and ∴ Torque required to hold the rotating
ρ =1000 arm stationary =Torque exerted by water
∴ 1901472×.001256 − 0+Fx =1000× on sprinkler
= 2.26Nm
0.02( 63.69 − 15.92)
Or 2388.24+Fx = 916.15 Speed of rotation of arm, if free to rotate
∴ Fx = −2388.24+ 916.15= −1472.09 Let ω = speed of rotation of the sprinkler
-ve sign indicates that the force exerted by The absolute velocity of flow of water at the
the nozzles on water is acting from right to nozzles A and B are
V1 = 10.0 − 0.25× ω and V2 = 10.0 − 0.20× ω
left.
Torque exerted by water coming out at A,
Example: on sprinkler
A lawn sprinkler shown in Fig has 0.8cm = rA × ρ ×Q× V1 = 0.25×1000×.0005026× (10 − 0.25ω)
diameter nozzle at the end of a rotating = 0.12565(10 − 0.25ω)
arm and discharges water at the rate of
10m/s velocity. Determine the torque Torque exerted by water coming out at B,
required to hold the rotating arm on sprinkler
stationary. Also determine the constant

© Copyright Reserved by Gateflix.in No part of this material should be copied or reproduced without permission
= rB × ρ ×Q× V2 = 0.20×1000×.0005026× (10.0 − 0.2ω) Example:
= 0.10052(10.0 − 0.2ω) A jet of water of 30mm diameter strikes a
hinged square plate at its centre with a
∴ Total torque exerted by water velocity of 20m/s. The plate is deflected
 0.12565(10.0  0.25 )  0.10052(10.0  0.2 ) through an angle of 20° . Find the weight of
Since moment of momentum of the flow the plate.
entering is zero and no external torque is If the plate is not allowed, to swing, what
applied on sprinkler, so the resultant will be the force required at the lower
torque on the sprinkler must be zero. edges of the plate to keep the plate in
 0.12565(10.0  0.25 )  0.10052(10.0  0.2 )  0 vertical position.
1.2565 − 0.0314ω +1.0052 − 0.0201ω = 0
1.2565 + 1.0052 = ω ( 0.0314 + 0.0201)
2.2617 = 0.0515ω
2.2617
∴ ω= = 43.9rad/ s
0.0515
60× ω 60× 43.9
and N = = = 419.2 r.p.m.
2π 2π
Solution:
Example: Given:
Water is flowing through a pipe at the end Diameter of the jet, d=30mm=3cm=0.03m
of which a nozzle is fitted. The diameter of ∴ Area,
the nozzle is 100 mm and the head of water π 2 π
(.03)
2 2
at the centre of nozzle is 100mm. Find the a=
4
d =
4
= .0007068 m
force exerted by the jet of water on a fixed Velocity of jet, V = 20 m/ s
vertical plate. The co-efficient of velocity is
Angle of swing θ = 20°
given as 0.95.
Using equation for angle of swing.
Solution:
Given:  AV 2
sin  
Diameter of nozzle d = 100mm = 0.1m W
Head of water H = 100 Or sin 20° = 1000×
.0007068× 202 282.72
=
Co-efficient of velocity Cv = 0.95 W W
282.72
Area of nozzle, a = π ( .1) = 0.007854m2
2 ∴ W= = 826.6 N
4
sin 20°
Theoretical velocity of jet water is given as If the plate is not allowed to swing, a force
p will be applied at the lower edge of the
Vth = 2gH = 2×9.81×100 plate as shown in fig. The weight of the
Actual velocity plate is acting vertically downward through
But Cv =
Theoretical velocity the C.G. of the plate.
∴ Actual velocity of jet of water, Now, let
V = Cv × Vth = 0.95× 44.294 = 42.08 m/ s F=Force exerted by jet of water
H=Height of plate
Force on a fixed vertical plate is given by =Distance of P from the hinge
equation The jet strikes at the centre of the plate and
F= ρaV 2 =1000 × .00785 × 442.082 hence distance of the center of the jet from
(In S.I. units ρ for water =
1000kg/ m ) 3
hinge =
h
2
= 13907.2N
= 13.9kN Taking moments about the hinge O,

© Copyright Reserved by Gateflix.in No part of this material should be copied or reproduced without permission
h Solution:
Ph  F 
2 Given:
F× h F ρ aV 2 Velocity at inlet V1 = 30 m/ s
∴ P= = =
2× h 2 2 Velocity at outlet, V2 = 25 m/ s
.0007068× 202 Mass per second = 0.8kg/ s
= 1000× = 141.36 N
2

Example:
A jet of water of diameter 10 cm strikes a
flat plate normally with a velocity of 15
m/s. The plate is moving with a velocity of
6 m/s in the direction of the jet and away
from the jet. Find the power and efficiency
of the jet.
Solution:
The given data from problem is
Force in the direction of jet,
a = .007854m2 , V = 15m/ s, u = 6m/ s
Fx = Mass per second× ( V1x - V2 x )
Also work done per second by the jet
W   a V  u  u =3817.02Nm/s
2 Where V1x = Initial velocity in the
(i)Power of the jet in kW direction of x=30m/s
Work done per second 3817.02 V2 x = Final velocity in the direction of x
= = = 3.817 kW
1000 1000 1
= 25cos 60° = 25× = 12.5 m/ s
(ii)Efficiency of the jet (η) 2
Output of the jet per second ∴ Fx = 0.8 [30 − 12.5] = 0.8×17.5 = 14.0 N
=
Input of the per second Similarly, force normal to the jet,
Where output of jet/ sec Fy = Mass per second× ( V1y − V2 y )
= Work done by jet per second = 3817.02 Nm/ s
= 0.8 [ 0 − 25sin 60°] = −17.32 N
And input per second
− ve sign means the force , Fy is acting in
 = Kinetic energy of the jet / sec
1  mass  2 1 1 the vertically downward direction .
  V  (  AV ) V 2
  AV 3 ∴ resultant force on the vane
2  sec  2 2
= Fx2 + Fy2 = 142 + ( −17.32 ) = 22.27 N
2
1
 1000 0.007854153  13253.6 Nm / s
2 The angle made by the resultant with x-axis
3817.02 Fy −17.32
∴ η of the jet = = 0.288 = 28.8% tan θ = = = −1.237
13253.6 Fx 14.0
− ve sign means the angle θ is in the
Example: clockwise direction with x-axis as shown in
A jet of water from a nozzle is deflected fig .
through 60° from its original direction by a
∴ θ = tan 1.237 = 51°2.86 '
−1
curved plate which it enters tangentially
without shocks with a velocity of 30m/s
Example:
and leaves with a mean velocity of 25m/s.
A jet of water of diameter 50mm, having a
If the discharge from the nozzle is 0.8kg/s
velocity of 20m/s strikes a curved vane
calculate the magnitude and direction of
which is moving with a velocity of 10m/s in
the resultant force on the vane, if the vane
the direction of the jet . The jet leaves the
is stationary.

© Copyright Reserved by Gateflix.in No part of this material should be copied or reproduced without permission
vane at an angle of 60° to the direction of Now Vw = HF = GF− GH
2
motion of vane at outlet .Determine: = u 2 − Vr cos φ = 10 − 10×cos 60° = 10 − 5 = 5 m/ s
i) The force exerted by the jet on the vane 2

(i) The force exerted by the jet on the vane


in the direction of motion
in the direction of motion is given by
ii) Work done per second by the jet
equation
Solution:
Given: Fx = ρ aVr1  Vw1 − Vw 2  ((-ve sign is taken as β
Diameter of the jet D=50mm=0.5m is an obtuse angle)
π = 1000×.001963×10 [ 20 − 5] N = 294.45 N
∴ Area, a = ( 0.5 ) = .001963m 2
2

4 (ii) Work done per second by the jet


Velocity of jet, V1 = 20 m/ s Fx × u = 294.45×10 = 2944.5 Nm/ s
Velocity of vane, u1 = 10 m/ s = 2944.5 W  Nm/ s = W ( watt ) 

Example:
A jet of water having a velocity of 15m/s
strikes a curved vane which is moving with
a velocity of 5m/s in the same direction as
that of the jet at inlet. The vane is so shaped
that the jet is deflected through 135° .The
diameter of jet is 100mm. Assuming the
vane to be smooth, find
i) Force exerted by the jet on the vane in
the direction of motion,
As jet and vane are moving in the same ii) Power exerted on the vane, and
direction iii) Efficiency of the vane
∴ α =0 Solution:
Angle made by the leaving jet, with the Given:
direction of motion= 60° Velocity of jet, V1 = 15 m/ s
∴ β = 180° - 60° = 120° Velocity of vane u = u1 = u 2 = 5 m/ s
For this problem ,we have
At inlet jet and vane are in the same
u1 = u 2 = u = 10 m/ s
direction , hence α = 0
Vr1 = Vr2 Diameter of jet, d=100mm=0.1m
From fig ,we have π
∴ Area, a = ( 0.1) = .007854 m 2
2
Vr1 = AB− AC = V1 − u1 4
= 20 − 10 = 10 m/ s Angle of deflection of the jet
Vw1 = V1 = 20 m/ s = 135° = 180° − φ .
∴ Vr = Vr = 10 m/ s
2 1
∴ φ = 180° − 135° = 45°
Now in ∆EFG, EG = Vr = 10 m/ s, 2

GF = u 2 = 10 m/ s ,
∠ GEF = 180° − ( 60° + φ ) = (120° − φ )
From sine rule , we have
EG GF 10 10
= or =
sin 60° sin (120° − φ ) sin 60° sin (120° − φ )
Or sin 60° = sin (120° − φ )
∴ 60° = 120° − φ or120° − 60° = 60°

© Copyright Reserved by Gateflix.in No part of this material should be copied or reproduced without permission
As vane is given smooth hence Vr1 = Vr2 From Problem, V1 = 15 m/ s
From the inlet velocity triangle, which is a u = u1 = u 2 = 5 m/ s
straight line in this case, we have α = 0 a = .007854 m 2
Vr1 = V1 − u1 = 15 − 5 = 10 m/ s
φ = 45°,
Vw1 = V1 = 15 m/ s
Vw1 = 15 m/ s and Vw 2 = 2.07 m/ s
From the outlet velocity triangle DEG, we For the series of vanes, mass of water
have striking per second
Vr2 = Vr1 = 10 m/ s =Mass of water coming out from nozzle
u 2 = u1 = u = 5 m/ s = ρ aV1 = 1000×.007854×15 = 117.72
Vr2 cos φ = u 2 + Vw 2 or 10 cos 45° = 5 + Vw 2 i) Force exerted by the jet on the vane in
∴ Vw 2 = 10 cos 45° − 5 = 7.07 − 5 = 2.07 m/ s the direction of motion
Fx = ρ aV1  Vw + Vw 
i) Force exerted by the jet on the vane in 1 2

the direction of motion is given by = 117.72 [15 + 2.07 ] = 2009.5 N


equation as ii) Power of the vane in kW
Fx = ρ aVr1  Vw1 − Vw 2  ((-ve sign is taken as Work done per second Fx × u 2009.5×5
= = kW =
β is an obtuse angle) 1000 1000 1000
= 10.05 kW
 1000 0.0078541015  (2.07) 
iii) Efficiency
=1340.6N Work done per second
η=
1
× ( mass per second ) × V12
ii) Power of the vane is given as 2
Fx × uNm/ s = 1340.6×5 = 6703 W = 6.703kW 2009.5× 5.0
= = 0.7586 or 75.86%
iii) Efficiency of the vane 1
×117.72×152
work done on vane per second 2

Kinetic energy supplied by jet per second

Fx × u Fx × u
= =
1 1
× ( mass per second ) × V 2 × ( ρ aV1 ) × V12
2 2
1340.6×5.0
= = 0.505 = 50.5%
1
× (1000×.007854×15 ) ×152
2

Example:
If in above problem, the jet of water instead
of striking single plate, strikes a series of
curved vanes, find for the data given in the
above problem.
i) Force exerted by the jet on the vane in
the direction of motion,
ii) Power exerted on the vane and
iii) Efficiency of the vane
Solution:
Given:

© Copyright Reserved by Gateflix.in No part of this material should be copied or reproduced without permission
GATE QUESTIONS

Q.1 A water container is kept on a


weighing balance. Water from a tap
is falling vertically into the container
with a volume flow rate of Q; the
velocity of the water when it hits the
water surface is U . At a particular
instant of time the total mass of the
container and water is m. The force a) ( ρU 2 / 2k ) πDS2
registered by the weighing balance
 D2 
at this instant of time is b) ( ρU 2 / 8k )  2 − 1 πDS2
a) mg + ρQU b) mg + 2ρQU  Dt 
c) mg + ρQU2/2 d) ρQU2/2
D 2

[GATE–2003] c) ( ρU 2 / 2k )  2 − 1 πDS2
 Dt 
Q.2 The following data about the flow of
D 4

liquid was observed in a continuous d) ( ρU 2 / 8k )  4 − 1 πDS2
chemical process plant:  Dt 
[GATE–2003]
Q.4 A venturimeter of 20 mm throat
diameter is used to measure the
velocity of water in a horizontal pipe
Mean flow rate of the liquid is of 40 mm diameter. If the pressure
a) 8.00 litres/sec b) 8.06 litres/sec difference between the pipe and
c) 8.16 litres/sec d) 8.26 litres/sec throat sections is found to be 30 kPa
[GATE–2004] then, neglecting frictional losses, the
flow velocity is
Q.3 Air flows through a venturi and into a) 0.2 m/s b) 1.0 m/s
atmosphere. Air density is ρ; c) 1.4 m/s d) 2.0 m/s
atmospheric pressure is 𝑝𝑝𝑎𝑎 ; throat [GATE–2005]
diameter is Dt ; exit diameter is D
and exit velocity is U . The throat is Q.5 A U-tube manometer with a small
connected to a cylinder containing a quantity of mercury is used to
frictionless piston attached to a measure the static pressure
spring. The spring constant is k . The difference between two locations A
bottom surface of the piston is and B in a conical section through
exposed to atmosphere. Due to the which an incompressible fluid flows.
flow, the piston moves by distance x. At a particular flow rate, the
Assuming incompressible frictionless mercury column appears as shown
flow, x is in the figure. The density of mercury
is 13600 kg/m3 and g = 9.81 m/s2.
Which of the following is correct ?

© Copyright Reserved by Gateflix.in No part of this material should be copied or reproduced without permission
a) Flow direction is A to B and
PA− PB= 20 kPa
b) Flow direction is B to A and a) 6.4 b) 9.0
PA − PB = 1.4 kPa c) 12.8 d) 25.6
c) Flow direction is A to B and [GATE–2011]
PB − PA= 20 kPa
d) Flow direction is B to A and Q.8 Water is coming out from a tap and
PB − PA= 1.4 kPa falls vertically downwards. At the
[GATE–2005] tap opening, the stream diameter is
20 mm with uniform velocity of 2
Q.6 Consider steady, incompressible m/s. Acceleration due to gravity is
and irrotational flow through a 9.81 m/s2 . Assuming steady,
reducer in a horizontal pipe where inviscid flow, constant atmospheric
the diameter is reduced from 20 cm pressure everywhere & neglecting
to 10 cm. The pressure in the 20 cm curvature and surface tension
pipe just upstream of the reducer is effects, the diameter in mm of the
150kPa. The fluid has a vapour stream 0.5 m below the tap is
pressure of 50 kPa and a specific approximately
weight of 5 kN/m3. Neglecting a) 10 b) 15
frictional effects, the maximum c) 20 d) 25
discharge (in m3/s) that can pass [GATE–2013]
through the reducer without
causing cavitation is Q.9 Within a boundary layer for a steady
a) 0.05 b) 0.16 incompressible flow, the Bernoulli
c) 0.27 d) 0.38 equation
[GATE–2009] a) holds because the flow is steady
b) holds because the flow is
Q.7 Figure shows the schematic for the incompressible
measurement of velocity of air c) holds because the flow is
(density =1.2 kg/m3) through a transitional
constant area duct using a pitot d) does not hold because the flow is
tube and a water tube frictional
manometer. The differential head of [GATE–2015 (2)]
water (density = 1000 kg/m3) in
the two columns of the manometer Q.10 A Prandtl tube (Pitot-static tube
is 10 mm. Take acceleration due to with C = 1) is used to measure the
gravity as 9.8 m/s2. The velocity of velocity of water. The differential
air in m/s is manometer reading is 10 mm of
liquid column with a relative density
of 10. Assuming g = 9.8 m/s2, the
velocity of water (in m/s) is_____.
[GATE–2015 (3)]

© Copyright Reserved by Gateflix.in No part of this material should be copied or reproduced without permission
Q.11 Water (ρ = 1000 kg/m3) flows
through a venturimeter with inlet
diameter 80 mm and throat
diameter 40 mm. The inlet and
throat gauge pressures are
measured to be 400 kPa and 130
kPa respectively. Assuming the
venturimeter to be horizontal and
neglecting friction, the inlet velocity
(in m/s) is _____. [GATE–2017 (2)]
[GATE–2015 (1)]

Q.12 The water jet exiting from a Q.14 A tank open at the top with a water
stationary tank through a circular level of 1 m, as shown in the figure,
opening of diameter 300 mm has a hole at a height of 0.5 m. A free
impinges on a rigid wall as shown in jet leaves horizontally from the
the figure. Neglect all minor losses smooth hole. The distance X (in m)
and assume the water level in the where the jet strikes the floor is
tank to remain constant. The net
horizontal force experienced by the
wall is ___________ kN. Density of
water is 1000 kg/m3.
Acceleration due to gravity g = 10
m/s2.

[GATE–2016 (3)] a) 0.5 b) 1.0


c) 2.0 d) 4.0
Q.13 The arrangement shown in the figure [GATE–2018 (1)]
measures the velocity V of a gas of density
1kg / m3 flowing through a pipe. The Q.15 Air flows at the rate of 1.5 m3 / s
acceleration due to gravity is 9.81m / s2 . If through a horizontal pipe with a
the manometric fluid is water (density gradually reducing cross section as
1000kg / m3 ) and the velocity V is 20 m/s, shown in the figure. The two cross-
sections of the pipe have diameters of
the differential head h (in mm) between the
400 mm and 200 mm. Take the air
two arms of the manometer is ____________
density as 1.2 kg / m3 and assume
inviscid incompressible flow. The
change in pressure (P2 − P1 ) (in kPa)
between sections 1 and 2 is

© Copyright Reserved by Gateflix.in No part of this material should be copied or reproduced without permission
a) - 1.28 b) 2.56
c) - 2.13 d) 1.28
[GATE–2018 (2)]

ANSWER KEY:
1 2 3 4 5 6 7 8 9 10 11 12 13 14 15
(a) (c) (d) (d) (a) (b) (c) (b) (d) 1.32 6 8.76 20.4 (b) (a)

© Copyright Reserved by Gateflix.in No part of this material should be copied or reproduced without permission
EXPLANATIONS

Q.1 (a) Q.3 (d).

First of all we have to take two


sections (1) and (2).
Applying Bernoulli’s equation at
Given : Flow rate = Q sections (1) and (2).
Velocity of water when it strikes the p1 V12 p V2
water surface = U + + Z1 = 2 + 2 + Z2
ρg 2g ρg 2g
Total Mass (container + water) = m
Force on weighing balance due to p 2 V2 p 2 V22
2
+ = + (Since Z1 = Z2)
water striking the surface = Change ρ 2 ρ 2
in momentum ρ 2
p1 − p=  V2 − V12  ...(i)
= Initial Momentum−Final 2
2

momentum Apply continuity equation, we get.


= ρQU– 0 ( Since final velocity is A1V1 = A2V2
zero) π 2 π
Weighing balance also experiences D t V1 = D 2 U
4 4
the weight of the container and 2
water. So, Weight of container and D
∴ V1 =   × U ...(ii)
water = mg. Therefore, total force on  Dt 
weighing balance = ρQU + mg Substitute the value of V1 from
equation (ii) into the equation (i)
Q.2 (c)
ρ  2  D  2
4
Flow Mean Frequency fx p1 – p 2 =  U −   U 
Rate value flow (f) 2  Dt  
rate (x) 
7.5-7.7 7.6 1 7.6 ρ  D   2
4

7.7-7.9 7.8 5 39 = 1 −    U ....(iii)


7.9-8.1 8.0 35 280 2   Dt  
 
8.1-8.3 8.2 17 139.4
From the figure, we have
8.3-8.5 8.4 12 100.8
8.5-8.7 8.6 10 86 Spring force = Pressure force due to
Σf = 80 air
π 2
Σfx = 652.8 −=kx A s ( p1 – = p2 ) Ds × ( p1 – p 2 )
Σfx 652.8 4
∴ Mean flow rate == = 8.16 From equation (iii)
Σx 80

© Copyright Reserved by Gateflix.in No part of this material should be copied or reproduced without permission
Given:
π 2 ρ  D  
4

=
kx Ds ×   − 1 U 2 ρmercury = 13600 kg/m3,
4 2  D t  
 Δh = 150 mm = 0.150 m
ρU 2  D   Static pressure difference for U-tube
4

x=   − 1 πDs2 differential manometer is given by,


8k  D t  
 PA−PB= ρg(hA − hB )
Q.4 (d). = 13600×9.81×0.15
= 20.01 × 103 Pa
≈ 20 kPa
PA – PB comes out to be positive.
Therefore, flow is from A to B.

Q.6 (b).

For continuity equation, we get


A1V1 = A 2 V2
π 2
d1
A1
=
V2 =V1 4 × V1
A2 π 2
d2 Given: PV= 50 kPa, w = 5 kN/m3= ρg
4
2 Consider steady, incompressible and
 d1 
2
 40  irrotational flow and neglecting
= =  V1   V1
 d2   20  frictional effects.
Applying Bernoulli’s equation, we Applying continuity equation at
get section (1) and (2).
A1 V1= A2 V2
p1 V12 p V2
+ + z1 = 2 + 2 + z 2 x x
( d1 ) ×= ( d 2 ) × V2
2 2
ρg 2g ρg 2g V1
4 4
For horizontal pipe; Substitute the values of d1 and d2,
z1 = z 2 we get
p1 − p 2 V22 − V12 x x
( 20 ) ×= (10 ) × V2
2 2
= V1
ρg 2g 4 4
∆p 1 2 400 V1 = 100 V2
=
ρ 2
(
V2 − V12 ) ⇒ V2 = 4V1 …(i)
30 ×103 1  Cavitation is the phenomenon of
⇒ = ( 4V1 ) − V12 
2
 formation of vapor bubbles in a
1000 2
2
flowing liquid in a region where the
15V1 pressure of liquid falls below the
= 30
2 vapor pressure[PL<PV]. So, we can
∴ V1 = 2m/s say that maximum pressure in
downstream of reducer should be
Q.5 (a) equal or greater than the vapor
It is a U -tube differential pressure. For maximum discharge
Manometer. In this manometer A PV= P2= 50 kPa
and B at different level and the Applying Bernoulli’s equation at
liquid contain in manometer has the point (1) and (2)
same specific gravity (only mercury P1 V12 P V2
is fill in the manometer) + + Z1 = 2 + 2 + Z2 =
ρg 2g ρg 2g

© Copyright Reserved by Gateflix.in No part of this material should be copied or reproduced without permission
Here Z1 = Z2 for horizontal pipe π π 2
= d12 × u= d2 × v
And w = ρg = 5 kN/m2 4 4
150 V12 50 (4V1 ) 2 V2 d2 × u
+ = + 20 = 15 1 d 22 = 1 2
5 2g 5 2g 2g v
V1 = 5.114 m/s (0.02) 2 × 2
And d2 =
2

(3.716)
V2=4V1 =4×5.114=20.46m/ s
d2 = 14.67 ≈ 15mm
Maximum discharge,
Qmax = A1 V1
Q.9 (d)
π
= D12 V1 Bernoulli equation does not hold
4 because the flow is frictional.
π
= × 0.22 × 5.114
4 Q.10 (1.3 to 1.34)
=0.16 m3/s Velocity of water = C v 2gh
Cv = 1 (Given)
Q.7 (c)
Given: ρs = 1.2 kg/m3, ρM = 1000  sg 
=h x  − 1
kg/m3  s0 
x = 10 × 10-3m, g = 9.8 m/s2 = 0.01(10-1) = 0.09m
If the difference of pressure head ‘h’ ∴ Velocity of flow =√2 × 9.8 × 0.09
is measured by knowing the = 1.328 m/s
difference of the level of the
manometer liquid say x .Then Q.11 (6)
 SG   pw  p1 v12 p 2 v 22 =p1 400 ×103 Pa 
=h x  w − 1=  x  − 1 + = +
 SG 2   p2   
ρg 2g ρg 2g  = p 2 130 ×103 Pa 
1000  v1 × 802 =v 2 × 402
= 10 ×10 − 3  − 1
 1.2  v 2 = 4v1
= 8.32 m
Substituting v2 and solving for v1
Velocity of air V = 2gh we get
= 2 × 9.8 × 8.32 = 12.8 m/s v1 = 6m/s

Q.8 (b) Q.12 (8.76 to 8.78)


Force, F = [ρAV ] v
= ρAV 2
Where velocity, V = 2gh
= 2 ×10 × 6.2
= 11.136 m/s
π
∴ F = 103 × × 0.32 ×11.1362
4
v 2 = u2 + 2gh = 8765.78 N
v 2 = 22 + 2 × 9.81 × 0.5 = 8.765 KN
v = 3.716 m/s
By continuity equation
Q1 = Q 2 Q.13 (20.4)

© Copyright Reserved by Gateflix.in No part of this material should be copied or reproduced without permission
ρ 
=v 2gX  m − 1
ρ 
ρ 
=V 2 2gX  m − 1 Applying Bernoulli’s equation for points (1)
ρ 
and (2) :
1000 
400 = 2 × 9.81× h  − 1 V12 V2
 1  P1 + ρair P2 + ρair 2 + Z 2 .......(1)
+ Z1 =
h = 20.4mm 2 2
Q.14 (b) But Z1 = Z2
4Q 4 1.5
and =
V1 = = 11.937 m/s
πd1
2
π  0.42
=  V1 47.746 m/s
& V2 4=
From equation (1) ,
ρair
(P 2 1 )
− P=
2
(V1
2
− V22 )

=
1.2
2
(11.937 2 − 47.7462 )

= −1.28 kPa

The velocity of jet at the exit of hole is given


by u = 2 gh
The jet performs projectile motion with x
component of velocity u = 2 gh
and y component v = 0 at the exit of hole.
The time‘t’ taken by jet to reach the ground
can be obtained by kinematic
equation of jet motion as:
1 1
= y  0  t + gt 2
2 2
2y
i.e., t =
g
The horizontal distance traveled by jet in the
same time is

2y
= t
x u= 2 gh 
g
h  y 2 0.5
= 2= =  0.5 1m
Q.15 (b)

© Copyright Reserved by Gateflix.in No part of this material should be copied or reproduced without permission
7 FLOW THROUGH CONDUITS/PIPES

7.1 INTERNAL FLOW

Fluids are conveyed (transported) through


closed conduits in numerous industrial
processes. It is found necessary to design
the pipe system to carry a specified
quantity of fluid between specified
locations with minimum head loss.
The flow may be laminar with fluid flowing
in an orderly way, with layers not mixing
macroscopically. The momentum transfer 7.1.2 ENTRY LENGTH
and consequent shear induced is at the
molecular level by pure diffusion. Such flow In internal flow, the boundary layer
is encountered with every viscous fluids. develops all over the circumference. At
The flow turns turbulent under certain some distance from the entrance, the
conditions with macroscopic mixing of fluid boundary layers merge. The velocity profile
layers in the flow. At any location, the beyond this point remains unchanged. The
velocity varies about a mean value. distance upto this point is known as entry
The flow is controlled by (i) pressure length.
gradient (ii) the pipe diameter or hydraulic
mean diameter (iii) the fluid properties like
viscosity and density and (iv) the pipe
roughness.

7.1.1 BOUNDARY LAYER


7.2 LAMINAR FLOW/VISCOUS FLOW
When fluid flows over surface, the
molecules near the surface are brought to 7.2.1 ANALYSIS OF FULLY
rest due to the viscosity of the fluid. The DEVELOPED LAMINAR FLOW IN
adjacent layers also slow down, but to a CIRCULAR DUCT
lower and lower extent. This slowing down i) Shear stress
is found limited to a thin layer near the
surface. The fluid beyond this layer is not
affected by the presence of the surface. The
fluid layer near the surface in which
velocity of fluid is less than free stream
velocity is known as boundary layer.

© Copyright Reserved by Gateflix.in No part of this material should be copied or reproduced without permission
1  dP  2
C= −  R
4µ  dx 
1  dP 
−   R 2 − r2 
U=
4µ  dx 

2) for, r = 0, U =Umax
1  dP 
Umax = −   R 2
4µ  dx 
  r 2 
=U Umax 1 −   
  R  

At low velocity the fluid moves in iii) Average velocity


layers. The shear stress in laminar flow dQ = UgdA
is given by dA = 2πrdr
du   r 2 
τw =µ dQ = Umax 1 −    × dA
dy   R  
force balance on fluid element gives   r 2 
Pgπr2 − τ× 2πrdx − (P + dP)πr2 = 0 ∫ dQ = ∫ Umax 1 −  R   × 2πrdr
 
dP 2
τw 2πr =− πr Q=

π −dP 4 
dx  R
8µ  dx 
dP r π  −dP  4
τw =− ×  R
dx 2 Q 8µ  dx  1  dP  2
U= = =−  R
A πR 2 8µ  dx 
ii) Velocity distribution Umax
du =2
τw =µ U
dy
y= R − r (y is measured from the pipe iv) Pressure drop for length (l)
wall) Average velocity is given by
dy = −dr 1  dP  2
U=  R
du 8µ  dx 
∴τw = µ
dr By rearranging above equation
P
du dP r 1
8µU
L
∴−µ =− × − ∫ dP =
dr dx 2
P2

0 R
2

1  dP  r
∴ ∫ du =
µ ∫  dx  2
  dr Pressure drop across length ‘L’ is
1  dP  1 32µUL
∴ U =   r2 + C P1 − P2 = 2
µ  dx  4 D
On applying boundary conditions: P1 − P2 32µUL
=hf =
ρg ρgD2
1) For r = R, U = 0 The above equation is Hagen
1  dP  2 Poiseuille equation
0=  R + C
4µ  dx  7.2.2 FLOW OF VISCOUS FLUID BETWEEN
TWO PARALLEL PLATES

© Copyright Reserved by Gateflix.in No part of this material should be copied or reproduced without permission
In turbulent flow, instantaneous values of
i) U =1  dP 
−    ty − y 2  the velocity fluctuate about an average
2µ  dx 
value, which suggests that the velocity can
Where, be expressed as the sum of an average
t is the distance between parallel plates value u and a fluctuating component u’,
Velocity is max at y = t/2 u= u + u'
1  dP  2
Umax = −  t When a liquid is flowing through a pipe, the
8µ  dx 
velocity of the liquid layer adjacent to wall
 y y2 
U= 4Umax ×  − 2  is zero. The friction resistance for turbulent
t t  flow is.
ii) Shear stress distribution 1) ∝ V n where n varies from 1.5-2.0
1  dP  2) ∝ ρ
τ = −    t − 2y 
2  dx  3) ∝ A Area of contact
at centre, shear stress will be zero. 4) ∝ P Pressure
5) ∝ Nature of surface
iii) U = Avg. velocity= −1  dP  t 2
12µ  dx  .
7.3.1 EXPRESSION FOR COEFFICIENT
Umax 3
= OF FRICTION IN TERMS OF SHEAR
U 2 STRESS
iv) Pressure head for a given length Consider a fluid element of length ‘L’ &
12µUL diameter ‘d’
P1 − P2 = 2
t

7.3 TURBULENT FLOW IN PIPES

Most flows encountered in engineering


P1 A − P2 A − F =
0
practice are turbulent, and thus it is
important to understand how turbulence (P1 − P2 )A =
F
affects the wall shear stress. Turbulent flow F = force due to shear stress.
is characterized by random and rapid F= (
τO πdL )
fluctuations of swirling regions of fluid,
called eddy, throughout the flow. These (P1 − P2 )A =τO ( πdL )
fluctuations provide an additional P1 − P2 4fLV 2 τO πdL
= =
( )
mechanism for momentum and energy
ρg 2gd ρg
transfer. In laminar flow, fluid particles
flow in an orderly manner along pathlines, π 2
A= d
and momentum and energy are transferred 4
across streamlines by molecular diffusion. 2τo
In turbulent flow, the swirling eddies f=
ρV 2
transport mass, momentum, and energy to
other regions of flow much more rapidly
7.3.2 SHEAR STRESS IN
than molecular diffusion, thus greatly
TURBULENT FLOW
enhancing mass, momentum, and heat
transfer. As a result, turbulent flow is
Shear stress in turbulent flow is sum of
associated with much higher values of
shear stress due to viscous flow &
friction, heat transfer, and mass transfer
turbulent flow
coefficients
τ = τv + τt

© Copyright Reserved by Gateflix.in No part of this material should be copied or reproduced without permission
Viscous/laminar shear stress is given by du  du 
2

du τ =µ + ρl2  
τV =µ dy  dy 
dy
J. Boussinesq expressed turbulent shear in 7.3.5 VELOCITY DISTRIBUTION IN
terms of average velocity gradient TURBULENT FLOW IN PIPES
du
τt =η
dy du  du 
2

Where, τ =µ + ρl2  
dy  dy 
η is Eddy viscosity,
Viscous shear stress is negligible except
du is average velocity.
near the boundary.
Prandtl assumed that mixing length is
7.3.3 REYNOLD’S EXPRESSION
proportional to ‘y’
FOR TURBULENT SHEAR STRESS
l∝y
Reynold expressed turbulent shear stress l = ky
between two layers of a fluid at a small Where,
distance in terms of velocity fluctuations K is Karman factor = 0.4
2 2
τt =ρu'v'  du   du 
τt =ρ ( ky )   =ρk 2y 2  
2

Where,  dy   dy 
u’ and v’ are fluctuating velocity by rearranging the above equation
components in the direction of x & y due to
du τ
turbulence. =
τt will also be varying, Hence to find the dy ρk 2y 2
shear stress time avg on both side is du 1 τ
considered τ = ρu'v' =
dy ky ρ
τ 1
7.3.4 PRANDTL MIXING LENGTH THEORY U = ( ln y ) g +C
FOR TURBULENT SHEAR STRESS ρ k
τ (U* is known as Shear velocity)
The turbulent shear stress can only be U∗ =
ρ
calculated if u’ and v’ is known. According
to Prandtl, the mixing length ‘l’, is that Since y is distance from the surface of pipe
distance between two layers in the U = Umax at y=R
transverse direction such that the lumps of U*
=Umax lnR + C
fluid particles from one layer could reach k
the other layer and the lumps are mixed in U*
=C Umax − lnR
the other layer in such a way that k
momentum of particle in the direction of U*  y 
= U ln   + Umax
motion is same. k R
Prandtll expressed velocity fluctuations in U*  y 
terms of mixing length = U ln   + Umax
0.4  R 
du du
U' = l & V' = l y
dy dy =U Umax + 2.5U* ln  
R  
2
 du 
τt =ρl2  
 dy 
Total shear stress is given by

© Copyright Reserved by Gateflix.in No part of this material should be copied or reproduced without permission
This is Prandtl’s universal velocity δ'
distribution equation for turbulent flow in y' =
107
pipes. 11.6ν
(Umax − U) is known as velocity defect. δ' = *
U
U  y.100 
*

7.3.6 HYDRO DYNAMICALLY SMOOTH U = ln  


k  δ 
& ROUGH BOUNDARIES

For turbulent flow analysis along a 7.3.8 REYNOLDS’ NO IN TERMS


boundary, the flow is divided in two OF ROUGHNESS
portions. Viscous portion near the surface
is known as laminar sub layer. The U* k
boundaries are considered as smooth or Re =
ν
rough on the basis of ratio of average
Where,
irregularities ‘k’ and laminar sub layer
k is average height of irregularities
thickness ‘δ’
ν is kinematic viscosity
if k < δ Smooth
if k > δ Rough τ
U* = Shear velocity =
k ρ
if < 0.25 boundary is smooth
δ U* k
i) < 4, boundary is smooth
if
k
>6 boundary is rough ν
δ U* k
ii) 4 < < 100, boundary is in transition
ν
7.3.7 VELOCITY DISTRIBUTION FOR
U* k
TURBULENT FLOW IN SMOOTH PIPES iii) > 100, the boundary is rough.
ν
U* 7.3.9 VELOCITY IN ROUGH PIPES
=U ln y + C
k
=
at y 0, U=-∞ (Nikurddse’s Experiment) show that
k
y' =
It means at some finite distance from wall, 30
the velocity will be equal to zero. Let y’ be u*
the distance from wall where velocity is u = In(y / y ')
k
zero
 y 
y = y ', U=0 u = 2.5u *In  
 k / 30 
U*
U= 0= ln y '+ C  30y 
k u = 2.5u *In  
−U *  k 
C= ln y '
k
Example:
U*  y  A Pipe carrying water has avg irregularities
U = ln  
k  y'  of 0.15 mm. What type of boundary it is?
For smooth boundary (Shear stress developed is
y ' ∝ δ' 4.9N / m , ν =0.01strokes )
2

Where, Solution:
δ' is thickness of laminar sub layer k 0.15 ×10−3 m
=
From experiments, y’& δ' is given by τ0 =4.9N / m 2

© Copyright Reserved by Gateflix.in No part of this material should be copied or reproduced without permission
ν = 0.01strokes = 0.01' cm 2 / s = ∆10−6 m 2 / s .
ρ =1000Kg / m3
11.6ν
δ=
u*
11.6 ×10−6
δ=
u*
τ0 4.9 Head loss is ‘m’ is given by
=
u* = = 0.07m / s
(v − v )
2
ρ 1000
he = 1 2
Roughness Reynold No.: 2g
u *k 0.07 ×15 ×103
= 2) Loss due to sudden contraction: As
v 10−6
the liquid comes out of orifice it
7.4 LOSS OF ENERGY IN FLUID FLOW contract further and the area just
outside the orifice is lower compared to
7.4.1 MAJOR LOSS the area of the orifice. This section is
called as vena-contracta. The liquid
Darcy-Weisbach Formula: expands from vena contracta due to
which the loss of energy takes place.
4f .L.V 2
hf =
d.2g
h f = Loss of head due to viscosity.
16
f= for Re<2000 (Viscous flow).
Re
0.079
= 1
for Re varying from 4000 to 106.
Re 4
Energy loss is given by
(for turbulent flow in smooth pipes).
Where, v 2 .k
he =
L=Length, 2g
V = mean velocity of flow, Where,
d = diameter of pipe. 1 
2

=
k  − 1
7.4.2 MINOR ENERGY LOSS  Cc 
Cc is coefficient of contraction
1) Loss of head due to sudden if= =
Cc 0.62, k 0.375
enlargement: Due to sudden change of
diameter the liquid flowing from the Cc = 0.62, then k=0.375
smallest pipe is not able to follow the v 2 .k v2
abrupt change of boundary. Thus the ∴ h e= = 0.375 2
2g 2g
flow separates from the boundary and
turbulent eddies are formed. if Cc is not given
v 22
h e = 0.5
2g

3) Loss of head at entrance of a pipe:

© Copyright Reserved by Gateflix.in No part of this material should be copied or reproduced without permission
v2 Q=A2 v2= 0.03275m3/s = 32.75lit/s
h i = 0.5
2g
7.5 FLOW THROUGH PIPES IN SERIES OR
FLOW THROUGH COMPOUND PIPES
4) Loss of head due to exit:
v2 7.5.1 EQUIVALENT PIPE
ho =
2g
This is defined as the pipe of uniform
5) Loss of Head due to an obstruction in diameter having loss of head and discharge
a pipe: Due to sudden enlargement of equal to the loss of head and discharge of a
the area of flow beyond the obstruction, compound pipe consisting of several pipes
head loss takes place. of different lengths and diameters.
After vena contracta, sudden
enlargement takes place.
a =max area of obstruction
A = area of pipe
V = velocity of liquid. If no minor loss is assumed
2
v 
2
A  H = ZA − ZB = hf
Head loss =  − 1 total

2g  cc (A − a)  4f L V 2
4f L V 2 4f L V 2
H= 1 1 1
+ 2 2 2 + 3 3 3
2gd1 2gd2 2gd3
6) Loss of head due to bend in pipe: πd2 πd22 πd32
=Q = .V1 = V2 V
k.v 2 4 4 4 3
hb = For friction coefficient,
2g
The value of ‘k’ depends on 4 × 16fQ2  L1 L2 L3 
=H  + + 
• Angle of bend π2 .2g  d15 d25 d35 
• Radius of curvature of bend
• Diameter of pipe 4fLV12 4 × 16Q2f  Leq 
=H =  
2g d π2 2g  deq5 
Example: L
Leq L L 
At a sudden enlargement of a water 5
=  15 + 25 + 35 
main from 240 mm to 480 mm dia the deq  d1 d2 d3 
hydraulic gradient rises by 10 mm.
Estimate the rate of flow. 7.5.2 POWER TRANSMISSION
Solution: THROUGH PIPES
Hydraulic Gradient
 P1   P2  Total head available at outlet of pipe
 + z1  −  + z 2  = H − hf (Minor loss neglected)
 ρg   ρg 
4f lv 2
v2 ( v − v )
2
P1 v2 P = H−
+ z1 + 1 = 2 + z 2 + 2 + 1 2 d.2g
ρg 2g ρg 2g 2g Weight of water flowing
πd2
Q= .v
d v = d v2
2
1 1
2
2 4
d1 = 240mm πd2
= = ρg
W ρgQ .v
4
d2 = 480mm
Power transmitted
v1=4v2
= W × head available at outlet
v2=0.1808=0.181m/s

© Copyright Reserved by Gateflix.in No part of this material should be copied or reproduced without permission
 4f lv 2  ρg πd2 Viscosity, µ = 0.1Ns/ m2
=
P H −  v
 2gd  4 Relative density = 0.9
density of oil = 0.9×1000= 900kg/ m3
7.6 FLOWS THROUGH NOZZLES D =50mm = .05m
L =300m
Total head at inlet of pipe
= total head energy + losses 3.5
Q =3.5liters/ S = = .0035m3 / s
1000
v2
But, total head at outlet of nozzle = (i) Pressure drop
2g
By Bernoulli’s equation for real fluids 32µuL Q .0035 .0035
Assuming minor loss to be negligible p1 − p2 = , where u = = = =1.782m/ s
D2 Area π 2 π
(.05)
2
D
v2 4 4
H − hf =
2g The Reynolds number (R e ) is given by
4f lv 2 ρ VD
Re =
hf = µ
2gd
Where
By continuity equation
AV = av ρ= 900kg / m3 , averagevelocity= u= 1.782m / s
2 ∴ 1.782×.05
v 2 4f L  a  R e = 900× = 801.9
=
H + 0.1
 v
2g 2gD  A  As Reynolds number is less than 2000, the
2gh flow is viscous/laminar
v=
 4f L a2  32 × 0.1×1.782 × 300
1 + × 2 = ∴ p1 − p 2 = 2
684288N / mm 2
 D A  0.05
= 68.43N / m 2

7.6.1 POWER TRANSMITTED THROUGH ii) Shear Stress at the pipe wall ( τ0 )
NOZZLE The shear stress at any radius r is given by
the equation
1 −δ p r
K.E of the jet at the outlet of Nozzle = mv 2
i.e., τ=
2 δx 2
Mass flow rate = ρav ∴ Shear stress at pipe wall, where r=R is
1 given by
K.E.= ρav3
2 −δ p R
τ=
power of outlet of Nozzle 1 / 2ρav3 δx 2
η =
Power at the inlet of pipe ρg Q.H δp −(p 2 − p1 ) p1 − p 2 p1 − p 2
Now= = =
δx x 2 − x1 x 2 − x1 L
Example: 2
684288 N/ m
An oil of viscosity 0.1Ns/ m2 and relative = = 2280.96N/ m3
300 m
density 0.9 is flowing through a circular And R = D = .05 = .025m
pipe of diameter 50mm and of length 2 2
300m. The rate of flow of fluid through the .025 N
τ0 = 2280.96× = 28.512N/ m2
pipe is 3.5 lit/s. Find the pressure drop in a 2 m 2

length of 300m and also the shear stress at


the pipe wall. Example:
Solution: A laminar flow is taking place in a pipe of
Given: diameter 200mm. The maximum velocity is

© Copyright Reserved by Gateflix.in No part of this material should be copied or reproduced without permission
1.5m/s. Find the mean velocity and the
radius at which this occurs. Also calculate
the velocity at 4cm from the wall of the
pipe.
Solution:
Given:
Dia of pipe, D=200mm=0.20m   r 2    .06 2 
= Umax 1 −    =1.5 1 −   
Umax =1.5m/ s   R     .1  
(i)Mean velocity , u =1.5 1.0 − .36 =1.5×.64 = 0.96m/ s
Ratio of
Umax 1.5 1.5
= 2.0 or = 2.0 ∴ u = = 0.75m/ s Example:
u u 2.0 Crude oil of µ =1.5 poise and relative
(ii)Radius at which 𝐮𝐮 occurs
density 0.9 flows through a 20mm
The velocity, u, at any radius ‘r’ is given by
diameter vertical pipe. The pressure gauges
1 δp 2 2 1 δp 2  r2  fixed 20m apart read 58.86N/cm2 and
u=− R − r  = − R 1 − 
4µ δ x   4µ δµ  R 2  19.62N/cm2 as shown in Fig. Find the
But from equation Umax is given by direction and rate of flow through the pipe.

1 δp 2   r 2 
Umax = − R ∴ u = Umax 1 −    .....(i)
4µ δ x   R  
Now the radius r at which
u = u = 0.75m/ s
   
2

∴ 0.75 = 1.5 1 −   
r
Solution:
 D  
Given:
  2  
1.5
  r 2  µ =1.5poise = = 0.15Ns/ m2
∴ 0.75 = 1.5 1 −    10
  0.1   Relative density = 0.9
2 ∴ Density of oil
0.75  r 
∴ =
1−   ∴
1.5  0.1  = 0.9×1000= 900kg/ m3
2 Dia. Of pipe, D= 20mm = 0.02m L = 20m
 r  0.75 1 1
∴  =1 − =1 − = PA =58.86N/ cm2 =58.86×104 N/ m2
 0.1  1.50 2 2
PB =19.62N/ cm2 =19.62×104 N/ m2
r 1 Solution:
∴ = = 0.5 (i)Direction of flow.
0.1 2
To find the direction of flow,
∴ r = 0.1× .5 = 0.1×707 = .0707m  p v2 
=70.7mm the total energy  + +Z  at the lower
 ρ g 2g 
(iii) Velocity at 4cm from the wall end A and at the upper end B is to be
∴ r = R − 4.0=10 − 4.0=6.0cm =0.06m calculated. The direction of flow will be
given from the higher energy to the lower
The velocity at a radius =0.06m Or 4cm
energy. As here the diameter of the pipe is
from pipe wall is given by equation(i)
same and hence kinetic energy at A and B

© Copyright Reserved by Gateflix.in No part of this material should be copied or reproduced without permission
will be same. Hence to find the direction of As Reynolds number is less than 2000, the
 p  flow is laminar.
flow, calculating  +Z  at A and B. ∴ Rate of flow = avg. velocity * area
 ρg 
π π
= u× D2 = 0.9× × ( .02) m3 / s = 2.827×10−4 m3 / s
2

4 4
Taking the level at A as datum.
p  = 0.2827lit res/ s
The value of  A +Z  at A
 ρg 
Example:
6 ×10 × 9.81
4
A shaft having a diameter of 50mm rotates
= = + 0 66.67m
900 × 9.81 centrally in a journal bearing having a
diameter of 50.15mm and length 100mm.
 p  The annular space between the shaft and
The value of  +Z  at B the bearing is filled with oil having
 ρg  viscosity of 0.9 poise. Determine the power
2×104 ×9.81 absorbed in the bearing when the speed of
= +20 = 22.22+20 = 42.22m
900×9.81 rotation is 60 rpm.
 p  Solution:
As the value of  +Z  is higher at A. Given:
 ρg  Dia. of shaft, D=50mm or .05m
Hence, flow takes place from A to B. Dia of bearing D1 = 50.15mm or 0.05015m
Length , L =100mm or 0.1m
(ii) Rate of flow. The loss of pressure head
0.9 Ns
for viscous flow through circular pipe is µ of oil = 0.9 poise =
given by 10 m2
N = 600r.p.m.
32µuL
hf = Power = ?
ρ gD2 D1 − D 50.15 − 50
∴ Thickness of oil film, t = =
For a vertical pipe hf = Loss of 2 2
peizometric head 0.15
= = 0.075mm = 0.075×10−3 m
p  p  2
=  A +ZA  −  B +ZB  = 66.67 − 42.22= 24.45m
 ρg   ρg 
Tangential speed of shaft,
32×0.15×u×20.0 π DN π ×0.05×600
∴ 24.45= V= = = 0.5× π m/ s
900×9.81× ( .02)
2
60 60
Or ∴ Shear stress
24.45×900×9.81×.0004 du V 0.9 0.5× π
u= = 0.889 ; 0.9m/ s τ=µ =µ = × =1883.52N/ m2
32×0.15×20.0 dy t 10 0.075×10 −3

The Reynolds number should be calculated. ∴ Shear force (F)


If Reynolds number is less than 2000, the = τ × Area =1883.52× π D×L
flow will be laminar and the above =1883.52× π ×.05×0.1 = 29.586N
expression for loss of pressure head for
laminar flow can be used. Resistance torque
ρ VD D .05
Now Reynolds number = T = F× = 29.586× = 0.7387Nm
µ 2 2
0.9×.02 2π N 2π NT
= 900× =108 Power, P = T× ω = T× = watts
0.15 60 60

© Copyright Reserved by Gateflix.in No part of this material should be copied or reproduced without permission
2π NT 2π ×600×0.7387 number. The kinematic viscosity of water =
Power = = = 46.41W. 0.1 Stokes.
60 60
Solution:
Example: Given:
An oil of S.G. 0.7 is flowing through a pipe Dia. of pipe, d = 200mm = 0.20m    
of diameter 300 mm at the rate of 500 Velocity, V = 3m/ s
liters/s. Find the head lost due to friction Length, L =5 m
and power required to maintain the flow Kinematics viscosity,
for a length of 1000m. Take ν = .29 stokes. = = .01 × 10−4 m2 / s
v 0.01stoke
Solution:
Reynolds number,
Given:
vd 3 × 0.2
S.G. of oil S = 0.7 Re= = −4
= 6 ×105
Dia. of pipe d =300mm = 0.3m ν 0.01×10
Value of f,
Discharge Q = 500 litres/ s = 0.5m3 / s
0.09 0.09
Length of pipe L =1000m f =0.02 + 0.3 =0.02 +
( 6 ×105 )
0.3
Re
Velocity,
Q 0.5 0.5 × 4 0.09
=V = = = 7.073m / s =0.02 + =0.02166
Area π 2 π× 0.32 54.13
d
4
Reynolds number, Head lost due to friction:
vd 7.073 × 0.3 4 × f × L × V 2 4.0 × 0.02166 × 5.0 × 32
= =
Re = 7.316 ×104 =hf = = 0.993 m of water
ν 0.29 ×10−4 d × 2g 0.20 × 2.0 × 9.81

∴ Co-efficient of friction, Example:


0.79 0.79 Determine the rate of flow of water
f= 1 = = 0.0048
( )
1
Re 4
7.316×10 4 4 through a pipe of diameter 20 cm and
length 50 m when one end of the pipe is
∴ Head lost due to friction, connected to a tank and other end of the
4 × f × L × V 2 4 × 0.0048 × 1000 × 7.0732 pipe is open to the atmosphere. The pipe is
=hf = = 163.18m
d × 2g 0.3 × 2 × 9.81 horizontal and height of water in the tank is
Power required 4 m above the centre of the pipe. Consider
ρg . Q . hf all minor losses and take f = .099 in the
= kW 4 × f × L × V2
1000 formula hf = .
Where d × 2g
=
ρ density of oil = 0.7 × 1000 = 700kg / m3
∴ Power required
700 × 9.81 × 0.5 × 163.18
= 560.28kW
1000

Example:
Water is flowing through a pipe of diameter
200mm with a velocity of 3m/s. Find the Solution:
head lost due to friction for a length of 5m Given
if the co-efficient of friction is given by Dia. of pipe = = 0.20m
d 20cm
.09 Length of pipe, L = 50m
=f 0.02 + 0.3 where R e is Reynolds Height of water
Re H = 4m

© Copyright Reserved by Gateflix.in No part of this material should be copied or reproduced without permission
Co-efficient of friction f = .009 elevation of 15m. The total length of the
syphon is 600m and the summit is 4 m
Let the velocity of water in pipe = V m / s above the water level in the upper
Applying Bernoulli’s equation at the top of reservoir. If the separation takes place at
the water surface in the tank and at the 2.8m of water absolute, find the maximum
outlet of pipe, we have[Taking point 1 on length of syphon from upper reservoir to
the top and 2 at the outlet of pipe]. the summit. Take f=0.004 and atmospheric
pressure =10.3m of water.
P1 V12 P2 V22
+ + z1 = + + z2 + all losses Solution:
ρg 2g ρg 2g Given
Consider datum line passing through the =
Dia. of syphon = 0.2m
d 200m
centre of pipe Difference of level in two reservoirs = 15m
V2 Total length of pipe = 600m
0 + 0 + 4.0 = 0 + 2 + 0 + (hi + hf )
2g Height of summit from upper reservoirs
V22 = 4m
Or 4.0 = + hi + hf Pressure head at summit
2g
pc
But the velocity in pipe= V , = 2.8m of water absolute
ρg
∴V =V2
Atmospheric pressure head
V2 pc
4.0 =
+ hi + hf = 10.3m of water absolute
2g ρg
From equation Co-efficient of friction, f = .004
V2 4 × f × L × V2
= =
hi 0.5 and hf
2g d × 2g
Substituting these values, we have
V2 V2 4 × f × L × V2
4.0 = + 0.5 +
2g 2g d × 2g
v2  4 × 0.009 × 50 
=
∴4 1.0 + 0.5 +  Applying Bernoulli’s equation to points A
2g  0.2  and C and taking the datum line passing
2
v through, A and C
=
∴4 1.0 + 0.5 + 9.0
2g pA VA 2 pC VC2
+ + z = + + z + Losses of head
v2 ρg 2g A ρg 2g C
∴4 = 10.5
2g due to friction between A and C
Substituting the values of pressures in
terms of absolute, we have
4 × 2 × 9.81
=∴ V = 2.734 m / sec V2
10.5 10.3 + 0 + 0= 2.8 + + 4.0 + hf 1
∴ Rate of flow, 2g
π = =
[Q VC velocity in pipe V]
Q = A × V = × (0.2)2 × 2.734 = 0.08589 m3 / s
4 V2 V2
= 85.89 litres / s. ∴ h f1
= 10.3 − 2.8 − 4.0 − = 3.5 − ..... ( i )
2g 2g

Example: Applying Bernoulli’s equation to points A


A syphon of diameter 200 mm connects and B and taking the datum line passing
two reservoirs having a difference in through B,

© Copyright Reserved by Gateflix.in No part of this material should be copied or reproduced without permission
Difference of water levels, H = 16m
pA VA 2 p V2
+ + z A = B + B + zB ++ Losses of head Length & dia. of pipe1,
ρg 2g ρg 2g L1 = 400m and d1 = 400 mm = 0.4 m
due to friction between A and B Length & dia of pipe 2
But , p=
A
pB
= atmospheric pressure
L2 = 200m and d2 = 200m = 0.2m
ρg ρg Length & dia of pipe 3
And,=VA 0,= VB 0,= z A 15,=
zB 0 L3 = 200m and d3 = 300m = 0.3m
∴ 0 + 0 + 15 = 0 + 0 + 0 + hf Also, f1= f2= f3= 0.005
(i) Discharge through the compound
4 × f × L × V2
hf =
15 or 15 pipe first neglecting minor losses.
d × 2g Let V1 , V2 , V3 are the velocities in the
4 × .004 × 600 × V 2
Or = 15 1st 2nd and 3rd pipe respectively.
0.2 × 2 × 9.81
From continuity, we have
Or V = 15 × 0.2 × 2 × 9.81
= 2.47m / s
4 × .004 × 600
A=V A=
1 1
V A3V3
2 2

Substituting this value of V in equation ( i ) π 2


A1 V1 4 d1
2
d12  0.4 
we get ∴ V= = × V= V=   V= 4V1
2
A2 π 2 1 d22 1  0.2  1
hf 1 =
3.5 −
2.472
2 × 9.81
=
3.5 − 0.311 = ( )
3.189m ..... ii
d
4 2
But And
4 × f × L1 × V 2 π 2
hf 2 = .... ( iii ) A1 V1 4 d1 d12  0.4 
2

d × 2g V= = × V= V=   V= 1.77V1
Where L1 =inlet leg of syphon or length of
3
A3 π 2 1 d32 1  0.2  1
d
4 3
syphon from upper reservoir to the
Now using equation of
summit.
4 × .004 × L1 × (2.47)2
4 × f1 × L1 × V12 4 × f2 × L2 × V22 4 × f3 × L3 × V32
=
hf 1 = 0.0248 × L1 H= + +
0.2 × 2 × 9.81 d1 × 2g d2 × 2g d3 × 2g
Substituting this value in equation ( ii ) , 4 × 0.005 × 400 × V12 4 × 0.005 × 200 × ( 4V1 )
2

=16 +
0.0248L1 =  3.189  0.4 × 2 × 9.81 0.2 × 2 × 9.81

3.189
=
∴ L1 = 128.58m 4 × 0.005 × 00 × (1.77V1 )
2

0.0248 +
0.3 × 2 × 9.81
2
Example: V1 V12
=
16 (20 + 320 + 63.14)
= × 403.14
Three pipes of 400 mm, 200 mm and 300 2 × 9.81 2 × 9.81
mm diameters have lengths of 400m, 200m 16 × 2 × 9.81
300m respectively. They are connected = in V1 = 0.882m / s
403.14
series to make a compound pipe. The ends
of this compound pipe are connected with ∴ Discharge,
two tanks whose difference of water levels π
Q = A1 × V1 = (0.4)2 × 0.882 = 0.1108 m3 / s
is 16m. If co-efficient of friction for these 4
pipes is same and equal to 0.005, (ii) Discharge through the compound
determine the discharge through the pipe considering minor losses also
compound pipe neglecting first the minor
Minor losses are:
losses and then considering them.
Solution: 0.5V12
hi =
Given: (a) At inlet, 2g

© Copyright Reserved by Gateflix.in No part of this material should be copied or reproduced without permission
(b) Between 1st and 2nd pipe, due to Two pipes have a length L each. One of
contraction, them has diameter D, and the other has
0.5V22 0.5(4V1 )2
diameter d. If the pipes are arranged in
=hC = =
(Q V2 4V1 ) parallel, the loss of head, when a total
2g 2g
quantity of water Q flows through them is
0.5 × 16 × V12 V2 h, but, if the pipes are arranged in series
= = 8× 1
2g 2g and the same quantity Q flows through
(c) Between 2nd and 3rd pipe, due to sudden D
them, the loss of head is H. If d = , find the
enlargement, 2
0.5V12 ratio of H to h, neglecting secondary losses
hi = and assuming the pipe co-efficient has a
2g constant value.
=he
(V2 − V3 )2 (4V1 − 1.77V1 )2
= (Q V3 = 1.77V1 ) Solution:
2g 2g Given:
V2 V2 Length & dia. of pipe1
= (2.23)2 × 1= 4.973 1
2g 2g L1 = L and d1 = D
(d)At the outlet of 3rd pipe, Length & dia. of pipe2
V 2 (1.77V1 )2 V2 V2 L2 = L and d2 = d
ho = 3 = = 1.772 × 1 = 3.1329 1
2g 2g 2g 2g
Total Discharge =Q
The major losses are
Head loss when pipes are arranged in
4 × f1 × L1 × V12 4 × f2 × L2 × V22 4 × f3 × L3 × V32
= + + parallel = h
d1 × 2g d2 × 2g d3 × 2g
Head loss when pipes are arranged in
4 × 0.005 × 400 × V12 4 × 0.005 × 200 × (4V1 )2 series = H
+
0.4 × 2 × 9.81 0.2 × 2 × 9.81 D
4 × 0.005 × 300 × (1.77V1 )2 d= and f is constant
+ 2
0.3 × 2 × 9.81 1st case:
V12 When pipes are connected parallel
= 403.14 ×
2 × 9.81 Q = Q1 +Q2 …(i)
∴ Sum of minor losses and major losses Loss od head in each pipe =h
 0.5V12 V2 V2 V2 V2
=  + 8 × 1 + 4.973 1 + 3.1329 1  + 403.14 1 4fL1 V12
 2g 2g 2g 2g  2g For pipe AB, = h,
d1 ×2g
V12
= 419.746
2g Q1 Q 4Q
where V1 = = 1 = 12
But total loss must equal to H (or 16m) A1 π 2 π D
D
V2 4
∴ 419.746 × 1 = 16
2g d1 = D
16 × 2 × 9.81
=∴ V1 = 0.864m / s
419.746
∴ Discharge
p
Q = A1 V1 = ×(0.4)2 ×0.864 = 0.1085m3 / s  4Q 
4 4fL  12  2
 π D  = h or 32fLQ1 = h
2

Example: ∴ .....(ii)
D× 2g π2 D5×g

© Copyright Reserved by Gateflix.in No part of this material should be copied or reproduced without permission
32fLQ 22 h h Q2 Q2  Q2 Q2 
For pipe AC, =h H = Q2 × 2
+ Q2 × 2 = 2 h + 2 h = h  2 + 2 
π2d5 × g Q1 Q 2 Q1 Q2  Q1 Q2 
32fLQ12 32fLQ22 Q15 Q25 H Q2 Q2
∴ = or = = +
π2D5 × g π2d5 × g D5 d5 h Q12 Q22

 Q1  D5 (2d )
5
2
But from equations (iv) and (v)
or   = 5 = 5 = 2 = 32
5
∴D = 2d  , Q1 = .85Q and Q 2 = 0.15Q
 Q2  d d
H Q2 Q2 1 1
Q1 = 2 2+ 2 2= 2+ 2
∴ = 32 =5.657 or Q1 =5.657Q2 h .85 Q .15 Q .85 .15
Q2
Substituting the value of Q1 in equation (i), =1.384 + 44.444 =45.828

we get
Q =5.657Q2 +Q2 =6.657Q2
Q
∴Q 2 = =0.15Q .....(iv
6.657
From (i)
∴ Q1 = Q − Q2 = Q-0.15Q = 0.85Q .....(v
2nd case:
When pipes are connected in series
Total loss = sum of head losses on two
pipes
4f .L.V12 4f .L.V22
=H +
d1 × 2g d2 × 2g
Q 4Q Q 4Q
where=
V1 = ,=
V2 =
π 2 πD2 π 2 πd2
D d
4 4

2 2
 4Q   4Q 
4f.L.  2  4f.L.  2 
 πD  +  πd 
∴=H
D × 2g d × 2g
32fLQ 2 32fLQ 2
=
or H + 5 2 ....(vi)
D π ×g d π ×g
5 2

32fl h
From equation(ii), = 2
π D ×g Q1
52

32fl h
and frome quation (iii), = 2
π d ×g Q 2
2 5

Substituting these values in equation (vi),


where

© Copyright Reserved by Gateflix.in No part of this material should be copied or reproduced without permission
GATE QUESTIONS

Q.1 For a fluid flow through a divergent pipe at sectionS1 (elevation: 10 m)


pipe of length L having inlet and is 50 kPa. At sectionS2 (elevation:
outlet radii of R1 and R2 respectively 12m) the pressure is 20 kPa and
and a constant flow rate of Q, velocity is 2 m/s. Density of water
assuming the velocity to be axial and is 1000 kg/m3 and acceleration due
uniform at any cross-section, the to gravity is 9.8 m/s2. Which of the
acceleration at the exit is following is TRUE
2Q(R1 − R 2 ) 2Q 2 (R1 − R 2 ) a) flow is fromS1 to S2 and head
a) b) loss is 0.53 m
πLR 32 πLR 32
b) flow is fromS2 to S1 and head
2Q 2 (R1 − R 2 ) 2Q 2 (R 2 − R1 ) loss is 0.53 m
c) d)
π 2 LR 52 π 2 LR 52 c) flow is fromS1 to S2 and head
[GATE–2004] loss is1.06 m
d) flow is fromS2 to S1 and head
Q.2 A siphon draws water from a loss is 1.06 m
reservoir and discharges it out at [GATE–2010]
atmospheric pressure. Assuming ideal
fluid & the reservoir is large, the Q.5 Oil flows through a 200 mm
velocity at point P in the siphon tube is diameter horizontal cast iron pipe
(friction factor, f = 0.0225) of length
500 m. The volumetric flow rate is
0.2 m3/s. The head loss (in m) due
to friction is (assume g = 9.81 m/𝑠𝑠 2 )
a) 116.18 b) 0.116
c) 18.22 d) 232.36
[GATE–2013]
a) 2gh1 b) 2gh 2 Q.6 For steady, fully developed flow
inside a straight pipe of diameter D,
c) 2g ( h 2 − h1 ) d 2g ( h 2 + h1 ) neglecting gravity effects, the
[GATE–2006] pressure drop ∆p over a length L
and the wall shear stress τW are
Q.3 Water at 25℃ is flowing through a
related by
1.0 km long C.I. pipe of 200 mm
∆pD ∆pD 2
diameter at the rate of 0.07 m3/s. If a)τw= b)τw=
value of Darcy friction factor for this 4L 4L2
pipe is 0.02 and density of water is ∆pD 4∆pL
c)τw= d)τw=
1000 kg/m3 , the pumping power (in 2L D
kW) required to maintain the flow is [GATE–2013]
a) 1.8 b) 17.4
Q.7 Water flows through a 10 mm
c) 20.5 d) 41.0
diameter and 250 m long smooth
[GATE–2009]
pipe at an average velocity of 0.1
Q.4 A smooth pipe of diameter 200 mm m/s. The density and the viscosity of
carries water. The pressure in the water are 997 kg/m3 and 855 ×

© Copyright Reserved by Gateflix.in No part of this material should be copied or reproduced without permission
10−6 Ns/m2, respectively. Assuming
fully-developed flow, the pressure Q.11 Three parallel pipes connected at
drop (in Pa) in the pipe is ____. the two ends have flow-rates Q1, Q2
[GATE–2014(2)] and Q3 respectively, and the
Q.8 A fluid of dynamic viscosity 2 × corresponding frictional head losses
10−5 kg/ms and density 1 kg/m3 are hL1, hL2 and hL3 respectively. The
flows with an average velocity of 1 correct expressions for total flow
m/s through a long duct of rate (Q) and frictional head loss
rectangular (25mmx 15 mm) cross- across the two ends (hL) are
section. Assuming laminar flow, the a) Q= Q1+ Q2 + Q3; hL = hL1+ hL2+ hL3
pressure drop (in Pa) in the fully b) Q = Q1+ Q2 + Q3; hL= hL1= hL2= hL3
developed region per meter length c) Q= Q1= Q2 = Q3; hL = hL1+ hL2+ hL3
of the duct is _____. d) Q = Q1 = Q3; hL = hL1= hL2= hL3
[GATE–2014(3)] [GATE–2015(3)]

Q.9 A siphon is used to drain water from Q.12 The head loss for a laminar
a large tank as shown in figure incompressible flow through a
below. Assume that the level of horizontal circular pipe is h1. Pipe
water is maintained constant. Ignore length and fluid remaining the same,
frictional effect due to viscosity and if the average flow velocity doubles
losses at entry and exit. At the exit of and the pipe diameter reduces to
the siphon, the velocity of water is half its previous value, the head loss
is h2. The ratio h2/h1 is
a) 1 b) 4
c) 8 d) 16
[GATE–2015(2)]

Q.13 A channel of width 450 mm


branches into two sub-channels
having width 300 mm and 200 mm
as shown in figure. If the volumetric
flow rate (taking unit depth) of an
incompressible flow through the
a) 2g(ZQ − ZR ) b) 2g(ZP − ZR )
main channel is 0.9 m3/s and the
c) 2g(ZO − ZR ) d) 2gZQ velocity in the sub-channel of width
200 mm is 3 m/s, the velocity in the
[GATE–2014(3)]
sub-channel of width 300 mm is
Q.10 Consider fully developed flow in a _____________ m/s. Assume both inlet
circular pipe with negligible and outlet to be at the same
entrance length effects. Assuming elevation.
the mass flow rate, density and
friction factor to be constant, if the
length of the pipe is doubled and the
diameter is halved, the head loss
due to friction will increase by a
factor of
a) 4 b) 16
c) 32 d) 64
[GATE–2015(1)]

© Copyright Reserved by Gateflix.in No part of this material should be copied or reproduced without permission
[GATE–2016(3)] Q.16 Water (density =1000 kg/m3) at ambient
temperature flows through a
Q.14 For steady flow of a viscous horizontal pipe of uniform cross
incompressible fluid through a circular section at the rate of 1 kg/s. If the
pipe of constant diameter, the pressure drop across the pipe is 100
average velocity in the fully developed kPa, the minimum power required to
region is constant. Which one of the pump the water across the pipe, in watts,
following statements about the is _______
average velocity in the developing [GATE–2017(1)]
region is TRUE?
a) It increases until the flow is fully
developed.
b) It is constant and is equal to the
average velocity in the fully developed
region.
c) It decreases until the flow is fully
developed.
d) It is constant but always lower than
the average velocity in the fully
developed region.

[GATE–2017(1)]

Q.15 Consider steady flow of an


incompressible fluid through two long
and straight pipes of diameters d1 and
d2 arranged in series. Both pipes are
of equal length and the flow is
turbulent in both pipes. The friction
factor for turbulent flow though pipes
is of the form, f =K ( Re )
−n
where K
and n are known positive constants
and Re is the Reynolds number.
Neglecting minor losses, the ratio of
the frictional pressure drop in pipe 1
to that in pipe 2, is given by
( 5− n ) 5
d  d 
(A)  2  (B)  2 
 d1   d1 
( 3− n ) ( 5+ n )
d  d 
(C)  2  (D)  2 
 d1   d1 

[GATE–2017(1)]

© Copyright Reserved by Gateflix.in No part of this material should be copied or reproduced without permission
ANSWER KEY:
1 2 3 4 5 6 7 8 9 10 11 12 13
(c) (c) (a) (c) (a) (a) 6840 1.82 (b) (d) (b) (c) 1
14 15 16
(b) (a) 100

EXPLANATIONS

Q.1 (c) ∂u
ax = u
∂u ∂u ∂u ∂u ∂x
ax = u +v +w +
∂x ∂y ∂z ∂t ∂u
a x =L = u
∂u ∂x x = L
∴ ax = u
∂x and also replacing a and b
A×u =Q
Q −2Q 2 (R 2 − R1 )
u= a x =L =
A π 2 LR 52
Q
u=
πR 2 Q.2 (c)

R 2 − R1
= R1 +
R .x In a steady and ideal flow of
L
R − R1 incompressible fluid, the total
=
Let. R1 a= and 2 b energy at any point of the fluid is
L
constant. So applying the Bernoulli’s
∴R = a+bx
Equation at section (1) and (2)
Q
u= P1 V12 P2 V22
π(a + bx) 2 + + Z1 = + + Z2
ρg 2g ρg 2g
∂u −2bQ
= V1= 0 = Initial velocity at point (1)
∂x π(a + bx)3 Z2 = 0 = At the bottom surface
P1= P2= Patm

© Copyright Reserved by Gateflix.in No part of this material should be copied or reproduced without permission
And Z1= h2−h1 P1 P2
= + Z1 + Z2 + h L
V12 ρg ρg
= 2g ( h1 − h 2 )
2g P −P 
=h 2  1 2  + ( z1 − z 2 )
V2  pg 
So, h 2 − h1 =1
2g ( 50 − 20 ) ×102 + 10 − 12
= ( )
=V2 2g ( h 2 − h1 ) (100 × 9.8)
Since velocity of fluid is same inside = 3.058 − 2 =1.06m = +ve
the tube Thus flow is from S1 to S2 and head
V= V= 2g ( h 2 − h1 ) loss is 1.06 m.
p 2

Q.5 (a)
Q.3 (a) From Darcy Weisbach equation
Given: L = 1 km = 1000 m, head loss
D =200 mm = 0.2m, Q = 0.07 m3/ s fLV 2
F = 0.02, ρ= 1000 kg/m3 hf =
Head loss is given by, D × 2g
FLV 2 8FLQ 2 200
=hf = Given that L= 500 m, D = =
D × 2g π 2 D5g 1000
0.2m.
8 × 0.02 ×1000 × 0.07 2 f=0.0225
hf = = 25.30 m
π 2 × 0.25 × 9.81 Since volumetric flow rate is given
Pumping power required. by
P = ρgQhf Q = Area × velocity
= 1000×9.81×0.07×25.3 0.2
=17373.51 = 17.4 kW V= = 6.366
π / 4 × 0.22
0.0225 × 500 × 6.3662
Q.4 (c) Hence, h f =
Given: p1= 50 kPa, Z1= 10 m, 0.2 × 2 × 9.81
p2= 20 kPa, Z2= 12 m,V2 =2 m/ sec, hf = 116.18 m
ρ= 1000 kg/m3, g = 9.81 m/ sec2
Q.6 (a)
For steady, fully developed flow
inside a straight pipe, the pressure
drop and wall shear stress are
∆pD
related by τw=
4L

Applying continuity equation at Q.7 (6840)


section S1 and S2, 32μuL
P1 − P2 = 2
A1 V1= A2V2 D
V1= V2, D1= D2 so A1= A2... (i) 32 × 855 ×10−6 × 0.1× 250
=
Applying Bernoulli’s equation at (0.01) 2
section S1 and S2 with head loss hL,
∴ P1 − P2 = 6840Pa
P1 V12 P2 V22
= + + Z1 + + Z2 + h L
ρg 2g ρg 2g Q.8 (1.82)
As area is constant, velocity head 32μuL
will be same ∆P = 2
D

© Copyright Reserved by Gateflix.in No part of this material should be copied or reproduced without permission
∆P 32μu h2
∴ = =8
L D2 h1
4A 4(25 ×15)
D= =
P 2[25 + 15] Q.13 (1)
4 × 25 ×15 Let the velocity in the sub-channel
= = 18.75mm of width 300mm be V.
80
∴ 0.9 = (0.3V) + (0.2×3)
∆P 32 × 2 ×10−5 ×1
∴ = ∴ 0.3V= 0.3
L (18.75 ×10−3 ) 2 V= 1 m/s
= 1.82 Pa/m Q.14 (b)
Q
Q.9 (b) Vavg =
A
Applying Bernoulli’s equation As given steady flow means Q →
between P and R: constant
V2 V2 Answer also given constant diameter
Patm + P + ZP = R + ZR + Patm
2g 2g pipe, therefore area also constant.
VR2 ∴ Vavg = Cons tan t
Or = (ZP — ZR )
2g
(∵VP = 0for still water) Q.15 (a)
k k
∴ VR = 2g(ZP — ZR ) =f = n n
⇒ f ∝ Dn
(Re)  
 ρ.Q.D 
Q.10 (d) π 
Q
2  D 2 .µ 
2 FL   4 
Head loss =
FLV
= A ∆P flQ 2
8
2gd 2gd Q= × 2
ρg gD π 5

L
⇒ head loss ∝ 5 f
d ∆P ∝ 5
D
L
h1* ∝ 5 ∆P ∝ D n −5
d n −5
∆P  D1 
h *2 ∝
2.L

L
64 ∴ 1 = 
∆P2  D 2 
( )
5
d d5
2 5− n
 ∆P1   D 2 
h1 1 h or  = =  
= ⇒ 2 = 64
h 2 64 h1  ∆P2   D1 

Q.11 (b) Q.16 (100) Q power= Q × ∆P


Total flow rateQ = Q1+ Q2 + Q3 =
m
× ∆P
Head loss =h h= L1 h=
L2 h L3 ρ
1×100 ×103
Q.12 (c) = = 100 watt
1000
u avg
Head loss, h ∝
D2
2
h 2  D1  u avg 2
∴ =   = 22 × 2
h1  D 2  u avg1

© Copyright Reserved by Gateflix.in No part of this material should be copied or reproduced without permission
8 EXTERNAL FLOW

8.1 BOUNDARY LAYER FORMATION


The variation of velocity from zero to free
stream velocity in the direction normal to
the boundary takes place in a narrow
region in the vicinity of solid boundary.
Boundary layer is a very thin layer of the
fluid, in the immediate neighbourhood of
the solid boundaries where the variation of
velocity is from zero to free stream
velocity. 8.2.1 LAMINAR BOUNDARY LAYER

In laminar region the fluid flows in


streamline. The viscous force is higher than
inertia force.
The Reynold’s No. for flat plate is given by
du U x
The velocity gradient = Re x  
dy 
Fluid exerts a shear stress on the wall in Where,
the direction of motion. The value of shear U ∞ = free stream velocity
µdu X = distance from leading edge
stress is given by τ =
dy ν  = kinematic viscosity of fluid
The velocity gradient is set up in the fluid
near the surface of the plate. This velocity For laminar flow
gradient develops shear resistance which R ex < 5 × 105
retards the fluid. Downstream the leading
edge, the boundary layer region increases 8.2.2 TURBULENT BOUNDARY LAYER
because the retarded fluid is further
retarded. When the length of plate is more than the
8.2 REGIONS OF BOUNDARY LAYER critical length ‘x cr ’ , calculated from
 Re 5105 
Boundary layer can be divided in three equation  
 
major regions: U
1. Laminar then, transition from laminar to turbulent
2. Transition takes place.
3. Turbulent
4. Laminar sub-layer 8.2.3 LAMINAR SUBLAYER

It is region in the turbulent zone adjacent


to the solid surface of the plate the velocity
variation is influenced only by viscous
effects. Though velocity distribution is

© Copyright Reserved by Gateflix.in No part of this material should be copied or reproduced without permission
parabolic for small thickness, we can
reasonably assume that velocity variation 8.3.3 ENERGY THICKNESS
is linear here
 du   ∆u  It is defined as the distance measured
τo = µ   ⇒ µ   normal to the boundary of the solid body,
 dy   ∆y  by which the boundary should be displaced
τo is constant in laminar sub-layer. to compensate for the reduction in K.E. of
the flowing fluid.
8.3 BOUNDARY LAYER THICKNESS  u  2 
 **   1 u dy
0 U  U 2 
It is defined as the distance from the  

boundary of the solid measured in the y-


direction to the point, where the velocity of 8.4 DRAG FORCE ON FLAT PLATE
fluid is approximately equal to 0.99 times DUE TO BOUNDARY LAYER
free stream velocity ( U ∞ )
8.4.1 VON KARMAN MOMENTUM
Slam ,Stur ,S' laminar sub layer INTEGRAL EQUATION
τ0 dθ
8.3.1 DISPLACEMENT THICKNESS (δ) * =
ρU∞ dx
2

It is defined as displacement of surface in


the direction normal to the surface to  dθ 
=
τ0   ρU ∞ 2
compensate for the reduction in the flow  dx 
rate, due to boundary layer formation. τ0 is shear stress.
θ is momentum thickness

The above equation is valid for


1. Zero pressure gradient.
2. Laminar & turbulent flow
3. Incompressible steady flow.

8.4.2 LOCAL COEFFICIENT OF DRAG


The plate is displaced by distance (δ) * τ0
cd* =
given by 1 2 
δ
 u   ρU∞ 
2 
δ= ∫0 1 − U∞ dy
*

Local shear stress in laminar flow is


obtained by
8.3.2 MOMENTUM THICKNESS τ0 =µ
du
dy
It is defined as the distance, measured Average coefficient of drag
normal to the boundary of the solid body, Favg
by which the boundary should be displaced cd =
to compensate for the reduction in 1 2 
 ρU∞ 
momentum of the flowing fluid on account 2 
of boundary layer formation. L

=θ ∫
δ
u  u  Favg = ∫ τ .b.dx
o
1 − dy o
0
U∞  U∞  b is the width of plate

© Copyright Reserved by Gateflix.in No part of this material should be copied or reproduced without permission
L is the length up to which average force is 2U d  2δ  2
to be evaluated µ=   ρU
δ dx  15 
8.5 BOUNDARY CONDITION FOR THE 15µdx
ρdδ =
VELOCITY PROFILE δU
du
1) at=y 0,= U 0, has some finite value.
dy δ2 15µ
= x +C
2) at y =δ, u = U∞ , 2 ρU
du
3) at y =δ, =0, at= = 0 at 
= 0,C
x 0,δ
dy
Assume velocity profile to be 30µx2
2 δ=
u y y ρUx
= a + b  + c  
U∞ δ δ
Using boundary conditions, value of ρux
= Re
coefficients obtained are μ
a = 0 , b = 2 , c = −1 ,
x2
The velocity profile for laminar boundary δ = 30 .
Re
layer flows is given by x

u Shear stress in terms of Reynolds No


= 2(y / δ) − (y / δ) 2
U∞ 2µU
τo =
Equation involved δ
τ0 do 2µU µU ρUx
= = τo = 0.365
ρU 2
dx x x µ
U is free stream velocity 5.48
Rex
µdu
τo = Coefficient of drag
dy
F0
du d   y   y   cD =
2

= 2  −    1 2
dy dy   δ   δ    ρAU 
  2 
du  2 2Y  L
= U  − 2  at y=0 FD = ∫τo .b.dx
dy δ δ 
o
du 2U 2u
=
dy δ
, τ=µ .... 1
δ
() Where b is thickness of plate
L µU ρUx
δ
u  u FD = ∫ 0.365 gbdx
=θ ∫ 1 −  dy 0 x µ
o
U U
ρU L 1
  Y   Y 2   Y Y  gb∫
2
δ
=
FD 0.365µU dx
=θ ∫ 2  −   1 − 2  +    dy
   µ 0
x
o 
δ   δ    δ   δ  
 
ρUL
2δ =
FD 0.73bµU
θ= µ
15
dθ d  2δ 
=   … (2)
dx dx  15 

© Copyright Reserved by Gateflix.in No part of this material should be copied or reproduced without permission
ρUL If R e > 5 × 105 & less then107
0.73bµU
µ δ=
0.37x ,
Cd =
(R )
1/5
1
ρAU2 ex

2 0.072
CD =
1.46µ ρUL (R )
1/5

Cd = eL
ρU µ Where,
µ x = distance from leading edge
Cd = 1.46
ρUL R e = Reynold’s No. for length x
x

R e = Reynold’s No at the length ‘L’ of plate


L
1.46
Cd = For R e > 107 ,R e > 109
ReL
0.455
CD =
( log10R )
2.58

eL
8.6 ANALYSIS OF TURBULENT
BOUNDARY LAYER Example:
Determine the thickness of the boundary
8.6.1 TURBULENT BOUNDARY LAYER layer at the trailing edge of smooth plate of
ON FLAT PLATE length 4m & width 1.5 m, when the plate is
moving with a velocity of 4 m/s. In
Blasius, on basis of his experiments gave stationary air, kinematic viscosity
expression for velocity profile in turbulent ν 1.5 × 10−5 m2 / s .
=
flow over falt plate
n
u y Solution:
= 
U∞  δ  UL 4× 4
=
Re = = 10.66 × 105
=
Where n 1 / 7,R e < 107
L
ν 1.5 × 10−5
1/7 Turbulent
u y 0.37x
=  = δ = 92.19mm
U∞  δ 
( )
1
Rex 5
Since shear stress in turbulent flow is given
by
 μ 
1/4 8.7 BOUNDARY LAYER SEPARATION
τ o = 0.0225ρu2  
 ρδu 
Separation of flow is said to occur when the
direction of the flow velocity near the
8.6.2 BOUNDARY LAYER THICKNESS surface is opposite to the direction of the
IN TURBULENT FLOW free stream velocity, which means
1
 µ  5 45 (du/dy) ≤ 0. If (dp/dx) increases to the
δ =0.37   gx extent that it can overcome the shear
ρ
 ∞U
near the surface, then separation will
0.37x occur. Such a pressure gradient is called
δ=
adverse pressure gradient. In the case of
( Rex ) 5
1

incompressible flow in a nozzle a


favourable pressure gradient exists &
8.6.3 COEFFICIENT OF DRAG Separation will not occur in such flows. In
the case of diverging section of a diffuser,

© Copyright Reserved by Gateflix.in No part of this material should be copied or reproduced without permission
separation can occur if the rate of area δ 
   y   y  
2 

increase is large. δ=
* ∫0   δ   δ    dy
1 − 2 −

   
δ  y   y  
2
 2y 2 y 3 
δ

=∫ 1 − 2  +   dy = y − + 
0  δ δ   2δ 3δ2  0
 
δ2 δ3 δ δ
=δ− + 2 = δ−δ+ =
δ 3δ 3 3
( ii ) Momentum thickness θ is given by
δ
u u
δ
 2y y 2    2y y 2  
=θ ∫0 U  1 − U =
 dy

∫0  δ − δ2  1 −  δ − δ2  dy
 
δ
 2y y 2   2y y 2 
= ∫0  δ − δ2  1 − δ + δ2  dy
δ
 2y 4y 2 2y 3 y 2 2y 3 y 4 
8.7.1 LOCATION OF SEPARATION POINT
= ∫0  δ − δ2 + δ3 − δ2 + δ3 − δ4 dy
δ
δ
 2y 5y 2 4y 3 y 4   2y 2 5y 3 4y 4 y 5 
 ∂u  = ∫0  δ − δ2 + δ3 − δ4 dy =  2δ − 3δ2 + 4δ3 − 5δ4 
  < 0 Separated flow 0

 ∂y y =0  δ2 5δ3 δ4 δ5  5δ δ
=  − 2 + 3 − 4  =δ− +δ−
 ∂u   δ 3 δ δ 5 δ  3 5
  = 0 Verge of separation
 ∂y  y =0 =
15δ − 25δ + 15δ − 3δ 30δ − 28δ 2δ
= =
15 15 15
 ∂u 
  > 0 No separation
 ∂y  y =0 ( iii ) Energy thickness δ * * is given by
 2y y 2    2y y 2  
2
u  u2 
δ δ

Example: δ *=
* ∫0 U 1 − U2  dy=  −   1 −  −   dy
∫0  δ δ2    δ δ2  
Find the displacement thickness, the  
momentum thickness and energy for the δ
 2y y 2    4y 2 y 4 4y 3  
velocity distribution in the boundary layer = ∫0  δ δ2    δ2 + δ4 − δ3   dy
−  1 −
2
 
u y y δ
 2y y 2  4y 2 y 4 4y 3 
= 2  −  
given by
U δ δ
= ∫0  δ δ2 
−  1 − − +
δ2 δ4 δ3 
 dy

Solution: δ
 2y 8y 3 2y 5 8y 4 y 2 4y 4 y 6 4y 5 
Given: = ∫0  δ − δ3 − δ5 + δ4 − δ2 + δ4 + δ6 − δ5 dy
2
u y y  2y y 2 8y 3 12y 4 6y 5 y 6 
δ
Velocity distribution,
= 2  −  
U δ δ
= ∫0  δ − δ2 − δ3 + δ4 − δ5 + δ6 dy
( i ) Displacement thickness δ * is given
 2y y 3 8y 4 12y 5 6y 6 y 7 
δ

by = − 2− 3+ − + 
δ
 2δ 3δ 4δ 5δ4 6δ5 7δ6  0
 u
δ=
* ∫  1 − U  dy =
δ2 δ3 2δ4 12δ5 δ6 δ7 δ 12
− 2 − 3 + 4 − 5 + 6 = δ − − 2δ + δ − δ +
δ
0 δ 3δ δ 5δ δ 7δ 3 5 7
2
Substituting the value of,=u y y δ 12 δ −210δ − 35δ + 252δ + 15δ
2  −   we = −2δ − + δ + =
U δ δ 3 5 7 105
have −245δ + 267δ 22δ
= =
105 105

© Copyright Reserved by Gateflix.in No part of this material should be copied or reproduced without permission
Example: plate is moving with a velocity of 4m/s in
Air is flowing over a smooth plate with a stationary air. Take kinematics viscosity of
velocity of 10 m/s. The length of the plate is air as 1.5×10−5 m2 / s
1.2m and width is 0.8m. If laminar
boundary layer exists up to a value of Solution:
R e = 2 × 105 , find the maximum distance Given:
from the leading edge upto which laminar Length of plate, L = 4m
boundary layer exists. Find the maximum Width of plate, b=1.5m
thickness of laminar boundary layer if the Velocity of plate, U = 4m/ s
velocity profile is given by Kinematics viscosity,   1.5105 m 2 / s
2
u y y Reynolds number,
= 2  −   U L 4 4
U δ δ Re L   5
 10.66105
 1.510
Take kinematics viscosity for air = 0.15 As the Reynolds number is more than
stokes.
5 × 105 and hence the boundary layer at the
Solution: trailing edge is turbulent.
Given: The boundary layer thickness for turbulent
Velocity of air, U =10m/ s boundary layer is given by
Length of plate, L = 1.2m δ=
0.37x=
Here x ( L=
and R e R e
x L
)
b= 0.8m (R )
1
Width of plate, 5
ex
Reynolds number upto which laminar
0.37 × 4.0
boundary exists = 2 × 10
5 = = = 92.1mm
0.0921m
( )
1
10.66 × 105 5
ν for = = 0.15 × 10−4 m2 / s
air 0.15 stokes
ρUx Ux Example:
=
Reynolds number, Re =
L
µ ν Water is flowing over a thin smooth plate
If R e = 2 × 10 , then x denotes the distance
5 of length 4m and width 2m at a velocity of
x
1.0m/s. If the boundary layer flow changes
from leading edge upto which laminar from laminar to turbulent at a Reynolds
boundary layer exists
number 5 × 10 , find ( i ) the distance from
5
10 × x
∴ 2 × 105 = leading edge upto which boundary layer is
0.15 × 10−4
laminar, ( ii ) the thickness of boundary
∴x 2 × 105 × 0.15 × 10−4
= = 0.30m= 300mm
10 layer at the transition point, and ( iii ) the
Maximum thickness of the laminar drag force on one side of the plate. Take
boundary for the velocity profile, viscosity of water µ = 9.81×10−4 Ns/ m2 .
2
u y y
= 2  −   Is given by
U δ δ
5.48 × x 5.48 × 0.30
=δ = = 0.00367m
= 3.67mm
Re 2 × 105
x

Example:
Determine the thickness of the boundary
layer at the trailing edge of smooth plate of Solution:
length 4m and of width 1.5m, when the Given:

© Copyright Reserved by Gateflix.in No part of this material should be copied or reproduced without permission
Length of plate, L = 4m
Width of plate, b= 2m b) Drag force due to turbulent boundary
Velocity of flow, U =1.0m/ s layer from F to G =Drag force due to
Reynolds number for laminar boundary turbulent boundary layer from E to G −
Drag force due to turbulent flow from E
layer = 5 × 10
5
to F
Viscosity of water, µ = 9.81×10−4 Ns/ m2
i) Let the distance from leading edge upto
= FEG ( )− FEF
turb
( ) turb

Now, 1
which laminar boundary layer ( FFG )turb =ρ
2
AU2 × CD
exists = x
Ux 10001 x Where CD is
 5105   0.072
 9.81104 CD =
(R )
1
−4 5
∴ x = 5 × 10 × 9.81 × 10
5
eL
1000
But
= 0.4900m
= 490mm
ρUL 1.0 × 4.0
ii) Thickness of boundary layer at the R e ==×
1000 =×
40.77 105
µ 9.81 × 10−4 L

point where the boundary layer


changes from laminar to turbulent i.e.,= 0.072
∴ C = 0.00343 D
( 40.77 × 105)
1
at Reynolds number = 5 × 10 , is given
5 5

by Blasius’s solution as
1
=δ =
5x 5g0.49
= 3.46mm
∴ (F )
EG turb
=ρAU2 × CD
2
Rex 5 × 105 1
= × 1000 × (4 × 2) × 12 × .00343 13.72N
=
(Here x= 49cm= 0.49m, R e = 5 × 10
x
5
) 2
1
iii) Drag force on the plate on one side
= Drag due to laminar boundary layer
Also FEF
turb 2
( )
= ρA EF × U2 × CD
+ Drag due to turbulent boundary Where A EF =Area of plate upto
a) Drag due to laminar boundary layer EF = EF× b = 0.49×2= 0.98 m2
(i.e., from E to F)
And
1
FEF = ρ AU2×CD ...... ( i ) CD =
0.072
=
0.072
= .00522
2
( R EF ) ( )
1 1
5 5 5
Where CD is given by Blasius solution 5×10
for laminar boundary layer as 1
(F ) = ×1000×0.98×12 ×.00522= 2.557N
CD =
1.328
=
1.328
( for EF,R ex
=5×10 5
) EF turb
2
∴ Drag force due to turbulent boundary
Re 5×10 5

layer from F to G
x

= 0.001878
A=Area of plate upto laminar boundary = ( FEG ) - ( FEF ) =13.72-2.557 =11.163N
turb turb
layer ∴ Drag force on the plate on one side
= 0.49×b= 0.49×2= 0.98 m2 =Drag force due to laminar boundary
layer upto F + Drag force due to
Substituting the value of CD and A in turbulent boundary layer from F to G
equation ( i ) , we get = 0.92+11.163 = 12.083N

1 Example:
FEF = ×1000×0.98×1.02 ×.001878 = 0.92N
2 For the following velocity profiles,
.... ( ii ) determine whether the flow has separated

© Copyright Reserved by Gateflix.in No part of this material should be copied or reproduced without permission
or on the verge of separation or will attach  δu 
with the surface. As   = 0 , the flow is on verge of
3 δy
  y =0
u 3 y  1 y 
=
i)  −   separation.
U 2 δ  2 δ 

u y y
2 3
3rd velocity profile
=
ii) 2  −   2
u y y
U δ δ =
−2  +  
2 U δ δ
u y y
iii) = −2  +   y y
2

U δ δ ∴u =
−2U   + U  
Solution: δ δ
Given δu 1 y 1
=
−2U   + 2U   ×
1st velocity profile δy δ δ δ
3 3
u 3 y  1 y  3U  y  U  y 
=−    or u =  −  
U 2 δ  2 δ  2 δ 2δ
Differentiating w.r.t., y, the above equation
At y=0,
becomes,
2  δu  2U 0 1 2U
δ u 3U 1 U  y  1   = − + 2U   × =−
= × − ×3  ×  δy  y =0 δ δ δ δ
δy 2 δ 2 δ δ
 δu 
At y =0 , As   is negative the flow has
2  δy  y =0
 δu  3U 3U  0  1 3U
  = −   × = separated.
 δ y  y =0 2δ 2  δ  δ 2δ
 u 
As   is positive. Hence flow will not
  y 
y 0

separate or flow will remain attached with


the surface.

2nd velocity profile


2 3
u y y
= 2  −  
U δ δ
2 3
y y
=
∴ u 2U   − U  
δ δ
2
δu y 1 y 1
∴ = 2U × 2  × − U × 3  ×
δy δ δ δ δ
 δu 
At y = 0,  
 δ y  y =0
2
0 1 0 1
= 2U × 2  × − U × 3  × = 0
δ δ δ δ

© Copyright Reserved by Gateflix.in No part of this material should be copied or reproduced without permission
GATE QUESTIONS

Q.1 Flow separation in flow past a solid the Darcy friction factor is 64/Re
object is caused by where Re is the Reynolds
a) a reduction of pressure to number. Given that the viscosity of
vapour pressure water is 1.0 × 10−3 kg/s-m, the force
b) a negative pressure gradient F in newtons required on the
c) a positive pressure gradient plunger is
d) the boundary layer thickness a) 0.13 b) 0.16
reducing to zero c) 0.3 d) 4.4
[GATE–2002] [GATE–2003]

Q.2 If x is the distance measured from Q.5 An incompressible fluid (kinematic


the leading edge of a flat plate, the viscosity, 7.4 × 10−7 m2/s , specific
laminar boundary layer thickness gravity, 0.88) is held between two
varies as parallel plates. If the top plate is
1 moved with a velocity of 0.5 m/s
a) b) x 4/5
x while the bottom one is held
c) x 2 d) x1/2 stationary, the fluid attains a linear
[GATE–2002] velocity profile in the gap of 0.5 mm
between these plates; the shear
Common Data Questions Q.3 to Q.4 stress in Pascals on the surfaces of
A syringe with a frictionless plunger top plate is
contains water and has at its end a 100 mm a) 0.651×103 b) 0.651
long needle of 1 mm diameter. The internal c) 6.51 d) 0.651×103
diameter of the syringe is 10 mm. Water [GATE–2004]
density is 1000 kg/m3. The plunger is
pushed in at 10 mm/s and the water comes Q.6 For air flow over a flat plate, velocity
out as a jet. (U) and boundary layer thickness
(δ) can be expressed respectively, as
3
U 3 y 1 y 4.64x
=−   ;δ =
U∞ 2 δ 2  δ  Re x
If the free stream velocity is 2 m/s,
and air has kinematic viscosity of
Q.3 Assuming ideal flow, the force F in 1.5 × 10−5 m2/s and density of
newtons required on the plunger to 1.23 kg/m3, the wall shear stress at
push out the water is x = 1 m, is
a) 0 b) 0.04 a) 2.36 × 102 N/m2
c) 0.13 d) 1.15 b) 43.6 × 10-3 N/m2
[GATE–2003] c) 4.36 × 10-3 N/m2
d) 2.18 ×10-3 N/m2
Q.4 Neglect losses in the cylinder and [GATE–2004]
assume fully developed laminar
viscous flow throughout the needle;

© Copyright Reserved by Gateflix.in No part of this material should be copied or reproduced without permission
Q.7 The velocity profile in fully plate to the drag force on the rear
developed laminar flow in a pipe of half, then
diameter D is given by u= u0(1 -4r a) F < 1/2 b) F = 1/2
/D2), where r is the radial distance c) F= 1 d) F> 1
from the center. If the viscosity of [GATE–2007]
the fluid is μ, the pressure drop Q.11 Consider steady laminar
across a length L of the pipe is incompressible anti-symmetric fully
µu 0 L 4µu 0 L developed viscous flow through a
a) b)
D 2
D2 straight circular pipe of constant
8µu 0 L 16µu 0 L cross-sectional area at a Reynolds
c) 2
d) number of 5. The ratio of inertia
D D2
force to viscous force on a fluid
[GATE–2006]
particle is
a) 5 b) 1/5
Common Data For Q.8 and Q.9
A smooth flat plate with a sharp leading c) 0 d)
edge is placed along a gas stream flowing at [GATE–2007]
U=10 m/s. The thickness of the boundary
layer at section r-s is 10 mm, the breadth of Common Data For Q.12 and Q.13 :
the plate is 1m (into the paper) and the Consider a steady incompressible flow
density of the gas ρ = 1.0 kg/m3. Assume through a channel as shown below.
that the boundary layer is thin, two-
dimensional, and follows a linear velocity
distribution, u = U(y/δ), at the section r-s,
where y is the height from plate.

The velocity profile is uniform with a value


of U0at the inlet section A. The velocity
Q.8 The mass flow rate (in kg/s) across profile at section B downstream is
the section q - r is  y
a) zero b) 0.05  Vm δ , 0≤y≤δ
c) 0.10 d) 0.15 
= u  Vm , δ ≤ y ≤ H−δ
[GATE–2006]
 H−y
Vm H−δ ≤ y ≤ H
Q.9 The integrated drag force (in N) on  δ
the plate, between p-s, is
a) 0.67 b) 0.33 Q.12 The ratio Vm/U0is
c) 0.17 d) zero 1
[GATE–2006] a) b) 1
1 − 2(δ − H)
1 1
Q.10 Consider an incompressible laminar c) d)
boundary layer flow over a flat plate 1 − (δ / H) 1 + (δ / H)
of length L, aligned with the [GATE–2007]
direction of an incoming uniform
free stream. If F is the ratio of the
drag force on the front half of the

© Copyright Reserved by Gateflix.in No part of this material should be copied or reproduced without permission
pA − pB two fixed parallel plates, is 6 m/s.
Q.13 The ratio (where pAand
1 2 The mean velocity (in m/s) of the
ρU 0 flow is
2
pBare the pressures at section A and a) 2 b) 3
Brespectively, and ρis the density of c) 4 d) 5
the fluid) is [GATE–2010]
1 1 Q.17 Match the following
a) b) −1 P.Compressible flow U.Reynolds number
[1 + (δ / H)] [1 − (δ / H)]
2 2
Q. Free surface flow V.Nusselt number
1 1 R.Boundary layer flow W. Weber number
c) − 1 d) S.Pipe flow X.Froude number
(1 − (2δ / H) 2
1 + (δ / H) T. Heat convection Y. Mach number
[GATE–2007] a) P-U; Q-X; R-V; S-Z; T-W
b) P-W; Q-X; R-Z; S-U; T-V
Q.14 The velocity profile of a fully c) P-Y; Q-W; R-Z; S-U; T-X
developed laminar flow in a straight d) P-Y; Q-W; R-Z; S-U; T-V
circular pipe, as shown in the figure, [GATE–2010(ME)]
is given by the expression Q.18 An incompressible fluid flows over
R 2  dp   r2  a flat plate with zero pressure
u(r) = −   1 − 
4μ  dx   R 2  gradient. The boundary layer
thickness is 1 mm at a location
dp
where is a constant. The where the Reynolds number is
dx 1000. If the velocity of the fluid
average velocity of fluid in the pipe alone is increased by a factor of 4,
is then the boundary layer thickness
at the same location, in mm will be
a) 4 b) 2
c) 0.5 d) 0.25
[GATE–2012]

Q.19 Consider the turbulent flow of a


fluid through a circular pipe of
R 2  dp  R 2  dp  diameter, D. Identify the correct pair
a) −   b) −  
8μ  dx  4μ  dx  of statements.
I. The fluid is well-mixed
R 2  dp  R 2  dp 
c) −   d) −   II. The fluid is unmixed
2μ  dx  μ  dx  III. ReD< 2300
[GATE–2009] IV. ReD> 2300
a) I and III b) II and IV
Q.15 A phenomenon is modeled using n c) II and III d) I and IV
dimensional variables with k [GATE–2014(3)]
primary dimensions. The number of
non-dimensional variables is Q.20 Water flows through a pipe having
a) k b) n an inner radius of 10 mm at the rate
c) n− k d) n+k of 36 kg/hr at 25°C. The viscosity of
[GATE–2010] water at 25°C is 0.001 kg/m.s. The
Reynolds number of the flow is_____.
Q.16 The maximum velocity of a one- [GATE–2014 (1)]
dimensional incompressible fully
developed viscous flow, between

© Copyright Reserved by Gateflix.in No part of this material should be copied or reproduced without permission
Q.21 For a fully developed flow of water If the average value of friction
in a pipe having diameter 10 cm, 1.328
coefficient is Cf = the total
velocity 0.1 m/s and kinematic Re x
viscosity 10−5 m2/s, the value of
drag force (in N) per unit width of
Darcy friction factor is _____.
the plate is _____.
[GATE–2014 (1)]
[GATE–2015 (1)]
Q.22 In a simple concentric shaft-bearing
Q.26 For a fully developed laminar flow
arrangement, the lubricant flows in
of water (dynamic viscosity 0.001
the 2 mm gap between the shaft and
Pa-s) through a pipe of radius 5 cm,
the bearing. The flow may be
the axial pressure gradient is
assumed to be a plane Couette flow
−10Pa/m. The magnitude of axial
with zero pressure gradient. The
velocity (in m/s) at the radial
diameter of the shaft is 100 mm and
location of 0.2 cm is __________.
its tangential speed is 10 m/s. The
[GATE–2015 (2)]
dynamic viscosity of the lubricant is
0.1 kg/ms. The frictional resisting Q.27 The instantaneous stream-wise
force (in newton) per 100 mm velocity of a turbulent flow is given
length of the bearing is ____. as follows:
[GATE–2014 (1)] u(x, y, z, t) = u(x, y, z) + u′(x, y, z, t)
The time-average of the fluctuating
Q.23 Consider laminar flow of water over velocity u’(x, y, z, t) is
a flat plate of length 1 m. If the a) 𝑢𝑢′/2 b) − u /2
boundary layer thickness at a c) zero d) 𝑢𝑢 /2
distance of 0.25 m from the leading [GATE–2016 (1)]
edge of the plate is 8 mm, the
boundary layer thickness (in mm), Q.28 Oil (kinematic viscosity, ν𝑜𝑜𝑜𝑜𝑜𝑜 = 1.0 ×
at a distance of 0.75 m, is _____ 10−5 m2/s) flows through a pipe of
[GATE–2014 (2)] 0.5 m diameter with a velocity of 10
m/s. Water (kinematic viscosity, ν𝑤𝑤
Q.24 Couette flow is characterized by = 0.89 × 10−6 m2/s) is flowing
a) steady, incompressible, laminar through a model pipe of diameter 20
flow through a straight circular mm. For satisfying the dynamic
pipe similarity, the velocity of water (in
b) fully developed turbulent flow m/s) is __________
through a straight circular pipe [GATE–2016 (1)]
c) steady, incompressible, laminar
flow between two fixed parallel Q.29 A steady laminar boundary layer is
plates formed over a flat plate as shown in
d) steady, incompressible, laminar the figure. The free stream velocity
flow between one fixed plate and of the fluid is Uo. The velocity profile
the other moving with a constant at the inlet a-b is uniform, while that
velocity at a downstream location c-d is
[GATE–2015 (3)]   y   y 2 
given by 𝑢𝑢 =  2   −    .
Q.25 Air (ρ= 1.2 kg/m3 and kinematic   δ   δ  
viscosity, υ = 2 × 10−5 m2/s) with a
velocity of 2 m/s flows over the top
surface of a flat plate of length 2.5 m.

© Copyright Reserved by Gateflix.in No part of this material should be copied or reproduced without permission
2 2
a) b) 1 −
π π
2
c) 1 + d) 0
π
[GATE–2017 (1)]
The ratio of the mass flow rate,ṁ bd ,
leaving through the horizontal
section b-d to that entering through Q.33 Consider a laminar flow at zero incidence
the vertical section a-b is ___________ over a flat plate. The shear stress at the
[GATE–2016 (1)] wall is denoted by τw The axial positions

Q.30 Consider fluid flow between two x1 and x 2 on the plate are measured
infinite horizontal plates which are from the leading edge in the direction of
parallel (the gap between them flow. If x 2 > x1 , then
being 50 mm). The top plate is
sliding parallel to the stationary a) τw x1 =
τw x 2 =
0
bottom plate at a speed of 3 m/s.
b) τw x1 =
τw x 2 ≠ 0
The flow between the plates is
solely due to the motion of the top c) τw x1 > τw x 2
plate. The force per unit area
(magnitude) required to maintain d) τw x1 < τw x 2
the bottom plate stationary is [GATE–2017 (2)]
_________ N/m2. Viscosity of the fluid
μ = 0.44 kg/m-s and density ρ = 888
kg/m3.
Q.34 For the laminar flow of water over a
[GATE–2016 (2)]
sphere, the drag coefficient CF is defined

Q.31 Consider a fully developed steady


as = ( )
CF F / ρU2D2 where F is the drag
laminar flow of an incompressible force, ρ is the fluid density, U is the fluid
fluid with viscosity μ through a velocity and D is the diameter of the
circular pipe of radius R. Given that sphere. The density of water is
the velocity at a radial location of
R/2 from the centerline of the pipe 1000kg / m3 . When the diameter of the
is U1, the shear stress at the wall is sphere is 100mm and the fluid velocity is
KμU1/R, where K is __________ 2m/s, the drag coefficient is 0.5. If water
[GATE–2016 (3)] now flows over another sphere of
Q.32 The velocity profile inside the diameter 200mm under dynamically
boundary layer for flow over a flat similar conditions, the drag force (in N)
u π y on this sphere is _____________
plate is given as = sin  
U∞ 2 δ
where U ∞ is the free stream velocity [GATE–2017 (2)]
and δ is the local boundary layer Q.35 The viscous laminar flow of air over a
thickness. If δ * is the local flat plate results in the formation of a
displacement thickness, the value of boundary layer. The boundary layer
δ* thickness at the end of the plate of
is length L is δL . When the plate length
δ

© Copyright Reserved by Gateflix.in No part of this material should be copied or reproduced without permission
is increased to twice its original
length, the percentage change in
laminar boundary layer thickness at
the end of the plate (with respect to δL )
is ________ (correct to two decimal
places).
[GATE–2018 (2)]

ANSWER KEY:
1 2 3 4 5 6 7 8 9 10 11 12 13 14
(c) (d) (b) (c) (b) (c) (d) (b) (c) (d) (a) (c) (b) (a)
15 16 17 18 19 20 21 22 23 24 25 26 27 28
(c) (c) (d) (c) (d) 636.93 0.06 15.7 13.8 (d) 0.015 0.64 (c) 22.25
29 30 31 32 33 34 35
0.33 26.4 2.67 (b) (c) 20 41.42

© Copyright Reserved by Gateflix.in No part of this material should be copied or reproduced without permission
EXPLANATIONS

Q.1 (c)
The pressure is minimum at point C. Apply continuity equation on
Along the region CSD of the curved section (1) and (2),
surface, the area of flow increases A1V1 = A2V2
and hence velocity of flow along the
direction of Fluid decreases. π
× 0.012
A1 4
V2 = = V1 ×=
0.01 1 m / s
A2 π
× 0.001 2

4
Again applying the Bernoulli’s
equation at section (1) and (2)
p1 V12 p 2 V22
+ + Z1 = + + Z2
ρg 2g ρg 2g
Since z1 = z2.
Due to decrease of velocity, the Taking gauge pressure
pressure increases in the direction p2 =0 (Atmospheric pressure)
dp p1 V22 V12
of flow and pressure gradient is = −
dx ρg 2g 2g
dp Putting the values of V1 and V2 ,
positive or >0
dx We get p1 = 499.95 N/m2
Force required on plunger
Q.2 (d) = p1 × A
For laminar flow π
δ 1 = 499.95× × 0.012
∝ 4
x R ex =0.04 N
x
δ∝ Q.4 (c)
Vx
ρV2 d 2
υ Re =
1
μ
δ∝ x ⇒δ∝x 2
1000 ×1× 0.001
=
1×10−3
Q.3 (b) Re=1000
Given: L = 100 mm, d = 1mm, Now Darcy’s friction factor,
D =10 mm, V1 = 10 / s 64 64
We have to take the two sections of f= =
the system (1) and (2). Re 1000
f = 0.064
Head loss in needle,
fLV 2
hf =
2gd

© Copyright Reserved by Gateflix.in No part of this material should be copied or reproduced without permission
0.064 × 0.1× (1) 2 = 0.6512 N/m2
=
2 × 9.81× 0.001
= 0.3262m of water Q.6 (c)
Applying Bernoulli’s equation at Given relation is,
3
points 1 to 2, we get U 3 y 1 y 4.64x
= −   ;δ =
p1 V1 2
p V 2
U∞ 2 δ 2  δ  Re x
+ + z1 = 2 + 2 + z 2 + h f
ρg 2g ρg 2g U∞ = 2 m/s, υ = 1.5 × 10−5 m2/s
Since z1 = z2 ρ = 1.23 kg/m3
And p2 = 0 (atmospheric) Kinematic viscosity
p1  V2 − V1 
2 2 µ
=  +
 f h v = ⇒ µ = v × p = 1.5 ×10−5 ×1.23
ρg  2g  p
ρ 2 = 1.845×10-5 kg/m.sec
p1=
2
( V2 − V1 ) + ρgh f
2
Reynolds Number is given as,
pUx 1.23 × 2 ×1
1000 =
(1) 2 − (0.01) 2  + 1000 × 9.81× 0.3262 Re x = = 1.33 ×105
= μ 1.845 ×10 −5
2
p1 = 3699.97 N/m2 4.64 × 1
= δ = 0.0127
Force required on plunger 1.33 ×102
p1 × A1 u 3y 1  y 
2

π And = −  
= 3699.97 × × (0.01) 2 u ∞ 2δ 2  δ 
4
d  3y 1  y  
3
= 0.29 N dU
= U∞  −   
dy dy  2δ 2  δ  
Q.5 (b)
du  3 1 3 y2 
Shear stress τ = μ = U ∞  × − 3 
dy 2 δ 2 δ 
For linear velocity profile, Where U ∞ = Free stream velocity
 dU   3   3U ∞ 
Now,   = U∞   = 
 dy  y =0  2δ   2δ 
Now, shears stress
 dU  −5 3U ∞
τ0 =µ   =1.8 ×10 ×
 dy  y =0 2δ
Substitute the values of and we
get
3× 2
= 1.845× 10−5 ×
2 × 0.0127
u = 435.82 × 10−5 N/m2
τ = μ.
y = 4.36 × 10−3 N/m2
μ
υ=
ρ Q.7 (d)
⇒μ= υ×ρ Given
=7.4 ×10−7 × (0.88 ×1000)  4r 2   r2 
u = u 0 1 − 2  = u 0 1 − 2 
= 6.512 ×10−4 Pa − s  D   R 
0.5 Drop of pressure for a given length
∴ τ = 6.512 ×10−4 × (L)of a pipe is given by,.
0.5 ×10−3

© Copyright Reserved by Gateflix.in No part of this material should be copied or reproduced without permission
32µuL δ δ δ
∆P = P1 − P2 = = − =
D2 2 3 6
Where u is the average velocity. ∴force on plate per unit width
We know that for a laminar flow = ρu ∞2 × (θ ×1)
through a pipe. Here, u∞ = U = 10m/s
uo = 2u or u=uo/2
 10 ×10−3 
Where uois the maximum velocity. FD = 1× (10) 2 ×  × 1
Substituting the value of u, we get  6 
32µL u 0 32µu 0 L FD = 0.1666N
∆=P ×=
D2 2 D2
Q.10 (d)
Q.8 (b) 1
Drag force F=
D CD × ρAV 2
2
1.328 1
= × ρAV 2
Re L 2
1.328 1
= × ρ × A × L × V2
Mass flow rate entering ρLV 2
(PQ) = ρ × volume μ
m1 = ρ × A × U i.e. FD ∝ L
= 1 × (1 × δ) × 10
= 1 × 10 × 10-3 × 10 Now drag force on front half
= 0.1 kg/ sec FD/2 ∝
L
Mass flow rate entering (SR) 2
1
m 2 = ρ×1× ( δ × U ) ∴ FD/2 = D
F
2 2
1 Drag force on rear half
= 1×1× ×10 ×10−3 ×10
2 F'D/2= FD − FD/2
= 0.05 kg/sec
∴ Mass leaving(QR) = m1– m2  1 
= 1 −  FD
= 0.1 – 0.05  2
= 0.05 kg/sec F
Now F = D/2
F'D/2
Q.9 (c)
FD
Drag force on plate = rate of change
2= 1
of momentum of the flowing fluid. = >1
Momentum thickness  1  2 −1
1 −  FD
δ
u  u   2
( ) ∫ 1 −  dy
=θ ∴ F >1
0 ∞ 
u u∞ 
δ
y y Q.11 (a)
=θ ∫ δ 1 − δ  dy Inertia force
0
Reynolds Number = =5
δ
 y y2  Viscous force
=θ ∫0  δ − δ2  dy
Q.12 (c)
3 δ
y 2
y Assume width of channel is b.
= − 2 Applying continuity equation at A
2δ 3δ 0
and B

© Copyright Reserved by Gateflix.in No part of this material should be copied or reproduced without permission
The velocity profile of a fully
developed laminar flow in a straight
ρU 0 ( H ×=
b ) ρVm (H − 2δ) × ba
circular pipe, as shown in figure, is
 δ  given by the expression
+2 ρ × ∫u.dy.b 
 0  R 2  dp   r 2 
u (r) =−   1−
Vm y 2
δ 4μ  dx   R 2 
U 0 H= Vm ( H − 2δ ) + 2 .
δ 2  r2 
0 = U max 1 − 2 
 R 
∴ U 0 H= Vm ( H − 2δ ) + Vm .δ R 2  dP 
Where U max = −  
Vm H 1 4μ  dx 
∴= =
U0 H − δ 1 − δ dP
Where is a constant
H dx
The average velocity of fluid in the
Q.13 (b) pipe is
Applying Bernoulli’s equation at A U R 2  dp 
and B Vavg = max = −  
2 8μ  dx 
p A VA2 p B VB2
+ = +
ρg 2g ρg 2g Q.15 (c)
p − p B VB2 − VA2 From Buckingham’s p-theorem
⇒ A = It states “If there are n variable
ρg 2g
(Independent & dependent variables)
p A − p B Vm2 − U o2
⇒ = in a physical phenomenon and if
ρ 2 these variables contain k
p − p B  Vm 
2
fundamental dimensions (M,L,T),
⇒ A= −1
1 2  U 0  then variables are arranged into (n
ρU 0 −k) dimensionless terms.
2
Here n = dimensional variables
V
Using the value of m from first k = Primary dimensions (M, L, T)
U0 So, non-dimensional variables is n-k
part of question, we get,
PA − PB 1 Q.16 (c)
= −1
1 2  δ 
2 In case of two parallel plates, when
ρU 0
2 1 −  flow is fully developed, the ratio of
 H
Vmax and Vavg is a constant.
Vmax 3
Q.14 (a) = , Vmax = 6m / sec
Vavg 2
2 2
Vav = × Vmax = × 6 = 4m / sec
3 3

© Copyright Reserved by Gateflix.in No part of this material should be copied or reproduced without permission
Q.17 (d) VD 0.1× 0.1
Re = =
Here type of flow is related to the υ 10−5
dimensionless numbers (Non- =1000
dimensional numbers). 64 64
P. Compressible flow Y. Mach number f= =
Q. Free surface flow W.Webernumber
Re 1000
R. Boundary layer Z.Skin friction coefficient = 0.064
S. Pipe flow U.Reynolds number
T. Heat convection V.Nusselt number Q.22 (15.7)
So, correct pairs are Given,
P-Y, Q-W, R-Z, S-U, T-V d = 100 mm
Q.18 (c) L =100 mm
For flat plate with zero pressure U = 10 m/s
gradient and Re = 1000 (laminar μ = 0.1 kg/ms
flow). Boundary layer thickness y= 2 ×10−3 m
U
varies as δ ∝ V −1/2 τ=μ
y
where V = velocity of fluid
(Linearization of Newton’s law of
δ2 V1 viscosity)
= =
δ1 V2 10
= 0.1 ×
V1 2 ×10−3
δ2 = × δ1 = × = 500 N/m2
4V1 Frictional area = πdL
1 = 3.14×0.1×0.1
δ2 = ×1 = 0.5 mm
2 Frictional resistance force
= τ × πdL
Q.19 (d) = 500×0.0314 = 15.7 N
For turbulent flow, Re > 2300
Turbulent flow is considered as well Q.23 (13.85)
mixed flow. δ ∝ √x
δ2 x2 0.75
Q.20 (636.94) = = = 3
δ1 x1 0.25
d =0.02 m
ṁ = 36 kg/hr ∴ δ2 = δ1 √3
= 0.01 kg/s = 1.732×8 =13.8564 mm
μ = 0.001kg/ms
ρVd md & Q.24 (d)
=Re= = (Q ρAV m)
& Couette flow is steady
μ μA
incompressible, laminar flow
4md& 4m& between one fixed plate and other
Re = =
μπd 2
μπd moving with constant velocity.
4 × 0.01
=
3.14 × 0.001× 0.02
= 636.94

Q.21 (0.064) Q.25 (0.0159)


Given, 1.328
Cf =
D = 10 cm Re x
V=0.1 m/s
υ= 10−5 m2/s

© Copyright Reserved by Gateflix.in No part of this material should be copied or reproduced without permission
ρVx Vx  δ 2
=
R ex = = ρU 0 δ −  = ρδU 0
μ υ  3 3
V = 2m/s ∴ m &=bd m&ab − m
&cd
L =2.5m 2
υ =2×10−5 m2/s = ρδU 0 − ρδU 0
3
1
F = Cf ρAV 2 1
2 = ρδU 0
3
A=2.5×1
1
&bd 3 ρδU 0 1
On substituting we get
m
F = 0.0159 N =
∴ = = 0.33
m&ab ρδU 0 3
Q.26 (6.24)
−1 ∂P 2 2 Q.30 (26.4)
= u (R − r ) The force required to hold the
4μ ∂x
bottom plate stationary will be
−1
= × ( −10 ) (0.05) 2 − (0.002) 2  equal to drag force exerted by the
4 × 0.001 upper plate.
u = 6.24m/s F U
∴ =μ
A h
Q.27 (c)
3
= 0.44 × = 26.4 N/m2
Q.28 (22.25) 50 ×10−3
For dynamic similarity – Reynold’s
Number should be equal Q.31 (2.67)
V D VD We know that
∴ oil oil = ω ω −1 ∂P 2 2
Voil ω = u [R − r ]
4μ ∂x
10 × 0.5 0.89 ×10−6
=
∴ Vω × R
10−5 0.02 =
at r = , u U1 (given)
2
Vω = 22.25m / s
−1 ∂P  2 R 2 
= R −
4μ ∂x  4 
U1
Q.29 (0.32 to 0.34)
−1 ∂P  3R 2 
U1 =  
4μ ∂x  4 
∂P −16μU1
= ----- (1)
∂x 3R 2
−∂P r
Also, τ =
Considering unit width of the plate ∂x 2
m&ab =ρ ( δ ×1) U 0 =ρδU 0 At r = R
∂P R
&cd ρ ( dy ×1) u
=
dm τ= −
∂x 2
  y   y 2 
δ
 −16μ  R
=&cd ρ ∫U 0  2   −    dy
∴m τ= − U1  × [From equation
  δ   δ    3R  2
2
0
δ (1)]
 y 2 y3  8 μU1
=m cd ρU 0 x 
& − 2 2
∴ τ=
 2δ 3δ  0 3 R

© Copyright Reserved by Gateflix.in No part of this material should be copied or reproduced without permission
μU1
τ = 2.67
R
∴ K = 2.67

Q.32 (b)
δ
u
Q δ*= ∫ (1 − u
0 ∞
)dy
δ
  π y 
δ*= ∫ 1 − sin  2 . δ dy
0
δ
  π y  2δ 
δ =  y + cos  .  
*

  2 δ  π 0

δ* = δ + 0 − 0 −
π
 2
δ* =δ 1 − 
 π
δ *
2
= 1−
δ π

Q.33 (c)
1
τ∝
x1/2
∴ as the distance from the
leading edge increases shear stress
decreases.

Q.34 (20)
For Dynamic similarity
Re1 = Re 2
ρV1D1 ρV2 D 2
=
µ µ
V1D1 = V2 D 2 ----- (1)
And we know
24
CD = (For flow over sphere )
Re
∴ CD1 = CD2
Now
=
F2 CD2 ρU 2 2 D 22
= CD1 ρ ( U1D1 )
2

F2 =
0.5 ×1000(2 × .1) 2
F2 = 20N

© Copyright Reserved by Gateflix.in No part of this material should be copied or reproduced without permission
Q.35(41.42)
In laminar flow over a flat plate,
δ ∝ x1/2
Given, δ = δ L at x = L
Let, δ δ=
= 2 L at x 2L
δ 2L
Then, =
2L
= =
2 1.414
δL L
δ2L − δ L
Therefor % change in δ =  100
δL
= (1.4142 − 1)  100 = 41.42 %

© Copyright Reserved by Gateflix.in No part of this material should be copied or reproduced without permission
9 HYDRAULIC TURBINES

9.1 INTRODUCTION This type of turbine is found suitable when


the available potential energy is high and
Turbines have been used for the centuries the flow available is comparatively low. In
to convert freely available mechanical this turbine, the fluid flow is along the
energy from rivers and wind into useful tangential direction.
mechanical work, usually through a In reaction turbines the available
rotating shaft. The rotating part of a pump potential energy is progressively converted
is called the impeller, whereas the rotating in the turbines rotor and the reaction of the
part of a hydro turbine is called the runner. accelerating water causes the rotation of
When the working fluid is water, the turbo- the wheel. These are again divided into
machines are called hydraulic turbines or radial flow, mixed flow and axial flow
hydro-turbines. When the working fluid is machines. Radial flow machines are found
air, and energy is extracted from the wind, suitable for moderate levels of potential
the machine is properly called a wind energy and medium quantities of flow. The
turbine. The word windmill should axial machines are suitable for low levels of
technically be applied only when the potential energy and large flow rates. The
mechanical energy output is used to grind potential energy available is generally
grains, as done in ancient times However, denoted as “head available”. With this
most people use the word windmill to terminology plants are designated as “high
describe any wind turbine, whether used to head”, “medium head” and “low head”
grind grain, pump water, or generate plants.
electricity. In coal or nuclear power plants, 9.3 TURBINE EFFICIENCIES
the working fluid is usually steam; hence,
the turbo machines that convert energy 9.3.1 HYDRAULIC EFFICIENCY
from the steam into mechanical energy of a
rotating shaft are called steam turbines. A It is defined as the ratio of the power
more generic name for turbines that produced by the turbine runner and the
employ a compressible gas as the working power supplied by the water at the turbine
fluid is gas turbine. inlet.
ηH =RP / WP
9.2 CLASSIFICATION OF TURBINES Water power produced is given by ρQ g H
Where,
The main classification depends upon the Q is the volume flow rate
type of action of the fluid on the turbine. H is the net or effective head.
These are The power produce by runner is evaluated
1) Impulse turbine by Euler’s equation. This reflects the
2) Reaction Turbine. runner design effectiveness.

In impulse turbine all the potential energy 9.3.2 VOLUMETRIC EFFICIENCY


is converted to kinetic energy in the
Volumetric efficiency is defined as the ratio
nozzles. The impulse provided by the jets is
between the volume of water flowing
used to turn the turbine wheel. The
through the runner and the total volume of
pressure inside the turbine is atmospheric.
water supplied to the turbine. The

© Copyright Reserved by Gateflix.in No part of this material should be copied or reproduced without permission
reduction in water flowing through runner The moment of momentum at inlet
is because of leakage through the clearance  w1r1
= mV
between the runner and casing without The moment of momentum at exit
passing through the runner  w2r2
= mV
9.3.3 MECHANICAL EFFICIENCY
The torque on the rotor equals the rate
The power produced by the runner is of change of moment of momentum of
always greater than the power available at the fluid as it passes through the runner.
the turbine shaft. This is due to mechanical Torque, T  m Vw1 r1  Vw2 r2 
losses at the bearings, friction.
P  m Vw1 r1  Vw2 r2  
ηm =Shaft power / Runner power P  m Vw1 r1    Vw2 r2   
9.3.4 OVERALL EFFICIENCY =  w1U1 ± Vw2U2 ]
P m[V
Power per unit weight is given
This is the ratio of power output at the = [Vw1U1 + Vw2U2 ]/ g
shaft and power input by the water at the
turbine inlet. 9.5 PELTON TURBINE
η0 = Shaft Power / Water Power
Also the overall efficiency is the product of The hydraulic turbine of the impulse type
the other three efficiencies defined in common use, is named after an American
ηo =ηH ηm ηv engineer Laster A Pelton. Therefore this
machine is known as Pelton turbine or
Pelton wheel. It is an efficient machine
9.4 POWER DEVELOPED BY TURBINE
particularly suited for high heads. The
(EULER’S EQUATION)
rotor consists of a large circular disc or
wheel on which a number (usually less
The fluid velocity at the turbine entry and
than 15) of buckets are spaced uniformly
exit can have three components in the
round its periphery. The high speed jet
tangential, axial and radial directions of the
then impinges on bucket-shaped vanes that
rotor. This means that the fluid momentum
transfers energy to the turbine shaft. The
can have three components at the entry
buckets of a Pelton wheel are designed so
and exit. This also means that the force
as to split the flow in half, and turn the flow
exerted on the runner can have three
nearly 165° around.
components. Out of these the tangential
force only can cause the rotation of the
runner and produce work. The axial
component produces a thrust in the axial
direction, which is taken by suitable thrust
bearings. The radial component produces a
bending of the shaft which is taken by the
journal bearings.

U1    is the tangental velocity at entry,


U 2 is the tangential velocity at exit, Pelton turbine
Vw1 is the tangential component of the
9.5.1 POWER DEVELOPED
absolute velocity of the fluid at inlet,
 Vw 2 is the tangential component of the
absolute velocity of the fluid at exit

© Copyright Reserved by Gateflix.in No part of this material should be copied or reproduced without permission
The bucket splits the jet into equal parts =
and changes the direction of the jet by
P m   V1 ± k ( V1 − u ) cos β − u  u
  { }
about 165° . The velocity diagram for Pelton Whether case1 or case 2 the power can be
turbine is shown in figure. The force acting written as
on blade is equal to rate of change of =
momentum of fluid passing through runner
P m   V1 ± k ( V1 − u ) cos β − u  u
  { }
=F m(V w1 ± Vw2 ) P= m  V1 − u 1 + k cosβ  u
  ( )( )
In case of blades mounted around
periphery of wheel, m  is given by ρAV1 at 9.5.2 HYDRAULLIC EFFICIENCY
entry. As already discussed in previous
chapter the mass flow rate for single blade m ( V1 – u )(1 + k cos β )  u
is given by ( V1 − U1 ) ηH =  

mgh
V1 = Cv 2gH  = 1 / 2mV
mgh  12
T  m Vw1  Vw2  r For maximum ηH ,
dηH
 w1 ± Vw2 )rω
P = m(V =0
du
U = V1 / 2
Substituting the value U = V1 / 2 , maximum
efficiency is
1 + k cos β
ηH =
2
Speed Ratio:
u
ku =
2gH
Vr2 cos β2 > u Vr2 cos β2 < u

9.6 REACTION TURBINE


Velocity Triangle of Pelton wheel

When,
Vr2cos β > u,Vw1 + Vw2
Vr2cos β > u,Vw1 − Vw2
From velocity triangle,
Vw1  V1

Vr1= V1 − u

The relative velocity Vr 2 becomes slightly


less than Vr1 mainly because of the friction
in the bucket.
Vr2 = kVr1
In the ideal case
= =
K 1,Vr2
Vr1
=
Vw2 Vr2 cos β − u
=
Vw2
k(V1 − u)cos β − u

© Copyright Reserved by Gateflix.in No part of this material should be copied or reproduced without permission
The other main type of energy-producing The runner is circular disc and has the
hydro-turbine is the reaction turbine, blades fixed on one side. In high speed
which consists of fixed guide vanes called runners in which the blades are longer a
stay vanes, adjustable guide vanes called circular band may be used around the
wicket gates, and rotating blades called blades to keep them in position. The shape
runner blades. Flow enters tangentially at of the runner depends on the specific speed
high pressure, is turned toward the runner of the unit. These are classified as
by the stay vanes as it moves along the i) slow runner
spiral casing or volute, and then passes ii) medium speed runner
through the wicket gates with a large iii) high speed runner
tangential velocity component. Momentum iv) very high speed runner
is exchanged between the fluid and the
runner as the runner rotates, and there is a 9.6.14 DRAFT TUBE
large pressure drop. Unlike the impulse After passing through the turbine runner,
turbine, the water completely fills the the exiting fluid still has kinetic energy, and
casing of a reaction turbine. For this perhaps swirl. To recover some of this
reason, a reaction turbine generally kinetic energy (which would otherwise be
produces more power than an impulse wasted), the flow enters an expanding area
turbine of the same diameter, net head, and diffuser called a draft tube, which turns
volume flow rate. the flow horizontally and slows down the
flow speed, while increasing the pressure
9.6.1 COMPONENTS OF REACTION prior to discharge into the downstream
TURBINE water

9.6.11 SPIRAL CASING

The spiral casing surrounds the runner


completely. Its area of cross section
decreases gradually around the
circumference. This leads to uniform
distribution of water all along the
circumference of the runner. Water from
the penstock pipes enters the spiral casing
and is distributed uniformly to the guide The head recovered by the draft tube will
blades placed on the periphery of a circle. equal the sum of the height of the turbine
The casing should be strong enough to exit above the tail water level and the
withstand the high pressure. difference between the kinetic head at the
inlet and outlet of the tube less frictional
9.6.12 GUIDE BLADES loss in head
V12 V22 
Water enters the guide blades Hd  H   hf
2g
circumferentially. the guide blades guide
the water in the proper direction. They also Different types of draft tubes are used as
act as nozzle. The velocity if water is the location demands. These are
increased by the pressure drop in the i) Straight diverging tube
passage. ii) Bell mouthed tube
iii) Elbow shaped tubes of circular exit or
9.6.13 RUNNER rectangular exit.

9.6.2 VELOCITY TRIANGLE OF REACTION

© Copyright Reserved by Gateflix.in No part of this material should be copied or reproduced without permission
TURBINE accelerating component and this will be
present only in the reaction turbines.
Reaction of turbine is given by
(
1 2
2
) ( )
U1 – U 22 + Vr22 – Vr12 
R=
(
1 2
2
) ( ) ( )
V1 – V22 + U12 – U 22 + Vr22 – Vr12 

=
For b 90,= Vw 2 0 and V2 = Vf 2
cot α
R= 1−
2 ( cot α − cot θ )

Vf 1 & Vf 2 is flow velocity 9.7 FRANCIS TURBINE


V1 & V2 is absolute velocity The inward flow reaction turbine having
Vr1 & Vr2 is relative velocity radial discharge is known as francis
turbine. Since the discharge is radial at
outlet, the whirl velocity at outlet will be
9.6.3 POWER DEVELOPED BY
zero. Hence the work done by water on the
REACTION TURBINE
runner per second will be
P=m  ( Vw1U1 )
1) Torque developed by reaction turbine is
given by
2) T  m Vw1 r1  Vw2 r2  9.8 KAPLAN TURBINE
m = ρQ
The popular axial flow turbines are the
Kaplan turbine and propeller turbine. In
3) Power is given by propeller turbine the blades are fixed. In
= P m ( Vw1r1  ± Vw2r2 ) ω the Kaplan turbines the blades are
Or mounted in the boss in bearings and the
 ( Vw1U1  ± Vw2U2 )  blades are rotated according to the flow
=P m
conditions by a servo mechanism
maintaining constant speed. In this way a
4) Discharge is given by constant efficiency is achieved in these
Q  V f1  D1b1  V f2  D2b2 turbines. The system is costly and where
Q is volume flow rate constant load conditions prevail, the
b is the thickness simpler propeller turbines are installed.
D is the diameter

5) Speed of rotation
U1 =
πD1N / 60     U2 =
πD2N / 60
N is rotation per minute
By geometry
  ± Vw2U2 1 / 2 ( V12 – V22 ) + (U12 – U22 ) + ( Vr22 – Vr12 )
Vw1U1=

Where,
½ (V12 – V2 2 ) is the dynamic component
of work done ½ ( U12 – U 2 2 ) is the
centrifugal component of work and this 9.8.1 SPECIFIC SPEED
will be present only in the radial flow
machines ½ (Vr2 2 – Vr12 ) is the The specific speed is given by

© Copyright Reserved by Gateflix.in No part of this material should be copied or reproduced without permission
N P load fluctuation vary considerably. In spite
Ns = of design methodology, it is found the
H 5/4
Where, designs have to be validated by actual
P is power in KW testing. In addition to the operation at the
N is speed of rotation in rpm design conditions, the characteristics of
H is head in ‘m’ operation under varying input & output
conditions should be established. It is
9.9 SPECIFIC SPEED SIGNIFICANCE found almost impossible to test a full size
unit under laboratory conditions. In case of
Specific speed does not indicate the speed variation of the operation from design
of the machine. It can be considered to conditions, large units cannot be modified
indicate the flow area and shape of the or scrapped easily. We define
runner. When the head is large, the velocity dimensionless groups (Pi groups) for
when potential energy is converted to turbines.
kinetic energy will be high. The flow area Dimensional variables:
required will be just the nozzle diameter. • gravity times net head ( gH )
This cannot be arranged in a fully flowing • volume flow rate (V)
type of turbine. Hence the best suited will • diameter of the runner blades(D)
be the impulse turbine. When the flow • runner rotational speed (N)
increases, still the area required will be • output power (P)
unsuitable for a reaction turbine. So multi • fluid density. (ρ)
jet unit is chosen in such a case. As the head
reduces and flow increases purely radial The dimension less parameters obtained are
flow reaction turbines of smaller diameter 1. The head coefficient, gH / N 2 D 2
can be chosen. As the head decreases still 2. The flow coefficient, Q / ND3
further and the flow increases, wider rotors 3. The power coefficient, P / ρN 3 D5
with mixed flow are found suitable. The
diameter can be reduced further and the 9.10.1 UNIT QUANTITIES
speed increased up to the limit set by
mechanical design. As the head drops The dimensionless constants can also be
further for the same power, the flow rate used to predict the performance of a given
has to be higher. Hence axial flow units are machine under different operating
found suitable in this situation. conditions. As the linear dimension will be
Dimensionles Dimensionles Types of turbine having the same, the same will not be taken into
specific speed specific speed the best efficient at account in the calculation. Thus Head
range in SI system these values
0.015-0.053 8-29 Single jet pelton turbine
coefficient is given by
0.047-0.072 26-40 Twin jet pelton turbine gH1 / N12 D 2 = gH 2 / N 2 2 D 2
0.72-0.122 40-67 Multiple jet pelton turbine Or
0.122-1.819 67-450 Radial flow turbine
francistype(H<350m) H1 / N12 = H 2 / N 2 2
0.663-1.66 364-910 Axial flow haplan Flow coefficient is given by
turbine(H<60m)
Q1 / N1D3 = Q 2 / N 2 D3
9.10 MODEL TESTING (DIMENSIONLESS Or
TURBINE PARAMETERS) Q1 / N1 = Q 2 / N 2
Using previous two relations
Hydraulic turbines are mainly used for Q1 Q
 12
power generation and because of this, 1
H1 2 H 2 2
these are large and heavy. The operating
conditions in terms of available head and Where,

© Copyright Reserved by Gateflix.in No part of this material should be copied or reproduced without permission
Q / H1/2 = constant known as unit discharge ∴ Vr = V1 − u1 = 80.86 − 36.387
1

N / H1/2 = constant is known as unit speed = 44.473m/ s


P / H 3/2 = constant known as unit power Also VW1 = V1 = 80.86 m/ s
From outlet velocity triangle , we have
Example: Vr2 = Vr1 = 44.473
The penstock supplies water from a Vr2 cos φ = u 2 + VW2
reservoir to the Pelton wheel with a gross
Or
head of 500m. One third of the gross head
44.473cos15° = 36.387 + VW2
is lost in friction in the penstock. The rate
of flow of water through the nozzle fitted at Or
the end of the penstock is 2.0m3 /s. The VW2 = 44.473cos15° − 36.387 = 6.57 m/ s
angle of deflection of the jet is 165° Work done by the jet on the runner per
Determine the power given by the water to second is given by equation as
the runner and also hydraulic efficiency of Fx = ρ aV1  Vw1 − Vw 2  × u = ρ Q  Vw1 + Vw 2  × u
the Pelton wheel .Take speed ratio=0.45
and Cv =1.0 = 1000× 2.0× [80.86 + 6.57 ] ×36.387 =
Solution: = 6362630 Nm/ s
Given: ∴ Power given by the water to the runner
Gross head, H g = 500 m in kW
Work done per second 6362630
= = = 6362.63kW
1000 1000
Hydraulic efficiency of the turbine is given
by equation as
2  Vw + Vw  × u 2 [80.86 + 6.57 ] ×36.387
ηh =  1
2
2
=
V1 80.86×80.86
= 0.9731 or 97.31%

Example:
A Pelton wheel is working under a gross
Head lost in friction head of 400m. The water is supplied
through penstock of diameter 1 m and
H g 500
hf = = = 166.7 m length 4km from reservoir to the Pelton
3 3 wheel. The co-efficient of friction for the
∴ Net head , penstock is given as .008.The jet of water of
H = H g − h f = 500 − 166.7 = 333.30 m diameter 150mm strikes the buckets of the
Discharge, Q = 2.0 m3 / s wheel and gets deflected through an angle
Angle of deflection = 165° of 165° . The relative velocity of water at
∴ Angle, φ = 185° − 165° = 15° outlet is reduced by 15% due to friction
between inside surface of the bucket and
Speed ration = 0.45
water. If the velocity of the buckets is 0.45
Co-efficient of velocity, C v = 1.0 times the jet velocity at inlet and
Velocity of jet, mechanical efficiency as 85%
V1 = C v 2 gH = 1.0× 2×9.81×333.3 = determine:
= 80.86 m/ s i) Power given to the runner
ii) Shaft power,
Velocity of wheel, u = speed ratio× 2 gH iii) Hydraulic efficiency & overall efficiency
Or u = u1 = u 2 = 0.45× 2×9.81×333.3 = 36.387 m/ s Solution:
Given:

© Copyright Reserved by Gateflix.in No part of this material should be copied or reproduced without permission
Gross head, H g = 400 m  0.0033V12  0.51V12 or 400=0.0543V12
Diameter of penstock, D = 1.0 m 400
Length of penstock ∴ V1 = = 85.83m/ s
.0543
L = 4 km = 4×1000 = 4000 m Now velocity of bucket,
Co-efficient of friction , f = .008 u1 = 0.45 V1 = 0.45×85.83 = 38.62 m/ s
Diameter of jet, d = 150 m = 0.15 m
From inlet velocity triangle,
Angle of deflection = 165° Vr1 = V1 − u1 = 85.83 − 38.62 = 47.21m/ s
∴ Angle, F = 180° − 165° = 15°
Vw1 = V1 = 85.83 m/ s
Relative velocity at outlet, Vr = 0.85 Vr
From outlet velocity triangle
2 1

Velocity of bucket, u = 0.45× Jet velocity Vr2 = 0.85× Vr1 = 0.85× 47.21 = 40.13 m/ s
Mechanical efficiency, ηm = 85% = 0.85 Vw 2 = Vr2 cos φ − u 2 = 40.13cos15° − 38.62
Let V* = Velocity of water in penstock, and = 0.143m/ s ( u = u1 = u 2 = 38.62 )
V1 = Velocity of jet of water.
Discharge through nozzle is given as
Q = Area of jet× Velocity ofjet = a× V1
π 2 π
d × V1 = (.15 ) ×85.83 = 1.516 m3 / s
2

4 4
Work done on the wheel per second is
given by equation as
= ρ aV1  Vw + Vw  × u = ρ Q  Vw + Vw  × u
1 2 1 2

= 1000×1.516 [85.83 +.143] ×38.62 = 5033540 Nm/ s


i) Power given to the runner in kW
Work done per second 5033540
= = =
1000 1000
Using continuity equation, we have = 5033.54 kW
Area of penstock × velocity of water in
penstock ii) Using equation for mechanical
= Area of jet ×V1 efficiency,
π 2 π Power at the shaft S.P.
Or D × V* = d 2 × V1 ηm = =
4 4 Power given to the runner 5033.54
2 ∴ S.P = ηm ×5033.54 = 0.85×5033.54
d 0.152
∴V * = × V1 = × V1 = .0225V1 = 4278.5 kW
D2 1.02
Applying Bernoulli’s equation to the free iii) Hydraulic efficiency is given by
surface of water in the reservoir and outlet equation as
of the nozzle, we get 2  Vw + Vw  × u
V2 ηh =  1 2 2 
H g = Head lost due to friction+ 1 V1
2g 2 [85.83 +1.43] ×38.62
Or =
85.83×85.83
= 0.9014 = 90.14%
*2 2 *2 2
400 =
4 fLV
+
V
=
4×.008× 4000× V
1
+
V 1 Overall efficiency is given by equation
D× 2 g 2 g 1.0× 2×9.81 2g as
Substituting the value of V * from equation η0 = ηm ×ηh = 0.85×.9014 =
. we get = 0.7662 or 76.62%
4×.008× 4000 V12
× ( 0.0225 V1 ) +
2
400 =
2×9.81 2g

© Copyright Reserved by Gateflix.in No part of this material should be copied or reproduced without permission
Example: Here Vr2 cos φ is less than u2 . Hence
A 137 mm diameter jet of water issuing velocity triangle at outlet will be as shown
from a nozzle impinges on the buckets of a in Fig
Pelton wheel and the jet is deflected ∴ Vw = u 2 − Vr cos φ = 40.75 − 37.092 = 3.658 m/ s
through an angle of 165° by the buckets 2 2

i) Force exerted by jet on buckets in


The head available at the nozzle is 400m.
tangential direction is given by ,
Assuming co-efficient of velocity as 0.97,
speed ratio as 0.46, and reduction in Fx = ρ aV1  Vw − Vw 
1 2

relative velocity while passing through (Here –ve sign is taken as Vw and Vw
buckets as 15% find:
1 2

are in the same direction )


i) The force exerted by the jet on buckets ∴
in tangential direction, Fx = 1000× 0.01474×85.93 ( 85.93 − 3.658 ) N = 104206 N
ii) The power developed ii) Power developed is given by,
Solution: Fx × u 104206× 40.75
Given: Power =
1000
kW =
1000
= 4246.4 kW
Given of jet,
d = 137 mm = 0.137 m Example:
∴ Area of jet, a = 0.01474m2 An inward flow reaction turbine has
Angle of deflection = 165° external and internal diameters as 0.9m
∴ Angle, φ = 180° − 165 = 15° and 0.45m respectively .The turbine in
Head of water, H = 400 m running at 200r.p.m. and width of turbine
Co-efficient of velocity, C v = 0.97 at inlet is 200mm. The velocity of flow
Speed ratio = 0.46 through the runner is constant and is equal
Relative velocity at outlet to 1.8m/s. The guide blades make an angle
= 0.85× relative velocity at inlet of 10° the tangent of the wheel and the
discharge at the outlet of the turbine is
Or Vr = 0.85 Vr2 1 radial. Draw the inlet and outlet velocity
Now velocity of jet, triangles and determine
V1 = C v 2 gH = 0.97 2×9.81× 400 = 85.93m/ s i) The absolute velocity of water at inlet of
runner
ii) The velocity of whirl at inlet,
iv) The relative velocity at inlet.
vi) Mass of water flowing through the
runner per second.
v) Width of the runner at outlet,
vii) Head at the inlet of the turbine
viii)Power developed and hydraulic
efficiency of the turbine
Solution:
Speed ratio =
u1
or 0.46 =
u1
2 gH 2×9.81× 400 Given:
External Dia: D1 = 0.9 m
∴ u1 = 0.46× 2×9.81× 400 = 40.75 m/ s
Internal Dia., D 2 = 0.45 m
Hence Vr = V1 − u1 = 85.93 − 40.75 = 45.18 m/ s
1
Speed N = 200 r .p.m
And Vr = 0.85 Vr = 0.85× 45.18 = 38.40 m/ s
1 1
Width at inlet , B1 = 200 mm = 0.2 m
For Pelton turbine, u1 = u 2 = u = 40.75 m/ s
Velocity of flow, Vf = Vf = 1.8 m/ s
Vr cos φ = 38.40× cos15° = 37.092 1 2
2
Guide of flow, α = 10°
Discharge at outlet = Radial

© Copyright Reserved by Gateflix.in No part of this material should be copied or reproduced without permission
∴ β = 90° andVW = 0 2
∴ B2 = D1B1 =
0.90× 0.20
= 0.40 m = 400 mm
D2 0.45
vi) Mass of water flowing through the
runner per second. The discharge,
Q = π D1B1Vf1 = π × 0.9× 0.20×1.8 = 1.0178 m3 / s
∴ Mass
= ρ × Q = 1000×1.0178 kg/ s = 1017.8 kg/ s
vii) Head at the inlet of turbine, i.e., H Using
equation, we have
H−
V22 1
( 1
)
= Vw1 u1 ± Vw 2 u 2 = Vw1 u1
2g g g
( )
( hereV w2 =0 )
Tangential velocity of wheel at inlet and
outlet are:
Vw1 u1 V22
H 
π D1 N π ×.9× 200 g 2g
u1 = = = 9.424 m/ s
60 60 10.927 9.424 1.82
π D 2 N π ×.45× 200 =
9.81

29.81
 V2  V f2 
u2 = = = 4.712 m/ s
60 60 = 9.805 + 0.165 = 9.97m
i) Absolute velocity of water at inlet of the viii) Power developed, i.e.,
runner i.e. V1 . From inlet velocity work done per second on runner
triangle, V1 sin α =Vf 1 P=
1000
V1 =
Vf1
=
18
= 10.365 m/ s ρ Q  Vw1 u1 
sin α sin10° =
1000
ii) Velocity of whirl at inlet , ie., Vw 1.0178×10.207×9.424
1
= 1000× = 97.9 kW
Vw1 = V1cos α = 10.365× cos10° = 10.207 m/ s 1000
Hydraulic efficiency is given by
iii) Relative velocity at inlet, i.e. Vr
1 equation as
( ) = 1.82 + (10.207 − 9.424 ) Vw1 u1
2
Vr1 = Vr31 + Vw1 − u1
2
10.207×9.424
ηh = = = 0.9834 = 98.34%
gH 1000
= 3.24 + 613 = 1.963m/ s
iv) The runner blade angles means the Example:
angle θ and Φ An outward flow reaction turbine has
Now internal and external diameters of the
tan θ =
Vf1
=
1.8
= 2.298
runner as 0.6m and 1.2m respectively. The
(V w1 − u1 ) (10.207 − 9.424 ) guide blade angle is 15° and velocity of flow
through the runner is constant and equal to
∴ θ = tan -1 2.298 = 66.48° or 66°29.'
4m/s. If the speed of the turbine is 2000
From outlet velocity triangle, we have
rpm, head on the turbine is 10m and
Vf 1.8
tan φ = = = tan 20.9°
2 discharge at outlet is radial, determine:
u 2 4.712 i) The runner vane angles at inlet & outlet
∴ φ = 20.9° or 54.4 ' ii) Work done by the water on the runner
v) Width of runner at outlet, i.e, B2 per second per unit weight of water
striking per second,
From equation, we have
iii) Hydraulic efficiency, and
π D1B1Vf = π D 2 B2 Vf or D1B1 = D 2 B2
1 2 iv) The degree of reaction
 V f1  V f2 as V f1  V f2  Solution:
Given:

© Copyright Reserved by Gateflix.in No part of this material should be copied or reproduced without permission
Internal diameter, D1 = 0.6 m iii) Hydraulic efficiency is given by
External diameter D 2 = 1.2 m equation
Vw u1 14.928×6.283
Guide blade angle, α = 15° ηh = 1 = = 0.9561or 95.61%
Velocity of flow, Vf1 = Vf2 = 4 m/ s gH 9.81×10
iv) Given : In this question, the velocity of
Speed N = 200 r .p.m
flow is constant through the runner (ie.,
Head, H = 10 Vf = Vf ) and the discharge is radial at
Discharge at outlet =Radial 1 2

∴ VW = 0.Vf = V2 outlet (ie., β = 90° orVw = 0),


2
the
2 2
degree of reaction (R ) is given by
equation
cot α
R = 1−
2 ( cot α - cot θ )
Here α = 13.928° and θ = 41.09° ( calculated )
Substituting the value of α and θ, we
get
cot13.928° 4.032
R = 1− = 1−
2 ( cot13.928° − cot 41.09° ) 2 ( 4.032 − 1.146 )
= 1 − 0.698 = 0.302  0.3
Tangential velocity of runner at inlet and For Francis turbine , the degree of
outlet are: reaction varies from 0 to 1 i.e., 0 ≤ R ≤ 1
π D1 N π × 0.6× 200
u1 = = = 6.283m/ s
60 60 Example:
π D 2 N π ×1.2× 200 A pelton turbine develops 3000kW under
u2 = = = 12.566 m/ s
60 60 a head of 300m. The overall efficiency of
From the inlet velocity triangle, turbine is 83%. If speed ratio =0.46,
Vf Cv = 0.98 and specific speed is 16.5, then
tan α = 1
Vw find:
1

Vf1 i) Diameter of the turbine, and


∴ Vw = =
4.0
= 14.928 m/ s ii) Diameter of the jet
1
tan α tan15°
Solution:
i) Runner vane Angles at inlet and outlet
Given:
are θ and φ
Power P = 3000 kW
Vf1 4.0
tan θ = = = 0.4627 Net head H = 300 m
Vw1 − u1 (14.928 − 6.283) Overall efficiency, ηo = 83% or 0.83
θ = tan −1 .4627 = 24.83 or 24°49.8' Speed ratio = 0.46
From outlet velocity triangle, Value of Cv' = 0.98
Vf 4.0 Specific speed *, N s = 16.5
tan φ = 2 = = 0.3183
u2 12.566 Using equation,
∴ φ = tan −1 .3183 = 17.65° or17°39.4 ' Ns =
N P
or N =
Ns H5/4 16.5×3005/4
= = 375r.p.m
H 5/4 P 3000
ii) Work done by water per second per
unit weight of water striking per second The velocity (V) at the outlet of nozzle is
given by
( Vw = 0 )
1
= Vw1 u1 V = C v 2× g× H = 0.98 2×9.81×300 = 75.1m/ s
g 2

Now speed ratio,


1
= × 14.928 × 6.283 =9.561Nm / N
9.81

© Copyright Reserved by Gateflix.in No part of this material should be copied or reproduced without permission
u P1 P2
=
2 gH
or u = Speed ratio× 2 gH Also we have 3/2
= 3/2
H1 H2
= 0.46× 2×9.81×300 = 34.95 m/ s 3/ 2 3/ 2
∴ P2 = P1H3/22 = 9000×18 =
687307.78
i) Diameter of the turbine (D) using, H1 3/ 2
30 164.316
π DN 60× u 60×34.95
= 4182.84 kW
u= or D = = = 1.78 m
60 π ×N π ×37.5
ii) Diameter of the jet (d)
Example:
Let Q = Discharge through turbine in
A pelton wheel is revolving at a speed of
m3 / s 190r.p.m. and develops 5150.25 kW when
Using the relation, working under a head of 220m with an
ηo =
P1
, overall efficiency of 80% Determine unit
 π × g× Q× H 
  speed, unit discharge and unit power. The
 1000 
speed ratio for the turbine is given as 0.47.
where π × g = 1000×9.81N/ m3 for water Find the speed. Discharge and power when
∴ 0.83 = 3000 this turbine is working under a head of
 1000×9.81× Q×300  140m
 
 1000  Solution:
∴ Q= 3000 Given:
= 1.23m3 / s
9.81×300× 0.83 Speed, N1 = 190 r .p.m.
But discharge through a Pelton turbine Power, P1 = 5150.25 kW
is given by,
Q = Area of jet× Velocity Head, H1 = 220 m
π Overall efficiency, ηo = 80% = 0.80
Or 1.23 = d 2 × 75.1
4 Speed ratio = 0.47
4×1.23 New head of water, H 2 = 140 m
∴d= = 0.142 m = 142 mm
Overall efficiency is given by
π × 75.1
P1 1000× P1
ηo = =
ρ × g× Q1× H1 ρ × g× Q1× H1
Example:
1000
A turbine develops 9000 kW when running
at 10r.p.m. The head on the turbine is ∴ Q1 = 1000× P1 = 1000×5150.25 =
h o × ρ ×g× H1 0.80×1000×9.81× 220
30m.If the head on the turbine is reduced
to 18m; determine the speed and power = 2.983m3 / s
developed by the turbine. Unit speed is given by equation
Solution: Nu =
N1
=
19
= 12.81 r .p.m.
Given: H1 220
Power developed P1 = 9000 kW Unit discharge is given by equation
Speed N1 = 100 r .p.m. Qu =
Q1
=
2.983
= 0.201m3 / s
Head, H1 = 30 m H1 220

Let for a head H 2 = 18 m Unit power is given by equation .


P1 5150.25
Speed = N2 Pu = 3/2 = = 1.578 kW
H1 2203/2
Power = P2 When the turbine is working under a new
Using equation, N1 N
= 2 head of 140m, the speed, discharge and
H1 H2 power are given by equation as
N1 H 2 N N
N2 = =
100 18 100× 4.2426
= = 77.46 r.p.m. For speed, 1 = 2
H1 30 5.4772 H1 H2

© Copyright Reserved by Gateflix.in No part of this material should be copied or reproduced without permission
H2 140
 N 2  N1  190  151.6rpm
H1 220

For discharge , Q1 Q
= 2
H1 H2

H2 140
 Q2  Q1  2.983  2.379m3 / s
H1 220

P1 P2
For power, 3/2
= 3/2
H1 H2
3 3
H2 2  140  2
 P2  P1  5150.25   2614.48kW
3  220 
H1 2

© Copyright Reserved by Gateflix.in No part of this material should be copied or reproduced without permission
GATE QUESTIONS

Q.1 If there are m physical quantities of diameter 0.2 m having Darcy’s


and n fundamental dimensions in a friction factor of 0.01. The average
particular process, the number of speed of water in the pipe is 2 m/s.
non-dimensional parameters is If it is to maintain a constant head of
a) m + n b) m × n 5 m in the tank, neglecting other
c) m− n d) m / n minor losses, then absolute discharge
[GATE–2002] pressure at the pump exit is
a) 0.449 bar b) 5.503 bar
Q.2 A centrifugal pump running at 500 c) 44.911 bar d) 55.203 bar
rpm and at its maximum efficiency [GATE–2004]
is delivering a head of 30 m at a flow
rate of 60 litres per minute. If the Q.5 At a hydroelectric power plant site,
rpm is changed to 1000, then the available head and flow rate are
head H in metres and flow rate Q in 24.5 m and 10.1 m3/s respectively. If
litres per minute at maximum the turbine to be installed is
efficiency are estimated to be required to run at 4.0 revolutions
a) H=60, Q=120 b) H=120, Q=120 per second (rps) with an overall
c) H = 60, Q=480 d) H = 120, Q = 30 efficiency of 90%, the suitable type
[GATE–2003] of turbine for this site is
a) Francis b) Kaplan
Q.3 Match List-I with List-II and select c) Pelton d) Propeller
the correct answer using the codes [GATE–2004]
given below the lists :
Q.6 Match List-I with List-II and select
List-I List-II the correct answer using the codes
P.Curtis 1.Reaction steam turbine given below the lists:
Q.Rateau 2. Gas turbine List-I
R.Kaplan 3. Velocity compounding P. Reciprocating pump
S.Francis 4.Pressure compounding Q. Axial flow pump
5.Impulse water turbine R. Microhydel plant
6. Axial turbine
7. Mixed flow turbine
S. Backward curved vanes
8. Centrifugal pump List-II
Codes : 1. Plant with power output below
P Q R S 100 kW
a) 2 1 1 6 2. Plant with power output
b) 3 4 6 7 between 100 kW to 1 MW
c) 1 3 1 5 3. Positive displacement
d) 3 4 7 6 4. Draft tube
[GATE–2003] 5. High flow rate, low pressure ratio
6. Centrifugal pump impeller
Q.4 A centrifugal pump is required to Codes :
pump water to an open water tank P Q R S
situated 4 km away from the a) 3 5 6 2
location of the pump through a pipe b) 3 5 2 6

© Copyright Reserved by Gateflix.in No part of this material should be copied or reproduced without permission
c) 3 5 1 6 Francis turbine is 90°. The blades
d) 4 5 1 6 are so shaped that the tangential
[GATE–2004] component of velocity at blade
outlet is zero. The flow velocity
Q.7 In a Pelton wheel, the bucket remains constant throughout the
peripheral speed is 10 m/s, the blade passage and is equal to half of
water jet velocity is 25 m/s and the blade velocity at runner inlet.
volumetric flow rate of the jet is 0.1 The blade efficiency of the runner is
m3/s. If the jet deflection angle is a) 25% b) 50%
120° and the flow is ideal, the power c) 80% d) 89%
developed is [GATE–2007]
a) 7.5 kW b) 15.0 kW
c) 22.5 kW d) 37.5 kW Q.11 A model of a hydraulic turbine is
[GATE–2006] tested at a head of 1/4th of that
under which the full scale turbine
Q.8 A large hydraulic turbine is to works. The diameter of the model is
generate 300 kW at 1000 rpm under half of that of the full scale turbine.
a head of 40 m. For initial testing, a If N is the RPM of the full scale
1:4 scale model of the turbine turbine, the RPM of the model will be
operates under a head of 10 m. The a) N/4 b) N/2
power generated by the model (in c) N d) 2N
kW) will be [GATE–2007]
a) 2.34 b) 4.68
c) 9.38 d) 18.75 Q.12 Match List-I with List-II and select
[GATE–2006] the correct answer using the codes
given below the lists :
Q.9 A horizontal-shaft centrifugal pump List-I
lifts water at 65℃. The suction P. Centrifugal compressor
nozzle is one meter below pump Q. Centrifugal pump
center line. The pressure at this R. Pelton wheel
point equals 200 kPa gauge and S. Kaplan turbine
velocity is 3 m/s. Steam tables show List-II
saturation pressure at 65℃ is 25 1. Axial flow
kPa, and specific volume of the 2. Surging
saturated liquid is 0.001020 m3 /kg. 3. Priming
The pump Net Positive Suction Head 4. Pure impulse
(NPSH) in meters is Codes :
P Q R S
a) 2 3 4 1
b) 2 3 1 4
c) 3 4 1 2
d) 1 2 3 4
[GATE-2007]

Q.13 Water, having a density of 1000


a) 24 b) 28 kg/m3, issues from a nozzle with a
c) 28 d) 30 velocity of 10 m/s and the jet
[GATE–2006] strikes a bucket mounted on a
Q.10 The inlet angle of runner blades of a Pelton wheel. The wheel rotates at

© Copyright Reserved by Gateflix.in No part of this material should be copied or reproduced without permission
10 rad/s. The mean diameter of the fluid and U denotes the blade
wheel is 1 m. The jet is split into velocity. Subscripts 1 and 2 refer to
two equal streams by the bucket, inlet and outlet respectively. If V2 =
such that each stream is deflected W1 and V1 = W2 , then the degree of
by 120° as shown in the figure. reaction is
Friction in the bucket may be
neglected. Magnitude of the torque
exerted by the water on the wheel,
per unit mass flow rate of the
incoming jet, is

a) 0 b) 1
c) 0.5 d) 0.25
[GATE–2012]

Q.17 In order to have maximum power


from a Pelton turbine, the bucket
speed must be
a) equal to the jet speed
b) equal to half of the jet speed.
a) 0 (N-m)/(kg/s)
c) equal to twice the jet speed
b)1.25(N-m)/(kg/s)
d) independent of the jet speed.
c) 2.5 (N-m)/(kg/s)
[GATE–2013]
d)3.75(N-m/(kg/s)
[GATE–2008]
Q.18 Kaplan water turbine is commonly
used when the flow through its
Q.14 A hydraulic turbine develops 1000
runner is
kW power for a head of 40 m. If the
a) axial and the head available is
head is reduced to 20 m, the power
more than 100 m
developed (in kW) is
b) axial and the head available is
a) 177 b) 354
less than 10m
c) 500 d) 707 c) radial and the head available is
[GATE–2010]
more than 100 m
d) mixed and the head available is
Q.15 A pump handing a liquid raises its
about 50 m
pressure from 1 bar to 30 bars.
[GATE–2014 (4)]
Take the density of the liquid as 990
kg/m3. The isentropic specific work
Q.19 An ideal water jet with volume flow
done by the pump in kJ/ kg is
rate of 0.05 m3/s strikes a flat plate
a) 0.10 b) 0.30
placed normal to its path and exerts
c) 2.50 d) 2.93
a force of 1000 N. Considering the
[GATE–2011]
density of water as 1000 kg/m3, the
diameter (in mm) of the water jet
Q.16 The velocity triangles at the inlet
is___
and exit of the rotor of a turbo [GATE–2014(1)]
machine are shown. V denotes the
absolute velocity of the fluid, W
Q.20 At the inlet of an axial impulse
denotes the relative velocity of the
turbine rotor, the blade linear speed

© Copyright Reserved by Gateflix.in No part of this material should be copied or reproduced without permission
is 25 m/s, the magnitude of absolute required to hold the plate is
velocity is 100 m/s and the angle _________
between them is 25°. The relative
velocity and the axial component of
velocity remain the same between
the inlet and outlet of the blades.
The blade inlet and outlet velocity
triangles are shown in figure.
Assuming no losses, the specific
work (in J/kg) is ___.

[GATE–2017(2)]

Q.24 Which one of the following statement


[GATE–2014(3)] is TRUE?
a) Both Pelton and Francis turbines
Q.21 Which of the following statements are are impulse turbines.
TRUE, when the cavitation parameter b) Francis turbine is a reaction turbine
σ = 0? but Kaplan turbine is an impulse
i. the local pressure is reduced to turbine.
vapor pressure c) Francis turbine is an axial – flow
ii. cavitation starts reaction turbine.
iii. boiling of liquid starts d) Kaplan turbine is an axial – flow
iv. cavitation stops
reaction turbine.
a) i, ii and iv b) only ii and iii
c) onlyi and iii d) i, ii and iii [GATE–2017(2)]
[GATE–2015(3)]
Q.25 For a Pelton wheel with a given water
Q.22 Consider two hydraulic turbines jet velocity, the maximum output
having identical specific speed and power from the Pelton wheel is
effective head at the inlet. If the obtained when the ratio of the bucket
speed ratio (N1/N2) of the two speed to the water jet speed is
turbines is 2, then the respective _______ (correct to two decimal
power ratio (P1/P2) is _____________. places).
[GATE–2016(1)] [GATE–2018(1)]
Q.23 A 60 mm-diameter water jet strikes a
plate containing a hole of 40mm
diameter as shown in the figure. Part Q.26 Select the correct statement for 50%
of the jet passes through the hole reaction stage in a steam turbine.
horizontally, and the remaining is a) The rotor blade is symmetric.
deflected vertically. The density of b) The stator blade is symmetric.
c) The absolute inlet flow angle is
water is 1000kg / m3 . If velocities are
equal to absolute exit flow angle.
as indicated in the figure, the
d) The absolute exit flow angle is
magnitude of horizontal force (in N)
equal to inlet angle of rotor blade.

© Copyright Reserved by Gateflix.in No part of this material should be copied or reproduced without permission
[GATE–2018(2)]

Q.27 A test is conducted on a one-fifth scale


model of a Francis turbine under a
head of 2 m and volumetric flow rate
of 1 m3 / s at 450 rpm. Take the water
density and the acceleration due to
gravity as 103 kg / m3 and 10 m / s2 ,
respectively. Assume no losses both
in model and prototype turbines. The
power (in MW) of a full sized turbine
while working under a head of 30 m is
_______ (correct to two decimal
places).
[GATE–2018(2)]

ANSWER KEY:
1 2 3 4 5 6 7 8 9 10 11 12 13 14
(c) (b) (b) (b) (a) (b) (c) (a) (a) (c) (c) (a) (d) (b)
15 16 17 18 19 20 21 22 23 24 25 26 27
(d) (c) (b) (b) 56.43 3280.5 (d) 0.25 628.32 (d) 0.5 (d) 29.05

© Copyright Reserved by Gateflix.in No part of this material should be copied or reproduced without permission
EXPLANATIONS

Q.1 (c) Q.4 (b)


From Buckingham’sπ -theorem, Given: L = 4 km = 4×1000 = 4000 m.
“If there are m variables d = 0.2 m, f = 0.01, V = 2m/s.
(Independent and dependent Head loss due to friction in the pipe,
variables)in a physical phenomenon fLV 2 0.01× 4000 × ( 2 )
2

and if these variables contain n =


hf = = 40.77m
fundamental dimensions (M, L, T) 2gd 2 × 9.81× 0.2
then variables are arranged into (m- of water.
n) dimensionless terms. Now total pressure (absolute
discharge pressure) to be supplied
Q.2 (b) by the pump at
πDN exit = Pressure loss by pipe +
=
Speed, U = 2gH pressure head of tank +
60
Atmospheric pressure head Total
H pressure,
∴ N∝
D p= ρghf + ρgH +ρghatm= 1000 × 9.81
For constant diameter, [40.77 + 5 + 10.3]
N ∝ √H = 5.503 × 105 N/m2
H2  N2 
2 = 5.503 bar
∴ = 
H1  N1  Q.5 (a)
H 2  1000 
2
Given H = 24.7 m, Q = 10.1 m3/s,
⇒ =  ηo = 90 %.
30  500 
∴ H2 = 30 × 4 = 120m N = 4 rps = 4 × 60 = 240 rpm.
We also know that specific speed Shaft power in kW
η0 =
will be constant. Water Power in kW
Q1 Q P
∴ = 32 =
 p×g×Q× H 
3
D1 N1 D 2 N 2
 
N   1000 
∴ = Q 2  2  × Q1 [Diameter is p×p×g×Q×H
 N1  P=
constant] 1000
0.90 ×1000 × 9.81×10.1× 24.5
 1000  P=
=Q2   × 60 1000
 500 
= 2184.74 kW
Q 2 = 120 lit/min
For turbine Specific speed
Q.3 (b) N P
Ns = 5/4
Curtis→Velocity compounding H
Rateau→ Pressure compounding 240 2184.75
Kaplan → Axial flow turbine = = 216.54
24.55/4
Francis →Mixed flow turbine Hence 51<Ns<255 for Francis
turbine

© Copyright Reserved by Gateflix.in No part of this material should be copied or reproduced without permission
Q.10 (c)
Q.6 (b) Given figure shows the velocity
So, correct pairs are triangle for the pelton wheel.
P-3, Q-5, R-2, S-6
Q.7 (c)
The velocity triangle for the pelton
wheel is given below.

Velocity Triangle For Francis


Turbine
Jet deflection angle = 1200 Given : Flow Velocity at Inlet Vf1 =
ϕ = 1800 – 1200 = 600
flow velocity at outlet Vf2
Power developed by pelton wheel
= ρQ(V ̶ u)( 1+ cos∅ )u V= V=
u1
(blade velocity)
=1000×0.1×( 25 ̶10)(1+cos60 ͦ)×10
f1 f2
2
= 22500 W V2 = Vf2
= 22.5 kW
θ =900
From inlet velocity triangle,
Q.8 (a) 2
u 
For similar turbines specific power V12 = ( Vf1 ) + ( Vf1 ) =  1  + ( u1 ) = u12
2 2 2 5
will be same  2 4
N m Dm N p Dp
= v −v
2 2
Blade efficiency = 1 2 2 ×100
Hm Hp v1
⇒Nm = 2000 5 / 4u12 − u12 / 4
Now specific speed will be same = × 100 = 80%
5 / 4u12
N P N p Pp
∴ m 5/4 m =
Hm H 5/4
p
Q.11 (c)
⇒ Pm = 2.34 kW πDN
=u = 2gH
60
Q.9 (a) From this equation √H ∝ DN
Net positive suction head, H
NPSH = Pressure head + static head = Constant
DN
Net Pressure difference,=200 -(-25) So using this relation for the given
= 225 kPa model an prototype.
Δp =2.25 bar
= 22.95 m of water  H  H
  =  
Static head = 1m (Given)  DN p  DN m
Now, NPSH = 22.95 + 1 = 23.95 m
=24 m of water N p Dm Hp
= ×
N m Dp Hm

© Copyright Reserved by Gateflix.in No part of this material should be copied or reproduced without permission
Np
1 =
(sinceU U= U)
=
× 4 1 2

Nm 2 Fixed Moving
Nm =Np=N ∵ V1 = W2 andV2 = W1
Q.12 (a) W22 − W12
R= 2
So, correct pairs are P-2,Q-3,R-4, S-1 W2 − W12 + W22 − W12
∴R=0.5
Q.13 (d)
Velocity of pelton wheel = w×r Q.17 (b)
∴u = 10× 0.5 = 5m/s V
u=
Torque =𝛒𝛒Q(V-u)(1+cos 𝛉𝛉)×R 2
Torque/mass = (V-u)(1+cos 𝛉𝛉) ×R Condititon for maximum power in
= (10-5)(1+cos 60°)×0.5 pelton wheel.
= 3.75(N-m)/kg/s
Q.18 (b)
Q.14 (b)
Given: P1 = 103 kW, H1= 40 m, H2= Q.19 (56.43)
20 m. If a turbine is working under
different heads, the behavior of
turbine can be easily known from
the values of unit quantities i.e. from
the unit power.
P1 P2
3/2
= 3/2
H1 H2
P2 =1000 × (20/40)3/2 Q = 0.05 m3/s
P2 =353.5 kW F= 1000 N
F = ρAV2
Q.15 (d) The above expression can also be
Given: P1 = 1bar, P2 = 30 bar, ρ = 990 written as
kg/m3 4ρQ 2
Isentropic work down by the pump F=
is given by, πd 2
4ρQ 2 4 ×1000 × 0.052
=W vdp =
m
=
dp, v
m =
d2 =
p p πd 2 3.14 ×1000
w 1 1 ∴ d = 3.1847 × 10−3
2

= dp= × ( 30 − 1) ×105 Or d=0.05643


m p 990 Or = 56.43 mm
=2929.29 J/kg = 2.93 kJ/kg
Q.20 (3280.5)
Q.16 (c)
Degree of reaction
Enthalpy drop in moving blade
=
Total enthalpy drop
W22 − W12
R= 2 2
V1 − V2 W22 − W12
2
+
14 22 43 14 22 43 Specific work = u Cw
Given, u=25m/s
Cw= EB+BF

© Copyright Reserved by Gateflix.in No part of this material should be copied or reproduced without permission
Let, ∠CBF = θ = &1V1 − m
F m &2 V2
As axial velocity is same, F = ρA1V1V1 − ρA 2 V2 V2
DE =CF … (i)
DE = 100 Sin 25°… (ii) ∴F=
ρV12 [A1 − A 2 ]
Also, CF= 58.6 Sin θ… (iii) =
QV1 V=
2 2m / sec
From equation (i), (ii) and (iii), we π π 
get =
1000 × 202  (0.06) 2 − (0.04) 2 
4 4 
100
Sin θ= Sin25°
58.6 F = 628.31N
⇒ θ =46.153°
EB= Cw1=100 Cos25° Q.24
=90.63m/s In Pelton Wheel turbine for maximum
BF= Cw2=58.6 Cos θ
efficiency,
= 58.6 × Cos46.153°
=40.59m/s u 1
ηmax= = = 0.50
∴ Cw= Cw1+ Cw2 v1 2
=90.63 + 40.59
=131.22m/s
∴ Specific work = 25×131.22 Q.25 (d)
= 3280.5 J/kg

Q.21 (d)
σ = 0 implies (i), (ii) and (iii)

Q.22 (0.25)
N P
N s = 5/4
H
2
P  N 
∴  1 = 2 
 P2   N1 
=Q N s1 N= s 2 andH1 H 2 
P1 1 50 % reaction stage
= = 0.25
P2 4 θ =
Q.23 (628.31) α =φ
Q.26

It is know that,
Power, P ∝ Q H ∝ D 2 H H ( )
⇒ P ∝ D 2 H 3/2
Thus, we can write for similarity between
model and prototype Francis turbine.
Holding force in horizontal
direction (F) = change in momentum
in horizontal direction

© Copyright Reserved by Gateflix.in No part of this material should be copied or reproduced without permission
2 3/2
Pp D   Hp 
= p  
Pm  D m   Hm 
Dm
=
where, 1= =
:5 H P 30 m, H m 2=
m, Q m 1m3 /s and
DP
Pm = ρgQ m H m
2 3/2
 Dp   Hp 
Thus Pp = ρgQ m H m 
 D   
 m  Hm 
3/2
 30 
= 104 10     ( 5 )   
2

 2 
= 29.047 MW = 29.05 MW

© Copyright Reserved by Gateflix.in No part of this material should be copied or reproduced without permission
ASSIGNMENT QUESTIONS

Q.1 What is the dimension of kinematic Q.6 An odd shaped body weighing 7.5 kg
viscosity of a fluid and occupying 0.01m3 volume will
a) LT-2 b) L2T-1 be completely submerged in a fluid
c) ML T
-1 -1 d) ML-2T-2 having specific gravity of
a) 1 b) 1.2
Q.2 The general form of expression for c) 0.8 d) 0.75
the continuity equation in a
Cartesian co-ordinate system for Q.7 The flow profile of a fluid depends
incompressible or compressible upon
flow is given by a) Velocity of the fluid only
∂u ∂v ∂w b) the diameter of the tube only
a) + + =0
∂x ∂y ∂z c) the Reynolds number
∂ ∂ ∂ d) the surface roughness
b) (ρu) + (ρv) + (ρw) = 0
∂x ∂y ∂z
Q.8 Stream line, path line and streak line
∂ρ ∂ ∂ ∂ are identical when the
c) + (ρu) + (ρv) + (ρw) = 0
∂t ∂x ∂y ∂z a) Flow is steady
∂ρ ∂ ∂ ∂ b) Flow is uniform
d) + (ρu) + (ρv) + (ρw) = 1
∂t ∂x ∂y ∂z c) Flow velocities do not change
steadily with time
Q.3 Point of application of a horizontal d) Flow is neither steady nor uniform
force on a curved surface
submerged in liquid is Q.9 The convective acceleration of fluid
2 in the x-direction is given by
IG IG + Ah ∂u ∂v ∂w
a) −h b) a) u +v +w
Ah Ah ∂x ∂y ∂z
Ah I ∂u ∂v ∂w
c) +h d) G + Ah b) + +
IG Ah ∂t ∂t ∂t
∂u ∂v ∂w
Q.4 The resultant pressure on a body c) u + u + u
immersed in a fluid is called ∂x ∂y ∂z
a) Fluid pressure b) Up thrust ∂u ∂u ∂u
d) u + v + w
c) Buoyancy d) Gravity ∂x ∂y ∂z

Q.5 The center of pressure of a surface Q.10 In a two-dimensional flow of a


subject to fluid pressure is the point viscous fluid Couette flow is defined
a) On the surface at which resultant for
pressure acts a) pressure gradient driven laminar
b) On the surface at which flow between fixed parallel plates
gravitational force acts b) pressure gradient driven laminar
c) At which all hydraulic forces meet flow through non-circular duct
d) Similar to metacenter c) pressure gradient driven laminar
flow through pipe

© Copyright Reserved by Gateflix.in No part of this material should be copied or reproduced without permission
d) Laminar flow between a fixed and 4σ σ cos θ
a moving plate c) d)
wd cos θ 4wd
Q.11 Most efficient channel section is Where,
a) half hexagon in the form of w= specific weight of liquid
trapezoid σ= surface tension
b) triangular θ= angle of contact between liquid
c) rectangular and surface
d) semicircular Q.16 The approximate value of θ (angle of
contact) for Mercury is
Q.12 Which one of the following is a a) 132 b) 182
typical example of non-Newtonian c) 152 d) None of these
fluid of Pseudo plastic variety
a) Milk b) Air Q.17 What should be the property of a
c) Water d) Printing ink good manometer fluid.
a) low vapour pressure, low density
Q.13 Match List-I (Physical properties of b) low vapour pressure, high density
fluid) with List-II c) high vapour pressure, high densit
(Dimensions/definitions) and select d) high vapour pressure, low density
the correct answer using the codes
given below the lists: Q.18 Fluid is a substance which offers no
List-I resistance to change of
A. Absolute viscosity a) Volume b) Pressure
B. Kinematic viscosity c) Shape d) Flow
C. Newtonian fluid
D. Surface tension Q.19 When a piezometer cannot be used
List-II for pressure measurement in pipes
1. Viscosity is constant a) the pressure difference is low
2. Newton per meter b) the velocity is high
3. Poise c) the fluid in the pipe is a gas
4. Stress/strain is constant d) the fluid in the pipe is highly
5. Stroke viscous
Codes: Q.20 The instrument preferred in the
A B C D measurement of highly fluctuating
a) 5 3 1 2 velocities in air flow is
b) 3 5 2 4 a) Pitot-static tube
c) 5 3 4 2 b) Propeller type anemometer
d) 3 5 1 2 c) Three cup anemometer
Q.14 Capillarity is due to d) Hot wire anemometer
a) Adhesion of liquid particle to a Q.21 The property by virtue of which a
surface liquid opposes relative motion
b) Cohesion of liquid particles between its different layers is called
c) Cohesion and Adhesion both a) Surface tension b) Cohesion
d) Surface tension c) Viscosity d) Caillarity
Q.15 For a Glass tube of diameter d, Q.22 The Bulk modulus of elasticity of a
height of capillary is given by fluid is defined as
4wd 4σ dV / V dP
a) b) cos θ a) − b) −
σ cos θ wd dP dV / V

© Copyright Reserved by Gateflix.in No part of this material should be copied or reproduced without permission
dP b) midpoint between centre of
c) d) dP / 8dρ gravity and centre of Buoyancy
dV
c) the point of intersection of the
Q.23 In the phenomenon of cavitation, line of action of Buoyant force
the characteristic fluid property and the centre line of the body
involved is d) the point of intersection of the
a) Surface tension line of action of Buoyant force
b) Viscosity and that of gravitational force
c) Bulk modulus of elasticity
d) Vapour pressure Q.29 The total pressure on a horizontally
immersed surface is
Q.24 The difference of pressure between a) wA b) wx�
the inside and outside the soap c) wAx� d)
wA
bubble is x�

4σ 2σ Where, w = specific weight of the


a) b) liquid
d d A = area of the immersed surface
8σ 6σ x� = depth of the center of gravity of
c) d)
d d the immersed surface from the
liquid surface
Q.25 The difference of pressure between
the inside and outside the liquid jet Q.30 When a body, floating in a liquid is
4σ 2σ given a small angular displacement,
a) b)
d d it starts oscillating about a point
8σ 6σ known as
c) d) a) Center of pressure
d d
b) Center of gravity
c) Center of buoyancy
Q.26 The difference of pressure between
d) Metacenter
the inside and outside the liquid
drop is Q.31 The time period of oscillation of a
4σ 2σ floating body is given by
a) b)
d d K2 hg
8σ 6σ a) 2π b) 2π
c) d) hg K2
d d
1 K2 1 hg
c) d)
Q.27 The buoyant force is 2π hg 2π K 2
a) equal to volume of liquid
displaced Q.32 The time period of oscillation of a
b) force necessary to maintain floating body with increase in
equilibrium of a submerged metacentric height will
body a) Increase
c) the resultant force acting on a b) decrease
floating body c) unaltered
d) the resultant force on a body due d) lower/higher depending on
to the fluid surrounding it weight of body
Q.33 A floating body is in stable
Q.28 The metacenter is equilibrium when
a) centroid of the displaced fluid
volume

© Copyright Reserved by Gateflix.in No part of this material should be copied or reproduced without permission
a) Its center of gravity is below the Where I = Moment of inertia of the
center of buoyancy plan of the floating body at the
b) its metacentric height is zero water surface.
c) its metacentric height is +ve V = Volume of the body submerged
d) Its metacentric height is -ve in water.
BG = Distance between the center of
Q.34 Which one of the following is the gravity (G) and the center of
condition for stable equilibrium of a Buoyancy (B).
floating body
a) The metacenter coincides with Q.38 The continuity equation represents
the center of gravity the conservation of
b) The metacenter is above the a) Mass b) Momentum
center of gravity c) Energy d) Vorticity
c) The metacenter is below the
center of gravity Q.39 The Bernoulli’s equation refers to
d) None of these conservation of
a) Mass b)Momentum
Q.35 The pressure center is c) Force d) Energy
a) The centroid of the pressure
prism Q.40 How are the velocity coefficient Cv,
b) a point of the line of action of the discharge coefficient Cd and the
resultant force contraction coefficient Cc of an
c) at the centroid of the submerged orifice related?
area a) Cv= CvCd b) CC = CvCd
d) Always above the centroid of the c) Cd= CCCv d) CCCvCd = 1
area
Q.41 The coefficient of velocity (Cv) for
Q.36 A submerged body will be in stable an orifice is
equilibrium if
a) The centre of buoyancy B is 4x 2 2x
a) C v = b) C v =
below the centre of gravity G yH 4yH
b) The centre of buoyancy B
x2
coincides with G c) C v = d) None of above
c) The centre of buoyancy B is 4yH
above the metacenter M
d) The centre of buoyancy B is Q.42 The equation of motion for a viscous
above G fluid is known as
a) Euler's equation
Q.37 How is the metacentric height (GM) b) Reynolds equation
expressed? c) Navier strokes equation
 I d) Hagen Poiseuille equation
a) GM = BG −  
V
Q.43 The rate of flow through
V
b) GM =   − BG venturimeter varies as
 I
a) H b) H
 I
c) GM =   − BG c) H 3/2
d) H 5/2
V
V
d) GM = BG −  
 I

© Copyright Reserved by Gateflix.in No part of this material should be copied or reproduced without permission
Q.44 How is the difference of pressure directions in terms of stream
head h measured by mercury oil function ( ) are
differential manometer expressed ∂ψ ∂ψ
 Sg  a) u = and v =
a)=h x 1 −  b)h= x[Sg - So] ∂x ∂y
 S0  ∂ψ ∂ψ
b) u = and v =
 Sg  ∂y ∂x
c) h = x[So - Sg] =d) h x  − 1
 S0  ∂ψ ∂ψ
c) u = - and v =
Where x = manometer reading Sg ∂y ∂x
and So are the specific gravities of ∂ψ ∂ψ
mercury and oil respectively d) u = and v =-
∂x ∂y
Q.50 It is recommended that the diffuser
Q.45 Differential manometer is used to angle of a venturimeter should be
measure kept less than 6° because
a) very low pressure a) Pressure decreases in the flow
b) pressure in pipes, channels etc direction and flow separation
c) atmospheric pressure may occur.
d) difference of pressure between b) Pressure decreases in the flow
two points direction and flow may become
Q.46 The diameter of the nozzle (d) for turbulent.
maximum transmission of power is c) Pressure increases in the flow
given by direction and flow separation
1 1 may occur.
D  5 2 D  5 3
d) Pressure increases in flow
a) d =   b) d =  
 8fl   8fl  direction and flow may become
1 1 turbulent.
 D5  4  D5  5
c) d =   d) d =   Q.51 If is the velocity potential function
 8fl   8fl 
in 2-D flow field, then the velocity
Where D = Diameter of pipe, components u and v are defined as
f =Darcy's friction factor ∂φ ∂φ
l= Length of pipe a) u = and v =
∂x ∂y
Q.47 The critical value of Reynolds ∂φ ∂φ
number for transition from laminar b) u =- and v =
∂y ∂x
to turbulent boundary layers in
external flow is taken ∂φ ∂φ
c) u =- and v =-
a) 2300 b) 4000 ∂x ∂y
c) 5 × 10 5 d)3 × 106 ∂φ ∂φ
d) u = and v = -
Q.48 For pipes, laminar flow occurs when ∂y ∂x
Reynolds number is
a) less than 2000 Q.52 Boundary layer separation is caused
b) between 2000 and 4000 by
c) more than 4000 a) Adverse pressure gradient
d) less than 4000 b) laminar flow changing to
turbulent flow
Q.49 In a two-dimensional flow, the c) reduction in pressure to vapour
velocity components in x and y pressure
d) none of these

© Copyright Reserved by Gateflix.in No part of this material should be copied or reproduced without permission
a) Which is along path of the particle
Q.53 Laminar sub-layer acts as b) Which is always parallel to the
a) An insulating medium main direction of flow
b) good conductor of heat c) Along which there is no flow
c) refractory substance d) On which tangent drawn at any
d) heat absorber point gives the direction of velocity
Q.54 In order to increase .sensitivity of U- Q.59 For a fully developed laminar flow
tube manometer, one leg is usually through a pipe the volumetric flow
inclined by angle 𝜃𝜃, sensitivity of is given by (symbols have the usual
inclined tube to sensitivity of U-tube meaning)
equal to π  −dp  π 4  −dp 
a) R 4   b) R  
1 8µ  dz  4µ  dz 
a) sinθ b)
sinθ π 4  −dp  π 4  −dp 
c) R   d) R  
1 32µ  dz  16µ  dz 
c) cos θ d)
cosθ
Q.60 Maximum efficiency of transmission
Q.55 At the point of boundary layer of power through a pipe is
separation a) 25% b) 66.66%
a) Shear stress is maximum c) 33.3% d) 50%
b) shear stress is zero
Q.61 Power transmitted through a pipe is
c) Velocity is positive
maximum when
d) density variation is maximum
H H
a) H L = b) H L =
Q.56 The energy thickness δe is given by 2 3
δ H
u  u  c) H L = d) HL = H
=
a) δe ∫u
0 ∞ 
1 −  dy
u∞ 
4
Where H = total head supplied
δ
u  u2  HL = head loss due to friction
=
b) δe ∫ 1 − 2  dy
0 ∞ 
u u∞  Q.62 Power transmitted through a pipe is
δ
u  u  given by
=
c) δe ∫u
0 ∞ 
2 
1−  dy
u∞  a) W × Q × H b) W ×Q × HL
c) W × Q × (H- HL) d)W×Q×(H+HL)
δ
 u  Where W = specific weight of the
d) δ=
e ∫0  u ∞  dy
 1 −
fluid flowing through pipe Q =
discharge
Q.57 The displacement thickness δ* is
δ Q.63 Boundary layer flow separates from
u  u  the surface if
=
a) δ ∫ 1 −
*
 dy
0 ∞ 
u u∞  du dp
δ
a) = 0 and =0
u  u2  dy dx
=
b) δ* ∫0 u ∞ 1 − u ∞2  dy du dp
b) = 0 and >0
δ dy dx
u  u 
=
c) δ ∫u 1−
*
2   dy du dp
0 ∞ 
u∞  c) = 0 and >0
dy dx
δ
 u  d) Boundary layer thickness is zero
d) δ= ∫ 1 −
*
 dy
0
u∞ 

Q.58 A stream line is a line

© Copyright Reserved by Gateflix.in No part of this material should be copied or reproduced without permission
Q.64 The hydraulic mean depth for a 1
u  y 7
circular pipe of diameter d when b) = 
running full is u∞  δ 
1
d
a) d b) u  y   y 7
2 = 2  −  
c)
d d
u∞ δ δ
c) d) 3
4 3 u 3 y 1 y
d)=  −  
Q.65 Which one of the dimensionless u∞ 2  δ  2  δ 
numbers identifies the Q.70 Given that
compressibility effect of a fluid 𝛅𝛅 = boundary layer thickness
a) Euler number 𝛅𝛅* = displacement thickness
b) Froude number 𝛅𝛅e = energy thickness
c) Mach number θ = momentum thickness
d) Weber number The shape factor H is given by
δ δ*
Q.66 Capillary tube viscometers used for a) H = e b) H =
measurement of viscosity are based δ θ
on δ δ
c) H = d) H = *
a) Stroke's law θ δ
b) Hagen-Poiseuille
Q.71 According to Blasius, the local skin
c) Darcy Weisbach equation
friction coefficient in the boundary
d) none of these
layer over a flat plate is given by
0.332 0.664
Q.67 The hydrodynamic boundary layer a) b)
thickness is defined as the distance Re x Re x
from the surface where the 0.647 1.328
c) d)
a) Velocity equals to the local Re x Re x
external velocity
b) Velocity equals the approach Q.72 The thickness of laminar boundary
velocity layer at a distance 'x' from the leading
c) Momentum equals 99% of the edge over a flat plate varies as
momentum of the tree stream a) x b) x1/2
d) Velocity equals 99% of the local c) x1/5 d) x4/5
external velocity
Q.73 The von-karman momentum
Q.68 Laminar boundary layer thickness integral equation expressed as
(δ) at any point x for flow over a flat (where θ is momentum thickness)
plate is given by τ0 ∂θ τ0 ∂θ
0.664x 1.328x a) = b) =
a) b) 1 2 ∂x
ρU ∞ 2ρU ∞ ∂x
2

Re x Re x 2
1.75x 5.0x τ ∂θ τ0 ∂θ
c) d) b) 0 2 = d) =
Re x Re x ρU ∞ ∂x 1 2 ∂x
ρU ∞
3
Q.69 The velocity profile for turbulent
Q.74 Geometric similarity between model
layer over a flat plate is
and prototype means the similarity
u x y in
a)= sin  − 
u∞ 2 δ a) Motion b) Discharge
c) Linear dimensions d) Forces

© Copyright Reserved by Gateflix.in No part of this material should be copied or reproduced without permission
Q.75 Kinematic similarity between model N P N Q
c) d)
and prototype is the similarity of  54   54 
a) Shape b) Discharge/motion H  H 
   
c) Stream d) Forces
Q.76 Euler number is defined as the square Q.83 Surge tank is used in pipe line
root of ratio of inertia forces to a) to reduce frictional loss in pipe
a) Viscous force b) Elastic force b) to ensure uniform flow in pipe
c) Pressure force d) Gravity force c) to relieve the pressure due to
Q.77 If 'n' variables in a physical water hammer
phenomenon contain 'm' d) to reduce cavitations
fundamental dimensions, then the
variables can be arranged into Q.84 Which one of the following is not a
a) n dimensionless terms positive displacement pump
b) m dimensionless terms a) Reciprocating pump
c) (n − m) dimensionless terms b) Centrifugal pump
d) (n + m) dimensionless terms c) Vane pump
d) Lobe pump
Q.78 Froude number is the square root of Q.85 Which one of the following turbine
ratio of inertia force to exhibits a nearly constant efficiency
a) Viscous force over a 60% to 140% of design speed
b) Surface tension force a) Pelton wheel
c) Elastic force b) Francis turbine
d) Gravity force c) Deriaz turbine
d) Kaplan turbine
Q.79 For laminar flow in pipes the
momentum correction factor is Q.86 Which phenomenon will occur when
a) >1 b) 1.03 the valve at the discharge end of a
c) 1.33 d) 2.00 pipe connected to a reservoir is
suddenly closed
Q.80 For turbulent flow in pipes the a) Cavitation b) Erosion
momentum correction factor is c) Hammering d) Surging
a) > 1 b) 1.2
c) 1.33 d) 2.00 Q.87 The function of guide vanes in a
reaction turbine is to
Q.81 The logarithmic velocity distribution a) Allow the water to enter the
is observed in runner without shock
a) open channel flow b) Allow the water to flow over
b) laminar boundary layer theory, without forming eddies
c) turbulent boundary layer c) Allow the required quantity of
d) laminar boundary layer over a water to enter the turbine
flat plate d) all of the above

Q.82 The specific speed of a turbine is Q.88 The degree of reaction of turbine is
given by defined as the ratio of
N P N Q a) Static pressure drop to total
a) b) energy transfer
 2
3
 32 
 
H H  b) Total energy transfer to static
    pressure drop

© Copyright Reserved by Gateflix.in No part of this material should be copied or reproduced without permission
c) Change of energy across the Q.95 Hydraulic efficiency of a turbine is
turbine to the total energy transfer defined as
d) Velocity energy to pressure energy a) Power available at the inlet of
turbine to power given by water
Q.89 The maximum number of Jets to runner
generally employed in impulse b) Power at the shaft of the turbine
turbine without Jet interference is to power given by water to the
a) 4 b) 8 runner
c) 6 d) Not defined c) Power at the shaft of the turbine
to the power at the inlet of turbine
Q.90 A draft tube is used in reaction d) Power given by water to the
turbine runner of a turbine to the power
a) To guide water downstream supplied by the water at the inlet
without splashing of the turbine
b) To convert residual pressure
energy into kinetic energy Q.96 The flow of water leaving the impeller
c) To convert residual kinetic in a centrifugal pump casing is
energy into pressure energy a) Free vortex flow
d) To stream line the flow in tailrace b) Forced vortex flow
c) Centrifugal flow
Q.91 A centrifugal pump gives maximum d) Coutte flow
efficiency when its blades are
a) Bent forward b) Bent backward Q.97 Operating characteristic curves of a
c) Straight d) Wave shaped turbine means
Q.92 The number of buckets on the a) curve drawn at constant speed
periphery of a pelton wheel is given b) curve drawn at constant efficiency
by c) curve drawn at constant head
m m d) curve drawn at constant load
a) 5+ b) 10+
2 2
m m Q.98 Air vessel in a reciprocating pump is
c) 15+ d) 20+ used
2 2 a) To obtain a continuous supply of
Q.93 If α is the blade angle at the outlet, water at uniform rate
then the maximum hydraulic b) To reduce suction head
efficiency of an ideal impulse c) To increase the delivery head
turbine is d) all of above
1 + cos α 1 − cos α
a) b) Q. 99 The stream function is given by
2 2
1 − sin α 1 + sin α Ψ=3xy, then the velocity at the point
c) d) (2, 3) is
2 2 a) 9 b) -6
Q.94 Which one of the following is an c) 117 d) 10.8
example of the pure (100%)
reaction machine Q.100 Why is multi-staging in centrifugal
a) Pelton wheel pumps used
b) Francis turbine a) For high flow rate
c) Modern gas turbine b) For high head
d) Lawn sprinkler c) For high speed
d) For high efficiency

© Copyright Reserved by Gateflix.in No part of this material should be copied or reproduced without permission
length and time dimension
Q.101 Match List-I(Properties of fluids) respectively, what is the dimension
with List-II(Definition) and select of the constant B?
the correct answer using the codes a) ML-1T-1 b) ML-1T-2
given below the lists: c) MT-1 d) ML-1
List-I
A. Ideal fluid Q.105 An increase in pressure of 2 bars
B. Newtonian fluid decreases the volume of a liquid by
C. μ/⍴ 0.01 percent. The bulk modules of
D. Mercury in glass elasticity of the liquid is
List-II a) 2×105 N/m2 b)2×107 N/m2
1. Viscosity does not vary with rate c) 2×10 N/m
9 2 d) 2×1011 N/m2
of deformation
2. Fluid of zero viscosity Q.106 The specific weight of ocean water
3. Dynamic viscosity changes according to the equation
4. Capillary depression W=w0+c h where w0 =specific
5. Kinematic viscosity weight of ocean water at the free
6. Capillary rise surface, then pressure as a function
Codes: of depth is
A B C D 5 2
a) 1 2 4 6 a) w0h b) w 0 h + ch 5
2
b) 1 2 3 4 3 3
c) 2 1 3 6 2 2
c) w 0 h + ch d) w 0 h + ch 2
d) 2 1 5 4 3
Q.107 The pressure inside a soap bubble of
Q.102 The capillary rise at 20°C in clean
50 mm diameter is 25 N/m2 above
glass tube of 1 mm diameter
the atmospheric pressure. What is
containing water is (Take σw−air =
the surface tension in soap film
0.0736 N/m)
a) 0.156N/m b) 0.312N/m
a) 15 mm b) 50 mm
c) 0.624 N/m d) 0.948 N/m
c) 20 mm d) 30 mm
Q.103 Consider a soap film bubble of Q.108 The work done in blowing a soap
diameter D. If the external pressure bubble of diameter 12 cm. Assume
is P0 and the surface tension of the the surface tension of soap solution
soap film is , the expression for the =0.040 N/n
a) 36.2×10-4 N-m b) 51.8×10-4 N-m
pressure inside the bubble is
c) 36.2×10-3N-m d) 58.8×10-4 N-m

a) P0 b) P0 +
D Q.109 Match List-I (fluid parameters) with
4σ 8σ List-II (basic dimensions) and select
c) P0 + d) P0 +
D D the correct answer using the codes
given below the lists:
Q.104 The relation between the stress
List-I
and the strain rate (dux/dy) for the A. Dynamic viscosity
rapid flow of a granular material is B. Chezy's roughness coefficient
2
 du  C. Bulk modulus of elasticity
given by τ =B  x  where B is a D. Surface tension ( )
 dy 
constant. If M, L and T are the mass, List-II

© Copyright Reserved by Gateflix.in No part of this material should be copied or reproduced without permission
1. M/T2 V2 V2
2. M/LT2 a) Lρ b) Dρ
D L
3. M/LT μVL
L c) 2 d) μv/L
4.� D
T
Codes: Q.113 Consider the following statements
A B C D 1. Gases are considered
a) 3 1 4 2 incompressible when Mach
b) 1 4 2 3 number is less than 0.2.
c) 3 4 2 1 2. A Newtonian fluid is
d) 1 2 4 3 incompressible and non-viscous.
Q.110 Match List-I (phenomenon) with 3. An ideal fluid has negligible
List-II (causes) and select the surface tension.
correct answer using the codes Which of these statement(s) is/are
given below the lists: correct?
List-I a) 2 and 3 b) 2 alone
A. Shock wave c) 1 alone d) 1 and 3
B. Flow separation.
C. Capillary rise Q. 114 The standard atmospheric pressure
D. Cavitation is 762 mm of Hg. At a specific
List-II location, the barometer reads 700
1 . Surface tension mm of Hg. At this place, what does
2. Vapour pressure the absolute pressure of 380 mm of
3. Compressibility Hg corresponds to?
4. Adverse pressure gradient a) 320 mm of Hg vacuum
Codes: b) 382 mm of Hg vacuum
A B C D c) 62 mm of Hg vacuum
a) 3 1 2 4 d) 62 mm of Hg gauge
b) 4 2 1 3 Q.115 The depth of centre of pressure for a
c) 3 4 1 2 rectangular lamina immersed
d) 4 1 2 3 vertically in water up to height h is
given by
Q.111 In an experiment, the following h h
shear stress-time rate of shear a) b)
2 4
strain values are obtained for a fluid 2h 3h
Time rate of shear (1/s) 0 2 3 4 c) d)
Shear stress (kPa) 0 1.4 2.6 4 3 2
How can the fluid be classified Q.116 What is the vertical component of
a) Newtonian fluid pressure force on submerged
b) Bingham plastic curved surface equal to
c) Pseudo plastic a) its horizontal component
d) Dilatant b) the product of the pressure at
Q.112 A Newtonian liquid (ρ = density, centroid and surface area
μ=viscosity) is flowing with velocity c) the force on a vertical portion of
V in a tube of diameter D. Let P be the curved surface
d) the gravity force of liquid
the pressure drop across the length
vertically above the curved
L. For a laminar flow, P is
surface up to the free surface
proportional to

© Copyright Reserved by Gateflix.in No part of this material should be copied or reproduced without permission
Q.117 The flow in a pipe whose valve is
being opened or closed gradually is
an example of
a) steady flow
b) nonsteady flow
c) steady uniform flow Consider flow of oil and water
d) steady nonuniform flow through a channel; the boundary
conditions at the interface are
Q.118 While measuring the velocity of air ρ
a) Velocity & shear stress are
=1.2 kg/m3, the difference in the continuous
stagnation and static pressure of a
b) Shear stress is continuous and
Pitot static tube was found to be velocity is discontinuous
380Pa. The velocity at that location c) Shear stress is zero and velocity
in m/sec is
is continuous
a) 24.03 b) 4.02 d) Shear stress is zero
c) 17.8 d) 25.17
Q.122 A float valve of the ‘ball-cock’ type
Q.119 A pipe flow system with flow required to close an opening of a
direction is shown in above figure. supply pipe feeding a cistern is as
The following table gives the shown in the given figure
velocities & the corresponding areas

The buoyant force FB required to be


exerted by the float to keep the
value closed against a pressure of
0.28 N/mm2
a) 4.4 N b) 5.6 N
The value of V2 is c) 7.5 N d) 9.2 N
a) 2.5 cm/sec b) 5 cm/sec
c) 7.5 cm/sec d) 10 cm/sec 3
Q.123 Given ϕ= 3xy and Ψ= (y2 –x2), the
Q.120 The correct sequence of the 2
centrifugal pump components discharge passing between the
though which the fluid flows is stream lines through the point (1, 3)
a) Impeller, suction pipe, foot valve and (3, 3) is
and strainer delivery pipe a) 2 unit b) 4 unit
b) Foot valve and strainer, suction c) 8 unit d) 12 unit
pipe, impeller & delivery pipe Q.124 A centrifugal pump having an
c) Impeller, suction pipe, delivery impeller of 10cm diameter discharges
pipe, foot valve and strainer 40 litre/second when running at
d) suction pipe delivery pipe, 1000 rpm. The corresponding speed
Impeller , foot valve and strainer of a geometrically similar pump
having an impeller of 40 cm diameter
Q.121 and 0.8 m3/s discharges will be
a) 276.4 rpm b) 298.3 rpm
c) 312.5 rpm d) 358.2 rpm

© Copyright Reserved by Gateflix.in No part of this material should be copied or reproduced without permission
Q.125 A U-tube manometer is used to Q.128 The head loss in a sudden expansion
measure the pressure in an oil pipe from 8 cm diameter pipe to 16cm
A as shown in the figure. The diameter pipe in terms of velocity V1
specific gravity of the oil is 0.8 and in the smaller pipe is
that of mercury is 13.6. The 1  V2  3  V12 
equivalent gauge pressure nearly. a)  1  b)  
4  2g  16  2g 
1  V12  9  V12 
c)   d)  
64  2g  16  2g 

Q.129 If a fluid jet discharging from a 50


mm diameter orifice has a 40 mm
diameter at its vena contracts, then
its coefficient of contraction will be
a) 0.32 b) 0.64
a) 8.53 kN/M2 b) 11.77 kN/M2 c) 0.96 d) 1.64
c) 13.34 kN/M2 d) 15.00 kN/M2
Q.126 A multi-tube manometer filled with Q.130 A U-tube manometer shown
water up to level A, B and C as attached to an air pipe reads a
shown in the figure is rotated about height of 20 cm of mercury as
the vertical axis at A. The water shown. What is the pressure in kPa
levels at A, B and C will all lie on in the air pipe?

a) 26.7 b) 32.4
a) a circle b) an ellipse c) 38.6 d) 42.5
c) a hyperbola d) a parabola Q.131 Pressure drop in a 100 mm
diameter horizontal pipe is 50 kPa
Q.127 What is the absolute pressure at A over a length of 10m. The shear
measured by an open tube stress at the pipe wall is
manometer as in the figure above? a) 0.25 kPa b) 0.125 kPa
Assume atmospheric pressure as c) 0.50 kPa d) 25.0 kPa
103 kN/m2 (Where SA and SB are the
specific gravities of the two fluids) Q.132 In flow through a pipe, the
transition from laminar to turbulent
flow does not depend on
a) density of fluid
b) length of pipe
c) diameter of pipe
d) velocity of the fluid
Q.133 Two pipe lines of equal length are
connected in series. The diameter of
a) 78.5 kN/M2 b) 180 kN/M2
second pipe is two times that of first
c) 1030 kN/M2 d) 103 kN/M2
pipe. The ratio of head loss between
first and second pipe

© Copyright Reserved by Gateflix.in No part of this material should be copied or reproduced without permission
a) 1 : 32 b) 32: 1
c) 1: 8 d) 1 : 4 Q.139 In a laminar boundary layer, the
velocity distribution can be assumed
Q.134 Darcy-Weisbach equation for the to be given in usual notation as
head loss in a flow through a pipe is u y
= . Which one of the following is
4fLV 2 v δ
given by h f = (The symbols
2gd the correct expression for the
have the usual meaning) for the displacement thickness (δ*) for this
laminar flow through a circular pipe, boundary layer
how does the friction factor f vary δ
a) δ*= δ b) δ*=
with a Reynolds number (Re)? 2
8 16 δ δ
a) f= b) f= c) δ* = d) δ* =
Re Re 4 6
32 64
c) f= d) f= Q.140 Using the Prandtl's mixing length
Re Re concept, how is the turbulent shear
Q.135 In a fully turbulent flow through a stress expressed?
rough pipe, the friction factor 'f' is du du
(Re is Reynolds number and ξSD is a) ρl b) ρl2
dy dy
relative roughness) 2
a) a function of Re  du 
2
 du 
c) ρlv   d) ρl2  
b) a function of Re and ξS D  
dy  dy 
c) a function of ξS D
d) independent of Re and ξS D Q.141 If the stream function is given by ψ
= 3xy then the velocity at a point (2 ,
Q.136 The Chezy's constant c is related to
3) will be
Darcy-weishbach friction factor 'f' as
a) 7.21 unit b) 18 unit
g    c) 10.82 unit d) 54 unit
a) C =   b) C =  g 
 8f  1
 4
f  Q.142 Both free vortex and forced vortex
 f  can be expressed mathematically in
 8g 
c) C =   d) C =   terms of tangential velocity v at the
 f   8g  corresponding radius r. Choose the
correct combination
Q.137 The stream function for a two-
Free vortex Forced vortex
dimensional flow is given by ψ =
a) v = r (constant) vr = (constant)
2xy. The velocity at (2,2) is b) v2r = (constant) v = r (constant)
a) 4 2 b) 4 c) vr = (constant) v2=r (constant)
c) 2 2 d) 2 d) vr = (constant) v = r (constant)
Q.138 The velocity distribution in a Q.143 Air flows past a golf ball of 20 mm
turbulent boundary layer is given by radius. It is observed that the
1
boundary layer becomes turbulent
u  y 7
=   What is the displacement at Reynolds number of 2 × 105. If the
v δ kinematic viscosity of air is 1.5 ×
thickness δ* 10-5 m2/sec then the speed of air
δ when the flow becomes turbulent is
a) δ b)
7 a) 150 m/sec b) 90 m/sec
7 δ c) 75 m/sec d) 62.5 m/sec
c) δ d)
8 8

© Copyright Reserved by Gateflix.in No part of this material should be copied or reproduced without permission
Q.144 An open circular cylinder of 1.2m
height is filled with a liquid to its
top. The liquid is given a rigid body
rotation about the axis of the
cylinder & the pressure at the centre
line at the bottom surface is found to The characteristics of a pump are as
be 0.6 m of liquid. What is the ratio shown in the given figure. Based on
of volume of liquid spilled out of the this figure, match List-I with List-II
cylinder to the original volume choose the correct answer using the
a) 1/4 b) 3/8 codes given below the list:
c) 1/2 d) ¾ List-I
A. Curve P
Q.145 By which one of the following, a B. Curve Q
small quantity of water may be lifted C. Curve R
to a great height List-II
a) Hydraulic ram b) Hydraulic crane 1. Discharge versus head
c) Hydraulic lift d) Hydraulic coupling 2. Head versus discharge
3. Power versus discharge.
Q.146 As water flows through the runner 4. Efficiency versus discharge
of a reaction turbine, pressure Codes:
acting on it would vary from A B C
a) more than atmospheric pressure a) 2 4 3
to vacuum b) 1 3 2
b) less than atmospheric pressure c) 1 4 3
to zero gauge pressure d) 4 3 1
c) atmospheric pressure to more
than atmospheric pressure Q.150 A pump running at 1000 rpm
d) atmospheric pressure to vacuum consumes 1 kW and generates head
of 10m of water. When it is operated
Q.147 Which of the following statements is at 2000 rpm. Its power consumption
NOT correct in the context of and head generated would be
laminar flow through with pipeline? a) 4 kW, 50 m of water
a) Shear stress is zero at the centre b) 6 kW, 20m of water
& varies linearly with pipe radius c) 3 kW, 30 m of water
b) Head loss is proportional to d) 8 kW, 40 m of water
square of the average flow velocity
c) The friction factor varies inversely Q.151 Match List-I (Type of pump) with
with flow Reynolds number List-II (Liquid handled) and select
d) No dispersion of die injected into the correct answer using the codes
the flow stream given below the lists:
List-I
Q.148 A penstock pipe of 10 m diameter A. Closed impeller pump
carries water under a pressure head B. Semi-open impeller pump
of 100 m. If the wall thickness is 9 C. Open impeller
mm, what is the tensile stress in the List-II
pipe wall in MPa? 1. Sandy water
a) 27.25 b) 544.4 2. Acids
c) 272.5 d) 1090 3. Sewage water
Q.149 Codes:
A B C

© Copyright Reserved by Gateflix.in No part of this material should be copied or reproduced without permission
a) 1 3 2
b) 3 1 2 Q.155 Priming is necessary in
c) 2 1 3 a) Centrifugal pumps to lift water
d) 1 2 3 from a greater depth.
b) Centrifugal pumps to remove air
Q.152 The transition Reynolds number for in the suction pipe and casing
flow over a flat plate is 5 × 105. What c) Hydraulic turbine to remove air
is the distance from the leading edge in the turbine casing
at which transition will occur for d) Hydraulic turbine to increase the
flow of water with a uniform velocity
speed of turbine and to generate
of 1m/sec (for water the kinematic
more power.
viscosity, υ=0.86×10-6 m2/sec)
a) 0.43 m b) 1 m Q.156 A pressure drop for a relatively low
c) 43 m d) 103 m Reynolds number flow in a 600 mm,
30 m long pipe line is 70 kPa. What
Q.153 A hydraulic press has a ram of 15
is the wall shear stress
cm diameter and plunger of 1.5 cm.
a) 0 Pa b) 1400 Pa
It is required to lift a mass of 100 kg.
c) 700 Pa d) 350 Pa
The force required on plunger is
nearly equal to
a) 100 N b) 1000 N
c) 10,000 N d) 10 N
Q.154 Match List-I (Hydraulic turbine)
with List-II (Application area) and
select the correct answer using the
codes given below the lists.
List-I
A. Pelton turbine
B. Francis turbine
C. Kaplan turbine
List-II
1. Low head, high discharge
2. Medium head, medium discharge
3. High head , low discharge
Codes:
A B C
a) 2 3 1
b) 2 1 3
c) 3 1 2
d) 3 2 1

© Copyright Reserved by Gateflix.in No part of this material should be copied or reproduced without permission
ANSWER KEY:
1 2 3 4 5 6 7 8 9 10 11 12 13 14
(b) (c) (b) (c) (a) (d) (c) (a) (d) (d) (a) (a) (d) (c)
15 16 17 18 19 20 21 22 23 24 25 26 27 28
(b) (a) (b) (c) (c) (d) (c) (b) (d) (c) (b) (a) (d) (c)
29 30 31 32 33 34 35 36 37 38 39 40 41 42
(b) (d) (a) (b) (c) (b) (b) (d) (c) (a) (d) (c) (c) (c)
43 44 45 46 47 48 49 50 51 52 53 54 55 56
(b) (d) (d) (c) (c) (a) (c) (c) (c) (a) (a) (b) (b) (b)
57 58 59 60 61 62 63 64 65 66 67 68 69 70
(d) (d) (a) (b) (b) (c) (b) (c) (c) (b) (d) (d) (b) (b)
71 72 73 74 75 76 77 78 79 80 81 82 83 84
(b) (b) (c) (c) (b) (c) (c) (d) (c) (b) (c) (c) (c) (b)
85 86 87 88 89 90 91 92 93 94 95 96 97 98
(d) (c) (d) (a) (c) (c) (b) (c) (a) (b) (d) (b) (a) (a)
99 100 101 102 103 104 105 106 107 108 109 110 111 112
(d) (b) (c) (d) (d) (d) (c) (c) (a) (a) (c) (c) (d) (c)
113 114 115 116 117 118 119 120 121 122 123 124 125 126
(d) (a) (c) (d) (b) (d) (b) (b) (b) (a) (d) (c) (b) (d)
127 128 129 130 131 132 133 134 135 136 137 138 139 140
(b) (d) (b) (a) (b) (b) (b) (d) (b) (c) (a) (d) (b) (d)
141 142 143 144 145 146 147 148 149 150 151 152 153 154
(c) (d) (c) (a) (a) (a) (b) (b) (a) (d) (c) (a) (d) (d)
155 156
(b) (d)

© Copyright Reserved by Gateflix.in No part of this material should be copied or reproduced without permission
EXPLANATIONS

Q.1 (b) ∂u ∂u ∂u
Where u +v +w is the
Unit of kinematic viscosity is m2/s. ∂x ∂y ∂z
∴Dimension of kinematic viscosity is ∂u
L2T-1 convective acceleration and is
∂t
the temporal acceleration.
Q.2 (c)
The general form of expression for
the continuity equation in a Q.10 (d)
Coutte flow is characterized as flow
Cartesian co-ordinate system
of very low value of Reynolds
∂ρ ∂ ∂ ∂
+ (ρu) + (ρv) + (ρw) = 0 number between two parallel plate,
∂t ∂x ∂y ∂z one is fixed and other is movable.

Q.3 (b) Q.11 (a)


I
C.P = G + h
Ah Q.12 (a)
Where h is distance of centre of
gravity from free surface and Ic area Q.13 (d)
moment of inertia.
Q.14 (c)
Q.4 (c)
Q.15 (b)
Q.5 (a) Weight of liquid raised or lowered
Centre of pressure is defined as the in the capillary to be = (area of tube
point of application of the total × rise or fall) × specific weight
pressure on the surface. π 2 
= d hw
4 
Q.6 (d)

Q.7 (c)
Reynolds number is the factor by
which type of flow is decided i.e.
laminar or turbulent.

Q.8 (a)
Vertical components of surface
Q.9 (d) tension force
∂u ∂u ∂u ∂u = σ cos θ ×circumference
ax = u +v +w +
∂x ∂y ∂z ∂t = σ cos θ × πd
= π d σ cos θ
In equilibrium, the downward
weight of the liquid column h is
balanced by the vertical component

© Copyright Reserved by Gateflix.in No part of this material should be copied or reproduced without permission
of the force of surface tension. Q.21 (c)
Hence From the definition of viscosity,
π 2 viscosity is the resistance offered by
d hw =πdσ cos θ
4 one layer of fluid to another layer &
Height of capillarity opposes relative motion between
4σ them.
=h cos θ
wd
Q.22 (b)
dp
Q.16 (a) Bulk modules of elasticity, K= −
dv
v
It represents the change in volume
with the change in pressure.
Q.17 (b)
Manometric fluid should have low Q.23 (d)
vapor pressure and high density to In any flow system, if the pressure at
have less height of piezometric tube. any point in the liquid approaches
Since, Mercury has both the the vapor pressure, vaporization of
properties, this is the reason, liquid starts, resulting in the pockets
Mercury is used as a manometric of dissolved gasses and vapors. The
fluid. bubbles of vapor thus formed are
carried by the flowing liquid into a
Q.18 (c) region of high pressure where they
A fluid does not offer any resistance collapse, giving rise to high impact
to change of shape and gets pressure. This phenomenon is known
deformed under the action of very as cavitation.
small shear force.
Q.24 (c)
Q.19 (c) A spherical soap bubbles has
Limitations of Piezometer:- surfaces in contact with air, one
• Piezometer cannot be used to inside and other outside, each one of
measure pressure which is which contributes the same amount
considerably excess of atmospheric of tensile force due to surface
pressure. Use of very long glass tension. As such, on a hemispherical
tube would be unsafe. It being both section of a soap bubble of radius r,
fragile and unmanageable. the tensile force due to surface
• Gas pressure cannot be measured tension is equal to 2σ(2πr).
as gas does not form any free However the pressure force acting
surface with atmosphere. on the hemispherical section of the
• Measurement of negative pressure soap bubble is same as in the case of
is not possible due to flow of a droplet and it is equal to ΔP(πr2).
atmospheric air into the container Thus equating these two forces for
through the tube. equilibrium, we have
ΔP(πr2) = 2σ (2πr)
Q.20 (d) 4σ 8σ
∆P= =
Hot wire anemometer is used in the r d
measurement of highly fluctuating
velocities in air flow. Q.25 (b)

© Copyright Reserved by Gateflix.in No part of this material should be copied or reproduced without permission
Consider a jet of liquid of radius r,
length l and having internal Q.31 (a)
pressure intensity p in excess of the Time of oscillation of a floating body
outside pressure intensity. k2
is T = 2π
P(2rl)=σ(2l) gh
σ 2σ
or P= = Where, k = radius of gyration
r d h = metracentric height
Q.26 (a) Q.32 (b)
Consider a spherical droplet of Time of oscillation
radius r having internal pressure
k2
intensity P in excess of the outside T = 2π
pressure intensity. If the droplet is gh
cut into two halves, then the forces 1
acting on one half will be those due T∝
h
to pressure intensity P in the
as h increases, time of oscillation
projected area (πr2) and the tensile
decreases.
force due to surface tension σ acting
around the circumference (2πr).
Q.33 (c)
These two forces will be equal and
For stable equilibrium
opposite for equilibrium and hence
we have
P(πr2) = σ(2πr)
2σ 4σ
Or= P =
r d
Q.27 (d)
A body submerged in a fluid Q.34 (b)
experiences an upward thrust due For stable equilibrium
to fluid pressure. This force is called
buoyant force and a body immersed
in a fluid is buoyed or lifted up by a
force equal to the weight of the fluid
displaced by the body. The body
apparently loses as much of its For stable equilibrium metacenter is
weight as the weight of the fluid above the center of gravity.
displaced by it.
Q.35 (b)
Q.28 (c)
Metacentre is the point of Q.36 (d)
intersection of normal axis of the
body with the new line of action of Q.37 (c)
buoyant force when body is tilted.

Q.29 (b)
F = wAx
P = wx
Where w is the weight density.
Metacentric height GM = BM – BG
Q.30 (d)

© Copyright Reserved by Gateflix.in No part of this material should be copied or reproduced without permission
I
BM =  Sg 
=h x  − 1
V
 S0 
Where I = Area moment of inertia of
Where, Sg=Specific gravity of
top view about longitudinal axis
Mercury
V = Volume of the fluid displaced
S0 = Specific gravity of Oil
Hence,
I
GM =  -BG Q.45 (d)
V
Q.38 (a) Q.46 (c)
The continuity equation represents For maximum power transmission
the conservation of mass. through nozzle:
1/4
Q.39 (d)  D5 
Nozzle diameter, d =  
Bernoulli’s equation refers to  8fl 
conservation of energy. Q.47 (c)

Q.40 (c) Q.48 (a)


The relation between Cd , Cc and Cv is For internal flow or pipe flow
Cd = Cc Cv (Re)cr = 2000
i.e. Re<2000 – Flow is laminar
Q.41 (c) Re>2000 – Flow is turbulent
V gx 2 1
=
C = ×
v
Vth 2y 2gh Q.49 (c)

Q.50 (c)
If diffuser angle is more, eddies
formation may take place and also
flow separation may occur.

Q.51 (c)
2
x x
Cv ⇒ Q.52 (a)
4yH 4yH
Boundary layer separation is caused
Q.42 (c) due to adverse pressure gradient or
In Naiver-stroke equation pressure positive pressure gradient.
force, gravity force and viscous force Q.53 (a)
are considered. In turbulent flow analysis along the
boundary layer, a thin layer of fluid
Q.43 (b) in the immediate neighborhood of
Rate of flow through a venturimeter the boundary develops where viscous
A1A 2 shear stress is predominant while
is given as: Q = 2gh
A12 −A 22 the shear stress due to turbulence is
negligible. This portion is known as
Hence Q ∝ H
laminar sub layer.
In turbulent flow hear transfer is
Q.44 (d) more as compared to laminar flow
and also in this laminar sub layer
heat transfer is very less while, it is

© Copyright Reserved by Gateflix.in No part of this material should be copied or reproduced without permission
present in turbulent flow hence it
acts as insulating medium in Q.63 (b)
turbulent flow. Boundary layer separation occurs at
adverse pressure gradient i.e.
Q.54 (b) dp du
> 0 and =0
Q.50 (b) dx dy
∂u
At the verge of separation =0
∂y y =0 Q.64 (c)
 ∂u  Q.65 (c)
Shear stress τ =  . Hence τ = 0
 ∂y  y =0 At higher Mach number the
compressibility effect is predominant.
Q.56 (b)
Q.66 (b)
Q.57 (d)
Q.58 (a) Q.67 (c)
Stream line is an imaginary line Hydrodynamic boundary layer
drawn in space such that tangent thickness is defined as the distance
drawn to it at any point gives measured in the y direction from
velocity at that point. boundary to the point where the
velocity is 99% of the free stream
Q.59 (a) velocity.
 Vmax 
Q= A × Vavg  Vavg =  Q.68 (d)
 2 
5x
1  1  dp   From the blasius solution δ =
Q= πR 2   −  R 2  Re x
2  4µ  dz  
π 4  −dp  Q.69 (b)
Q= R  
8µ  dz  Turbulent velocity profile follows
1
U y 7
Q.60 (b) 1/7 power law. Hence = 
U∞  δ 
Efficiency of power transmission is
given by will be velocity distribution.
H − HL
η= Q.70 (b)
H
Shape factor is given by
H
For maximum efficiency, H L = Displacement thickness
3 H=
Momentum thickness
H−H/3
We get, ηmax =
H
Q.71 (b)
ηmax = 66.66%
Q.72 (b)
Q.61 (b) 5x 1−
1

For maximum power transmission =δ ⇒δ∝x 2


Re x
H
HL = 1
3 δ∝x 2

Q.62 (c)
Q.73 (c)

© Copyright Reserved by Gateflix.in No part of this material should be copied or reproduced without permission
Von-karman momentum
integral during speed regulation of a
τ0 ∂θ turbine.
equation is expressed as =
ρU ∞ ∂x
2 Due to rapid velocity fluctuations,
large magnitude pressure transients
Q.74 (c) are set-up and these excessive
Geometric similarity ⇒ Similarly in pressures may lead to bursting of
pipe. This phenomenon is known as
linear dimensions.
water hammer.
Some arrangement needs to be
Q.75 (b)
made to convert the rapid velocity
If the ratio of velocity and
fluctuations into slow velocity
acceleration at the corresponding
fluctuations and for this surge tank
points in the model and prototype
is provided just upstream of the
are same then there exists kinematic
power unit in a hydroelectric plant.
similarity.
Q.84 (b)
Q.76 (c)
Ineria force
Euler Number =
Pr essure force

Q.77 (c)
If ‘n’ is the number of variables and
‘m’ is the fundamental dimensions,
then variable can be arranged into
(n - m) dimensionless group known
as π terms.
Note that centrifugal pump is not
Q.78 (d) positive displacement pump. It is a
roto dynamic or dynamic pressure
Ineria force pump.
Froude number =
Gravity force
Q.85 (d)
Q.79 (c) A special type of propeller turbine is
called Kaplan turbine in which the
Q.80 (b) individual runner blades are pivoted
on the hub so that their inclination
Q.81 (c) may be adjusted automatically
Logarithmic profile exist is in rotating about pivots with the help
turbulent flow. of a governor servo-mechanism. In
Kaplan turbine, because of the
Q.82 (c) arrangement for automatic variation
of inlet blade angle with variation in
Q.83 (c) load, the turbine can be run at max
Rapid velocity fluctuations occur in efficiency at all loads.
pipe line during
Q.86 (c)
1. Sudden opening and closing of
When valve at the discharge end is
valves closed suddenly, this causes a sudden
2. Start and shut down of a turbine rise in pressure. This phenomenon
or closure of wicket GATE’s of sudden rise in pressure is called

© Copyright Reserved by Gateflix.in No part of this material should be copied or reproduced without permission
water hammer. To reduce the effect Hydraulic Efficiency: It is defined as
of water hammer surge tank is the ratio of power given by the
provided. water to runner of a turbine (runner
is a rotating part of a turbine and on
Q.87 (d) the runner vanes are fixed) to the
Guide vanes are the vanes which are power supplied by the water at the
attached to the casing and guides inlet of the turbine.
the water to enter into the runner.
Q.96 (b)
Q.88 (a)
Degree of reaction is defined as the Q.97 (a)
ratio of pressure head drop within
the runner to the total energy drop Q.98 (a)
in the runner. Functional or air vessel in a
reciprocating pump
Q.89 (c)
Q.90 (c)
It is a diverging tube which is
attached at outlet of the runner to
carry water from the exit of runner
to tail race. The diverging tube helps
to increase the pressure on the
account of conversion of kinetic
head into pressure head.

Q.91 (b)
Backward blades are used for 1. To obtain a continuous supply of
maximum efficiency. It gives better liquid at a uniform rate.
performance over wide range of 2. To save a considerable amount
application where forward blades of work in overcoming the
are used for higher pressure ratio. frictional resistance in suction
and delivery pipes.
Q.92 (c) 3. To run the pump at a high speed
Number of buckets on the periphery without separation.
of a pelton wheel is given by an
empirical relation ship Q.99 (d)
m D Ψ = 3xy
Z = 15 + , where m =
2 d −∂Ψ
U= = −3x
∂y
Q.93 (a)
=−3 ×2=−6 m/s
∂Ψ
Q.94 (b) ∴v = = 3y = 9m / s
Lawn sprinkler is an example of ∂x
70% reaction machine. =v u 2 + v2
While pelton when is an example of = 62 + 92 = 36 + 81
100% impulse machine.
== =117 10.817m / s
Q.95 (d)
Q.100 (b)

© Copyright Reserved by Gateflix.in No part of this material should be copied or reproduced without permission
Multi-staging (series combination)  12 
2

of pump is used to develop high head. = 0.040 × 4π  ×10−2  × 2


 2 
Q.101 (c) = 36.2 × 10 N-m
-4

Q.102 (d) Q.109 (c)


4σ cos θ N −s M
Heights of Capillary (h) = Dynamic viscosity → =
wd m2 LT
4 × 0.0736 Chezy’s roughness coefficient
= = 0.03 m = 30 mm L
9810 ×10−3 → g=
T
Q.103 (d) Bulk modules of elasticity
8σ N M
P = P0 + → 2 →
D m LT 2
Q.104 (d) N M
2
Surface tension → → 2
 du  m T
τ = B  Q.110 (c)
 dy 
2
 du  ML−1
⇒B=
F
= B  Q.111 (d)
A  dy 
Q.112 (c)
Q.105 (c)
−dp Q.113 (d)
Bulk modulus of elasticity K =
dv
v
Q.114 (a)
−(2 ×105 ) Reference is always taken from
K= = 2 X 109 N / m 2
−0.01 atmospheric pressure which is
100 indicated by Barometer.
Barometer reads the local
Q.106 (c) atmospheric pressure.
dp dp So pressure = 700 – 380 = 320 mm
=w ⇒ =w 0 + c h (given)
dh dh of Hg vacuum.
∫ dp ∫ (w 0 + c h )dh
= Q.115 (c)
3
2
=
P w 0 h + ch 2
3

Q.107 (a)
8σ 8× σ
∆p=
⇒ 25=
d 0.05 IG
C.P = C.P= x +
25 × 0.05
=σ = 0.156N / m Ax
8 h b × h3
C.P =+
2 12 × b × h × h
Q.108 (a)
2
W.D= surface tension × total surface
h h 2h
area + =
2 6 3

© Copyright Reserved by Gateflix.in No part of this material should be copied or reproduced without permission
Q.116 (d)
Vertical component of pressure Q.123 (d)
force on the submerged curved Note the difference between two
surface is equal to the gravity force stream line is discharge per unit
of liquid vertically above the curved width
surface upto the free surface. dQ = ψ 2 − ψ 1
Q.117 (b) 3
ψ (at1,3)= (9 − 1)= 12 unit
2
Q.118 (d) Ψ (at 3,3) = 0
P = ρgh ⇒ dQ = 12 unit
P
h= Q.124 (c)
ρg
380 = D1 10 = cm, Q1 40 lit / s= 40 ×10−3 m3 / s
h= N1 = 1000 rpm
1.2 × 9.812
V = 2gh D2 = 40 cm,Q2 = 0.8 m3/s
From the model relationship
2 × 9.812 × 380 Q1 Q2
V= =
1.2 × 9.812 N1D1 N 2 D32
3

V = 633.33 N 2 = 312.5rpm
V = 25.17 m/sec
Q.125 (b)
Q.119 (b) Pressure at A= ρm gh1 - ρ0gh2
Qin = Qout
= [13.6 × 9.81× 0.1 − 0.8 × 9.81× 0.2] ×103
Q1 + Q2 = Q3 + Q4
(A1V1)+(A2V2) = (A3V3)+(A4V4) =
11.77 ×103 N / m 2 =
11.77 kN / m 2
500 + 50 V2 = 400 + 350
Q.126 (d)
50 V2= 250
The free surface joining the tube will
V2= 5 cm/sec
make a parabola and the water level
in tubes will be highest in C and
Q.120 (b)
lowest in A. It means that water
level in A will decrease from original
level and increase in B and C from
their original level.
Q.127 (b)
PA – ( p A gh A ) – ( p m gh m ) – Pa = 0
PA − (0.8 ×103 × 9.812 ×1 .25) −
× 3 × 9.812 × 0.5) − 103 ×103 =
(13.6 10 0
PA= [9.812 + 66.7216 + 103] ×10            
3

Q.121 (b) PA = 179.53 KPa

Q.122 (a) Q.128 (d)


Taking moment about hinge. We get
π
Fg × 500= 0.28 × ×102 ×100
4
Fg = 4.4N

© Copyright Reserved by Gateflix.in No part of this material should be copied or reproduced without permission
h L1 32D5 32
= =
h L2 D5 1

Q.134 (d)
64
 f =
Re
∴ A1V1 = A 2 V2
Q.135 (b)
V
∴ V2 = 1
4 Q.136 (c)
2
 V
 V1 − 1 
( V1 − V2 =
)  Q.137 (a)
2 2 2
4  V1 3
h=
h = × dψ
2g 2g 2g 42 u= − ⇒
dy
9  V12 
hL =   u = -2x
16  2g 
u = -4 at (2,2)
∂ψ
Q.129 (b) v= ⇒ v= 2y
∂x
Area of orifice v = 4 at (2,2)
CC =
Area of jet
Resultant = 16 + 16 = 4 2
π
( 40 )
2

=CC 4= 0.64 Q.138 (d)


π
( 50 ) Displacement thickness
2

4 δ δ
1
 u  y 7
Q.130 (a)
δ*= ∫ 1 − dy=
0
v ∫0 1 −  δ  dy
δ
Q.131 (b) 1
+1
7
 dp  R 1( y )
τ=-   = y−
1 
1
 ∂x  2 (δ ) 7  + 1
∵ dp = −ve 7  0
50 ×1000 × 50 ×10−3 8
∴τ = 7 (δ ) 7 7 8 1

10 × 2 = δ− 1
= δ − (δ ) 7 7

= 125Pa
= 0.125KPa 8
(δ ) 7
8

7 δ
Q.132 (b) = δ − ( δ) =
Transitional flow depends upon 8 8
Reynolds number
∴ Re depends on density of fluid Q.139 (b)
δ δ
Velocity of fluid  u  y
Viscosity of fluid
δ*
= ∫0 1 − v  dy= ∫ 1 − δ  dy
0
Diameter of pipe δ
y2  δ δ
= y−  = δ− =
Q.133 (b) 2δ  0 2 2
1 Q.140 (d)
hL ∝
D5 Turbulent shear stress

© Copyright Reserved by Gateflix.in No part of this material should be copied or reproduced without permission
du kinetic energy of water discharged
τ = ρu ' v ' ⇒ u ' = v ' = l
 
dy by the runner into the pressure
2 energy so that water is discharged
 du  to the tail race with a positive
⇒τ=ρl   2

 dy  pressure head to avoid cavitation.

Q.141 (c) Q.147 (b)


dψ dψ Laws of fluid friction for laminar
u=− ;v = flow are as follows:
dy dx
Head loss is
δ
⇒u= − ( 3xy ) ⇒ u = −3x  i) proportional to the velocity of
δy flow
⇒ at(2,3) ⇒ u =−3 × 2 =−6 unit ii) independent of the pressure
∂ iii) proportional to the area of
=v (3xy)=
⇒ v 3y surface in contact
∂x
⇒ at(2,3)v = 3 × 3 = 9 unit iv) independent of the nature of the
surface in contact.
= u 2 + v2
Resultant v) greatly influenced by the variation
= 36 + 81 = 10.82 unit of the temperature of the flowing
fluid.
Q.142 (d) So, option (b) is wrong and valid for
turbulent flow.
Q.143 (c)
ρVD vD Q.148 (b)
Re =×
2 105 = = Tensile stress in the pipe wall
μ υ
= Circumferential stress in pipe wall
V × 40 ×10−3
⇒= =
Pd
Where, P=ρgH=980000 N/m2
1.5 × 10−5 2t
3 ×103 980000 ×10
= = 75m / sec ∴ Tensile stress =
4 2 × 9 ×10−3
= 544.4 × 106 N/m2
Q.144 (a) = 544.4 MN/m2
= 544.4 MPa
Q.145 (a)
Q.149 (a)
Q.146 (a) Operating characteristic curve of a
At inlet of reaction turbine water is centrifugal pump
admitted with high pressure head
(above atmospheric pressure) with
some kinetic energy. The pressure
head of water gradually decreases
from inlet to exit. Finally water leaves
the runner with low pressure and
small kinetic energy to tail race. Quit
often, the pressure head of the water
at turbine exit fall below the ambient
pressure. A draft tube attached to Q.150 (d)
the exit of the turbine, working like For head H ∝ N2
a diffuser converts the remaining

© Copyright Reserved by Gateflix.in No part of this material should be copied or reproduced without permission
H 2 N 22 Kaplan Turbine: It is low head (less
∴ = than 60m) and high discharge
H1 N12
turbine.
∴ H2 = 40 m
For power P ∝ N3 Q.155 (b)
P N3 Filling the pump casing and the
∴ 2 =23
P N1 suction pipe with the liquid before
∴P2 = 8 kW it is started is known as priming.
Unless the centrifugal pump is
Q.151 (c) primed, it will not start as there is
Closed impeller provides better not centrifugal head impressed on
guidance for the liquid and most the liquid. Positive displacement
suited when the liquid to be pumped type reciprocating or rotary pump
is pure and comparatively free from do not needed priming because the
debris. Hence acid is the suitable air if any in the cylinder or casing is
candidate. driven out by the piston or vanes.
Semi-open type impeller is suitable
even if the liquids are charged with Q.156 (d)
some debris. Hence sandy water is −dp  R 
τ=
 
the suitable this for semi-open dx  2 
impellers. Open impellers are useful ∵ Pressure drop is there
in the pumping liquids containing ∴ dp = -70 KPa
suspended solid matter such as paper
−[−70 ×103 ] 0.3
pulp, sewage and water containing =∴τ = × 350Pa
sand or grit. These impellers are less 30 2
liable to clog when handling liquids
charged with a large quantity of
debris.

Q.152 (a)
Ux
5 ×105 =
υ
x
5=
×105 = 0.43m
0.86 ×10−6

Q.153 (d)
F F
  = 
 A  Punger  A  ram
F 100 ×10
=
π / 4(1.5) 2
π / 4(15) 2
F = 10 N

Q.154 (d)
Pelton wheel: it is high head and low
discharge turbine.
Francis turbine: it is medium head
(60-250m) and medium discharge
turbine.

© Copyright Reserved by Gateflix.in No part of this material should be copied or reproduced without permission

You might also like